Pharm Final Mod 1-13

Lakukan tugas rumah & ujian kamu dengan baik sekarang menggunakan Quizwiz!

(Mod 11) A patient with type 1 diabetes mellitus is ordered insulin therapy once daily to be administered at bedtime. What is the type of insulin the patient is most likely receiving? a. Insulin glargine b. Lente insulin c. Lispro insulin d. Regular insulin

ANS: A Insulin glargine (Lantus) is long-acting insulin with an onset of 1 hour. It is evenly distributed over a 24-hour duration of action; thus, it is administered once a day, usually at bedtime. Intermediate-acting insulins include neutral-protamine-Hagedorn (NPH), Lente, and Humulin N. Rapid-acting insulins include insulin lispro. Regular insulin is short acting.

7. An 80-year-old patient is being treated for an infection. An order for which type of antibiotic would cause concern for the nurse caring for this patient? a. Aminoglycoside b. Cephalosporin c. Penicillin d. Sulfonamide

ANS: A a. Aminoglycoside Penicillins, cephalosporins, tetracyclines, and sulfonamides are normally considered safe for the older adult. Amino glycosides are excreted in the urine and are not usually prescribed for patients older than 75 years.

(Mod 6) 19. Which topical antifungal medication is used to treat vaginal candidiasis? a. Haloprogin (Halotex) b. Miconazole (Monistat) c. Oxiconazole (Oxistat) d. Terbenafine HCl (Lamisil)

ANS: B b. Miconazole (Monistat) Topical miconazole is used to treat vaginal candidiasis.

(Mod 9) 2. A patient who is taking aspirin for arthritis pain asks the nurse why it also causes gastrointestinal upset. The nurse understands that this is because aspirin a. increases gastrointestinal secretions. b. increases hypersensitivity reactions. c. inhibits both COX-1 and COX-2. d. is an acidic compound.

ANS: C c. inhibits both COX-1 and COX-2. Aspirin is a COX-1 and COX-2 inhibitor. COX-1 protects the stomach lining, so when it is inhibited, gastric upset occurs. Aspirin does not increase gastrointestinal secretions or hypersensitivity reactions. It is a weak acid.

(Mod 8) 1. The nurse is preparing to administer a drug and learns that it is an indirect-acting cholinergic agonist. The nurse understands that this drug a. acts on muscarinic receptors. b. acts on nicotinic receptors. c. inhibits cholinesterase. d. inhibits cholinergic receptors.

ANS: C c. inhibits cholinesterase. Agents that inhibit cholinesterase, which is the enzyme that destroys acetylcholine, indirectly enhance the actions of acetylcholine.

(Mod 6) 2. A patient is diagnosed with mycoplasma pneumonia. Which antibiotic will the nurse expect the provider to order to treat this infection? a. Azithromycin (Zithromax) b. Clarithromycin (Biaxin) c. Erythromycin (E-Mycin) d. Fidaxomicin (Dificid)

ANS: C c. Erythromycin (E-Mycin) Erythromycin is the drug of choice for treating mycoplasma pneumonia.

(Mod 8) 5. The nurse is caring for a patient who is receiving intravenous dopamine (Intropin). The nurse notes erythema and swelling at the IV insertion site. What is the nurse's initial action? a. Apply warm soaks to the area. b. Monitor the patient closely for hypertension. c. Obtain an order for an electrocardiogram. d. Notify the provider of a need for phentolamine mesylate (Regitine).

ANS: D d. Notify the provider of a need for phentolamine mesylate (Regitine). Extravasation of dopamine causes tissue necrosis; if extravasation occurs, the antidote phentolamine mesylate should be infiltrated into the area. (book has similar question: same thinking, giving phentolamine counteracts the vasoconstrictive effects of DA)

(Mod 8) 4. A patient experiences severe muscle weakness, and the provider orders edrophonium bromide (Tensilon). The patient begins to show improved muscle strength within a few minutes after administration of this drug. The nurse anticipates the provider will order which drug? a. Atropine sulfate b. Edrophonium bromide (Tensilon) c. Intravenous immune globulin (IVIG) d. Pyridostigmine HCl (Mestinon)

ANS: D d. Pyridostigmine HCl (Mestinon) In this case, edrophonium is used to diagnose myasthenia gravis. Since symptoms improved with the AChE inhibitor, the patient will benefit from a longer-acting AChE inhibitor such as pyridostigmine. Atropine is given for AChE inhibitor overdose. Edrophonium is very short-acting, so it will not be used for treatment. IVIG is used when other AChE inhibitors fail.

(Mod 10) 2. A nurse is assessing a client who has alcohol use disorder and is experiencing withdrawal. Which of the following is an expected finding. Select All That Apply: A.) Bradycardia B) Fine tremors of both hands C.) Hypotension D.) Vomiting E.) restlessness

B) fine tremors of both hands D) vomiting E) restlessness

(Mod 11) A patient has administered regular insulin 30 minutes prior but has not received a breakfast tray. The patient is experiencing nervousness and tremors. What is the nurse's first action? a. Administer glucagon. b. Give the patient orange juice. c. Notify the kitchen to deliver the tray. d. Perform bedside glucose testing.

b. Give the patient orange juice. The patient is symptomatic and has hypoglycemia. The nurse should give orange juice. Glucagon is given for patients unable to ingest carbohydrates. The kitchen should be notified, and bedside glucose testing should be performed, but only after the patient is given carbohydrates.

3. To assist an older, confused patient to adhere to a multidrug regimen, the nurse will provide which recommendation? a. Avoid the use of over-the-counter medications. b. Bring all medications to each clinic visit. c. Review the manufacturer's information insert about each medication. d. Save money by getting each drug at the pharmacy with the lowest price.

ANS: B b. Bring all medications to each clinic visit. Patients who take multiple medications should be advised to bring medications to each clinic visit. Patients may take OTC medications to each clinic visit. Patients may take OTC medications as long as those are included in the list of medications reviewed by the provider. Manufacturers' inserts provide an overwhelming amount of information. Patients should be advised to use only one pharmacy.

(Mod 12) 6. The nurse is preparing to care for a Native-American patient who has hypertension. The nurse understands that which antihypertensive medication would be most effective in this patient? a. Acebutolol (Sectral) b. Captopril (Capoten) c. Carteolol HCl (Cartrol) d. Metoprolol (Lopressor)

ANS: B b. Captopril (Capoten) Captopril is an angiotensin II inhibitor. Native-American patients do not respond well to beta blockers. Acebutolol, carteolol, and metoprolol are all beta blockers.

(Mod 12) 13. The nurse is caring for a patient who will begin taking captopril (Capoten) for hypertension. The nurse reviews the patient's laboratory test results and notes increased BUN and creatinine. Which action will the nurse take? a. Administer the captopril and monitor vital signs. b. Contact the provider to discuss changing to fosinopril (Monopril). c. Obtain an order for intravenous fluids to improve urine output. d. Request an order to add hydrochlorothiazide (HydroDIURIL).

ANS: B b. Contact the provider to discuss changing to fosinopril (Monopril). Patients who have renal insufficiency will not require a decrease in dose with fosinopril, as they would with other angiotensin-converting enzyme (ACE) inhibitors. If captopril is given, it should be given in a reduced dose. Increased IV fluids are not indicated.

6. The nurse is caring for an 80-year-old patient who is taking warfarin (Coumadin). Which action does the nurse understand is important when caring for this patient? a. Encouraging the patient to rise slowly from a sitting position b. Initiating a fall-risk protocol c. Maintaining strict intake and output measures d. Monitoring blood pressure frequently

ANS: B b. Initiating a fall-risk protocol Patients who take anticoagulants have an increased risk of hemorrhage. Older patients have an increased risk of falls that can lead to bleeding complications. Initiating a fall-risk protocol is important. Warfarin does not affect blood pressure and would not cause orthostatic hypotension. Warfarin does not alter urine output.

5. Which assessment is categorized as objective data? a. A list of herbal supplements regularly used b. Lab values associated with drugs the patient is taking c. The ages and relationship to the patient of all household members d. Usual dietary patterns and intake

ANS: B b. Lab values associated with drugs the patient is taking Objectie data are measured and detected by another person and would include lab values. The other examples are subjective data.

(Mod 8) 2. A patient who has asthma is diagnosed with hypertension. The nurse understands that which drug will be safe to give this patient? a. Pindolol (Visken) b. Metoprolol (Lopressor) c. Nadolol (Corgard) d. Propranolol (Inderal)

ANS: B b. Metoprolol (Lopressor) Metoprolol is a selective adrenergic blocker that has a greater affinity for receptors that decrease heart rate and blood pressure and is less likely to cause bronchospasm. The other adrenergic blockers are not selective and can cause bronchoconstriction.

(Mod 12) 14. The nurse is caring for a patient who experiences a rapid rise in blood pressure. The nurse will contact the provider to discuss administering which medication? a. Amlodipine (Norvasc) b. Nifedipine (Procardia) c. Nifedipine extended release (Procardia XL) d. Verapamil (Calan)

ANS: B b. Nifedipine (Procardia) The short-acting nifedipine is used to treat rapid rises in blood pressure but cannot be used for out-patient treatment at high dosages because of an increased risk for sudden cardiac death. The other drugs are not used for rapid rise in BP.

(Mod 8) 5. The nurse is teaching a patient who will begin taking bethanechol (Urecholine). Which statement by the patient indicates a need for further teaching? a. "Excessive sweating is a normal reaction to this medication." b. "Excess salivation is a serious side effect." c. "I should get out of bed slowly while taking this drug." d. "I will not take the drug if my heart rate is less than 60 beats per minute."

ANS: A a. "Excessive sweating is a normal reaction to this medication." Patients taking bethanechol should be instructed to report increased salivation and diaphoresis since they can be early signs of overdosing. They should also be taught to rise slowly to avoid orthostatic hypotension and to hold the drug if their heart rate is low.

(Mod 8) 14. The nurse is caring for a patient whose provider has just ordered a switch from atenolol (Tenormin) to reserpine. When preparing the patient to take this medication, what will the nurse do? a. Ask about herbal supplements. b. Counsel that NSAIDs are safe to take with reserpine. c. Teach about potential side effects of mood elevation and euphoria. d. Tell the patient to expect immediate therapeutic effects.

ANS: A a. Ask about herbal supplements. St. John's wort may antagonize hypotensive effects of reserpine. Reserpine should not be taken with NSAIDs. Side effects include depression, not mood elevation. Therapeutic effects may take 2 to 3 weeks. (answer is from a one liner in book: COMPLEMENTARY AND ALTERNATIVE THERAPIES 15.1)

(Mod 10) A patient arrives in the emergency department in an acute state of alcohol intoxication and reports chronic consumption of "several six packs" of beer every day for the past year. The nurse anticipates administering which medication or treatment? a. Chlordiazepoxide (Librium) b. Disulfiram (Antabuse) c. Gastric lavage d. Vasoconstrictors

ANS: A a. Chlordiazepoxide (Librium) To prevent acute withdrawal and delirium tremens, a long-acting benzodiazepine, such as chlordiazepoxide, is given. Disulfiram would cause an acute drug interaction. Gastric lavage should no longer be performed, and vasoconstrictors are not indicated.

(Mod 9) 1. A patient reports having recurring headaches described as 1 to 2 headaches per day for several weeks. The nurse understands that these headaches are most likely descriptive of which type of headache? a. Cluster headache b. Migraine headache c. Simple headache d. Tension headache

ANS: A a. Cluster headache Cluster headaches reoccur 1 to 3 times daily in a period lasting from approximately 2 weeks to 3 months. Migraine headaches are severe and characterized by an aura prior to the headache. Tension headaches are related to stress.

2. The nurse learns that a patient cannot afford a prescribed medication and enlists the assistance of the social worker and an outside agency to provide medications at a lower cost. Which QSEN competency do the nurse's actions best demonstrate? a. Collaboration and teamwork b. Evidence-based practice c. Patient-centered care d. Quality improvement

ANS: A a. Collaboration and teamwork Collaboration and teamwork involve inter professional communication and shared decision-making to provide patient care.

(Mod 13) The nurse is preparing to administer a first dose of clopidogrel (Plavix) to a patient. As part of the history, the nurse learns that the patient has a previous history of peptic ulcers, diabetes, and hypertension. The nurse understands that it will be necessary to notify the provider and obtain an order for a. a proton pump inhibitor (PPI) medication. b. frequent serum glucose monitoring. c. increased antihypertensive medications. d. nonsteroidal anti-inflammatory medications

ANS: A a. a proton pump inhibitor (PPI) medication. Patients with a previous history of peptic ulcer are at increased risk for gastric bleeding and should take a PPI or histamine2 blocker to prevent this. There is no indication for increased glucose monitoring or an increase in antihypertensive drugs. NSAIDs are contraindicated

(Mod 12) 4. The nurse is caring for an African-American patient who has been taking a beta blocker to treat hypertension for several weeks with only slight improvement in blood pressure. The nurse will contact the provider to discuss a. adding a diuretic medication. b. changing to an ACE inhibitor. c. decreasing the beta blocker dose. d. doubling the beta blocker dose.

ANS: A a. adding a diuretic medication. African Americans do not respond well to beta blockers and ACE inhibitors, but do tend to respond to diuretics and calcium channel blockers. Changing to an ACE inhibitor or altering the beta blocker dose are not indicated. Hypertension in African-American patients can be controlled by combining beta blockers with diuretics.

(Mod 12) 15. The nurse is caring for a 70-year-old patient who has recently begun taking amlodipine (Norvasc) 5 mg/day to control hypertension. The nurse notes mild edema of the patient's ankles, a blood pressure of 130/70 mm Hg, and a heart rate of 80 beats per minute. The patient reports flushing and dizziness. The nurse will notify the provider and a. ask to decrease the dose to 2.5 mg/day. b. discuss twice daily dosing. c. request an order for a diuretic. d. suggest adding propranolol to the regimen

ANS: A a. ask to decrease the dose to 2.5 mg/day. This patient is experiencing side effects of the medication. Elderly patients often require lower doses, so the nurse should ask about a dose reduction. Older adults generally require 2.5 to 5.0 mg/day. Twice daily dosing is not recommended. Unless edema persists, a diuretic is not indicated.

(Mod 6) 12. The nurse is caring for a patient who is receiving a high dose of tetracycline (Sumycin). Which laboratory values will the nurse expect to monitor while caring for this patient? a. Blood urea nitrogen (BUN) and creatinine levels b. Complete blood counts c. Electrolytes d. Liver enzyme levels

ANS: A a. Blood urea nitrogen (BUN) and creatinine levels High doses of tetracyclines can lead to nephrotoxicity, especially when given along with other nephrotoxic drugs. Renal function tests should be performed to monitor for nephrotoxicity.

(Mod 6) 14. A patient is receiving high doses of a cephalosporin. Which laboratory values will this patient's nurse monitor closely? a. Blood urea nitrogen (BUN), serum creatinine, and liver function tests b. Complete blood count and electrolytes c. Serum calcium and magnesium d. Serum glucose and lipids

ANS: A a. Blood urea nitrogen (BUN), serum creatinine, and liver function tests Cefazolin will produce an increase in the patient's BUN, creatinine, AST, ALT, ALP, LDH, and bilirubin.

(Mod 6) 9. A patient taking trimethoprim-sulfamethoxazole (TMP-SMX) to treat a urinary tract infection complains of a sore throat. The nurse will contact the provider to request an order for which laboratory test(s)? a. Complete blood count with differential b. Throat culture c. Urinalysis d. Coagulation studies

ANS: A a. Complete blood count with differential A sore throat can indicate a life-threatening anemia, so a complete blood count with differential should be ordered.

(Mod 12) 8. The nurse is performing an assessment on a patient who will begin taking propranolol (Inderal) to treat hypertension. The nurse learns that the patient has a history of asthma and diabetes. The nurse will take which action? a. Administer the medication and monitor the patient's serum glucose. b. Contact the provider to discuss another type antihypertensive medication. c. Request an order for renal function tests prior to administering this drug. d. Teach the patient about the risks of combining herbal medications with this drug.

ANS: B b. Contact the provider to discuss another type antihypertensive medication. Patients with chronic lung disease are at risk for bronchospasm with beta blockers, especially those like propranolol which are non-selective. Beta blockers, with the exception of carvedilol, also decrease the efficacy of many oral antidiabetic medications. The nurse should discuss a change in medications to one that does not carry this risk.

(Mod 7) 13. The nurse is caring for a patient who has been taking an NSAID for 4 weeks for osteoarthritis. The patient reports decreased pain, but the nurse notes continued swelling of the affected joints. The nurse will perform which action? a. Assess the patient for drug-seeking behaviors. b. Notify the provider that the drug is not effective. c. Reassure the patient that swelling will decrease eventually. d. Remind the patient that this drug is given for pain only.

ANS: B b. Notify the provider that the drug is not effective. This medication is effective for both pain and swelling. After 4 weeks, there should be some decrease in swelling, so the nurse should report that this medication is ineffective. There is no indication that this patient is seeking an opioid analgesic. The drug should be effective within several weeks. NSAIDs are given for pain and swelling.

(Mod 6) 15. A patient who is taking metronidazole (Flagyl) reports reddish-brown urine. Which action will the nurse take? a. Obtain an order for BUN and creatinine levels. b. Reassure the patient that this is a harmless effect. c. Request an order for a urinalysis. d. Test her urine for occult blood.

ANS: B b. Reassure the patient that this is a harmless effect. Reddish-brown urine is a harmless side effect of metronidazole and is not cause for concern.

(Mod 6) 2. A patient who has chronic liver disease reports contact with a person who has tuberculosis (TB). The nurse will counsel this patient to contact the provider to discuss a. a chest x-ray. b. a TB skin test. c. liver function tests (LFTs). d. prophylactic antitubercular drugs.

ANS: B b. a TB skin test. Patients who have exposure to TB should have a TB skin test. A chest x-ray is performed if the skin test is positive. LFTs do not need to be done simply because of TB exposure. This patient is not a candidate for antitubercular drug prophylaxis.

(Mod 6) 4. A patient who will begin taking trimethoprim-sulfamethoxazole (TMP-SMX) asks the nurse why the combination drug is necessary. The nurse will explain that the combination is used to a. broaden the antibacterial spectrum. b. decrease bacterial resistance. c. improve the taste. d. minimize toxic effects.

ANS: B b. decrease bacterial resistance. The combination drug is used to decrease bacterial resistance to sulfonamides. It does not broaden the spectrum, improve the taste, or decrease toxicity.

(Mod 6) 1. The nurse caring for a patient who will receive penicillin to treat an infection asks the patient about previous drug reactions. The patient reports having had a rash when taking amoxicillin (Amoxil). The nurse will contact the provider to a. discuss giving a smaller dose of penicillin. b. discuss using erythromycin (E-mycin) instead of penicillin. c. request an order for diphenhydramine (Benadryl). d. suggest that the patient receive cefuroxime (Ceftin).

ANS: B b. discuss using erythromycin (E-mycin) instead of penicillin. Erythromycin is the drug of choice when penicillin is not an option. Giving smaller doses of penicillin does not prevent hypersensitivity reactions. Benadryl is useful when a hypersensitivity reaction has occurred. A small percentage of patients allergic to penicillins may be hypersensitive to cephalosporins.

(Mod 6) 9. A patient who has completed the first phase of a three-drug regimen for tuberculosis has a positive sputum acid-bacilli test. The nurse will tell the patient that a. drug resistance has probably occurred. b. it may be another month before this test is negative. c. the provider will change the pyrazinamide to ethambutol. d. there may be a need to remain in the first phase of therapy for several weeks.

ANS: B b. it may be another month before this test is negative. The goal is for the patient's sputum test to be negative 2 to 3 months after the therapy. The positive test does not indicate drug resistance. The provider will not change the drugs or keep the patient in the first phase longer than planned.

(Mod 7) 3. An adolescent female has dysmenorrhea associated with heavy menstrual periods. The patient's provider has recommended ibuprofen (Motrin). When teaching this patient about this drug, the nurse will tell her that ibuprofen a. may decrease the effectiveness of oral contraceptive pills. b. may increase bleeding during her period. c. should be taken on an empty stomach to increase absorption. d. will decrease the duration of her periods.

ANS: B b. may increase bleeding during her period. When nonsteroidal antiinflammatory drugs (NSAIDs) are used to treat dysmenorrhea, excess bleeding may occur during the first 2 days of a period. NSAIDs do not decrease the effect of OCPs. NSAIDs are irritating to the stomach, so patients should take with food or a full glass of water. NSAIDs will not decrease the duration of periods.

(Mod 6) 13. A patient who is taking acyclovir (Zovirax) to treat an oral HSV-1 infection asks the nurse why oral care is so important. The nurse will tell the patient that meticulous oral care helps to a. minimize transmission of disease. b. prevent gingival hyperplasia. c. reduce viral resistance to the drug. d. shorten the duration of drug therapy

ANS: B b. prevent gingival hyperplasia. Good oral care can prevent gingival hyperplasia in patients with HSV-1.

(Mod 6) 1. Which is a characteristic that distinguishes sulfonamides from other drugs used to treat bacterial infection? a. Sulfonamides are bactericidal. b. Sulfonamides are derived from biologic substances. c. Sulfonamides have antifungal and antiviral properties. d. Sulfonamides increase bacterial synthesis of folic acid.

ANS: B (there's actually no right answer) b. Sulfonamides are derived from biologic substances. Sulfonamides are bacteriostatic, not bactericidal. They are not derived from biologic substances. They are not antifungals or antivirals. They act by decreasing bacterial synthesis of folic acid.

(Mod 6) 4. A patient is being treated with isoniazid (INH), rifampin, and pyrazinamide in phase I of treatment for tuberculosis. The organism develops resistance to isoniazid. Which drug will the nurse anticipate the provider will order to replace the isoniazid? a. Ciprofloxacin (Cipro) b. Ethambutol (Myambutol) c. Kanamycin d. Streptomycin sulfate

ANS: B b. Ethambutol (Myambutol) If there is bacterial resistance to isoniazid, the first phase may be changed to ethambutol, rifampin, and pyrazinamide. Ciprofloxacin, kanamycin, and streptomycin are not generally first-line antitubercular drugs.

(Mod 8) 18. Cholinergic drugs have specific effects on the body. What are the actions of cholinergic medications? (Select all that apply.) a. Dilate pupils b. Decrease heart rate c. Stimulate gastric muscle d. Dilate blood vessels e. Dilate bronchioles f. Increase salivation g. Constrict pupils

ANS: B, C, D, F, G b. Decrease heart rate c. Stimulate gastric muscle d. Dilate blood vessels f. Increase salivation g. Constrict pupils Decreasing heart rate, stimulating gastric muscles, dilating blood vessels, increasing salivation, and constricting pupils are actions of the cholinergic drugs.

(Mod 9) 17. The nurse checks on a patient who has received sumatriptan (Imitrex) for treatment of a migraine headache. The patient reports moderate improvement in headache pain and reports feeling dizzy. The nurse notes a blood pressure of 160/85 mm Hg. Which action by the nurse is correct? a. Notify the provider of the dizziness. b. Notify the provider of the increased blood pressure. c. Plan to administer a second dose in 1 hour. d. Request an order for intranasal sumatriptan.

ANS: B b. Notify the provider of the increased blood pressure. Triptans can cause increased blood pressure, which is an adverse drug reaction and should be reported to the provider. Dizziness is a common side effect but not potentially life-threatening. The second dose should not be given if the patient is experiencing elevated blood pressure. Intranasal sumatriptan has the same adverse effects.

(Mod 13) A patient has been receiving intravenous heparin. When laboratory tests are drawn, the nurse has difficulty stopping bleeding at the puncture site. The patient has bloody stools and is reporting abdominal pain. The nurse notes elevated partial thromboplastin time (PTT) and activated PPT (aPTT). Which action will the nurse perform? a. Ask for an order for oral warfarin (Coumadin). b. Obtain an order for protamine sulfate. c. Request an order for vitamin K. d. Suggest that the patient receive subcutaneous heparin.

ANS: B b. Obtain an order for protamine sulfate. Protamine sulfate is given as an antidote to heparin when patient's clotting times are elevated. Oral warfarin will not stop the anticoagulant effects of heparin. Vitamin K is used as an antidote for warfarin. Administering heparin by another route is not indicated when there is a need to reverse the effects of heparin.

(Mod 10) 1.A patient who smoked 1 pack of cigarettes a day for 2 years abruptly stopped smoking 2 days ago. The nurse performing an assessment on the patient identifies manifestations of nicotine withdrawal as including A. increased appetite and blood pressure. B. restlessness and increased blood pressure. C. depression and decreased blood pressure. D. nausea, confusion, and seizures.

ANS: C C. depression and decreased blood pressure. Nicotine withdrawal symptoms include nicotine craving, restlessness, depression, hyperirritability, headache, insomnia, decreased blood pressure, decreased heart rate, and increased appetite.

(Mod 10) 3. A patient with a known opioid addiction is to have surgery. In planning the patient's postoperative pain management, the nurse will A. withhold opioid medications. B. treat the addiction before managing pain. C. provide pain management as needed. D. anticipate that the patient will experience less pain.

ANS: C C. provide pain management as needed. All patients have the right to receive pain management therapies. The patient history of opioid addiction should have no bearing on this right. There is no evidence that providing opioid analgesia to patients addicted to these drugs will in any way worsen their addictive disease.

(Mod 8) 4. The nurse administers bethanechol (Urecholine) to a patient to treat urinary retention. After 30 minutes, the patient voids 800 mL of urine and reports having a loose stool but no cramping or gastrointestinal pain. The patient's blood pressure is 110/70 mm Hg. The nurse will perform which action? a. Notify the provider of bethanechol adverse effects. b. Record the urine output and the blood pressure and continue to monitor. c. Request an order for intravenous atropine sulfate. d. Suggest another dose of bethanechol to the provider.

ANS: B b. Record the urine output and the blood pressure and continue to monitor. The patient is exhibiting desired effects and mild side effects of bethanechol, so the nurse should record information and continue to monitor the patient. There is no need to notify the provider, give an antidote, or repeat the dose.

(Mod 9) 10. The nurse assumes care of a patient in the post-anesthesia care unit (PACU). The patient had abdominal surgery and is receiving intravenous morphine sulfate for pain. The patient is asleep and has not voided since prior to surgery. The nurse assesses a respiratory rate of 10 breaths per minute and notes hypoactive bowel sounds. The nurse will contact the surgeon to report which condition? a. Paralytic ileus b. Respiratory depression c. Somnolence d. Urinary retention

ANS: B b. Respiratory depression The patient's respiratory rate of 10 breaths per minute is lower than normal and is a sign of respiratory depression, which is a common adverse effect of opioid analgesics. The other effects may occur with opioids but are also not expected this soon after abdominal surgery.

(Mod 8) 7. The nurse is caring for a patient who will begin taking atenolol (Tenormin). What information will the nurse include when teaching the patient about taking this medication? a. The drug must be taken twice daily. b. The patient must rise slowly from a chair or bed. c. The medication is safe to take during pregnancy. d. Use NSAIDs as needed for mild to moderate pain.

ANS: B b. The patient must rise slowly from a chair or bed. The side effects commonly associated with beta blockers include bradycardia, hypotension, and dizziness. Patients should be instructed to use caution when rising from a sitting or lying position to avoid orthostatic hypotension. Atenolol may be taken once daily. Atenolol is contraindicated in the pregnant patient. NSAIDs decrease the effects of beta blockers and should be avoided.

7. The provider has ordered that vitamin D drops be given to a newborn. Based on the knowledge of drug distribution in infants, the nurse understands that the infant may need a. a higher dose. b. a lower dose. c. less frequent dosing. d. more frequent dosing.

ANS: B b. a lower dose. Neonates and young infants tend to have less body fat than older children, meaning that they need less of fat-soluble medications since these medications won't be found in fat tissue. Higher doses would lead to drug toxicity. Body fat does not affect the frequency of dosing.

(Mod 8) 16. The nurse is performing an admission assessment on a patient who has recently begun taking reserpine. The patient reports using St. John's wort. The nurse anticipates that the patient will have a. hypotension. b. hypertension. c. bradycardia. d. tachycardia.

ANS: B b. hypertension. St. John's wort antagonizes the hypotensive effects of reserpine, causing hypertension. (answer is from a one liner in book: COMPLEMENTARY AND ALTERNATIVE THERAPIES 15.1)

(Mod 9) 1. A nursing student asks how nonsteroidal antiinflammatory drugs (NSAIDs) work to suppress inflammation and reduce pain. The nurse will explain that NSAIDs a. exert direct actions to cause relaxation of smooth muscle. b. inhibit cyclooxygenase that is necessary for prostaglandin synthesis. c. interfere with neuronal pathways associated with prostaglandin action. d. suppress prostaglandin activity by blocking tissue receptor sites.

ANS: B b. inhibit cyclooxygenase that is necessary for prostaglandin synthesis. NSAIDs act by inhibiting COX-1 and COX-2 to help block prostaglandin synthesis. They do not have direct action on tissues, nor do they interfere with chemical receptor sites or neuronal pathways.

(Mod 9) 7. An adolescent female has dysmenorrhea associated with heavy menstrual periods. The patient's provider has recommended ibuprofen (Motrin). When teaching this patient about this drug, the nurse will tell her that ibuprofen a. may decrease the effectiveness of oral contraceptive pills. b. may increase bleeding during her period. c. should be taken on an empty stomach to increase absorption. d. will decrease the duration of her periods.

ANS: B b. may increase bleeding during her period. When nonsteroidal antiinflammatory drugs (NSAIDs) are used to treat dysmenorrhea, excess bleeding may occur during the first 2 days of a period. NSAIDs do not decrease the effect of OCPs. NSAIDs are irritating to the stomach, so patients should take with food or a full glass of water. NSAIDs will not decrease the duration of periods.

12. A patient who is diagnosed with atrial fibrillation is to begin taking warfarin (Coumadin). The patient refuses to take the medication because "it is rat poison." After the nurse provides teaching, the patient still refuses. What action will the nurse take? a. Hold the dose and document the patient's refusal. b. Hold the dose, notify the provider of the situation, and document these actions. c. Put the medication in the patient's food. d. Tell the patient that the drug is necessary for treatment.

ANS: B b.Hold the dose, notify the provider of the situation, and document these actions. Patients have a right to refuse medications, but the provider should be notified if omitting the medication can have serious effects. According to the principle of autonomy, it is unethical to put a medication in a patient's food without their knowledge.

3. The nurse is teaching a patient who will begin taking ciprofloxacin. What instruction will the nurse include when teaching this patient about this drug? a. "Do not take this medication with oral contraceptive pills." b. "Take at least 1 hour after or 2 hours before taking antacids." c. "Take in the morning with your multivitamin tablet." d. "Take with milk to reduce gastric upset."

ANS: B b. "Take at least 1 hour after or 2 hours before taking antacids." Dairy products, multivitamins, and antacids should be avoided 1 hour before and 2 hours after taking ciprofloxacin because these products contain divalent cations that form a drug complex that prevents absorption of the ciprofloxacin.

15. If a drug has a half-life of 12 hours and is given twice daily starting at 0800 on a Monday, when will a steady state be achieved? a. 0800 on Tuesday b. 0800 on Wednesday c. 0800 on Thursday d. 0800 on Friday

ANS: B b. 0800 on Wednesday Steady-state levels occur at 3 to 5 half-lives. Wednesday at 0800 is 4 half-lives from the original dose.

1. A patient has been taking a drug for several years and tells the nurse it is no longer working. The nurse learns that the patient has recently begun taking an over-the-counter antacid medication. What does the nurse suspect is occurring? a. An adverse drug reaction b. A drug interaction c. Drug incompatibility d. Drug tolerance

ANS: B b. A drug interaction Drug interactions are an altered or modified action or effect of a drug as a result of interaction with one or more other drugs. An adverse drug reaction can occur with one or more drugs and has effects ranging from mild to severe toxicity. Drug incompatibility is a chemical reaction of two or more drugs that occurs in vitro. Drug tolerance is the development of reduced response to a medication over time.

16. The nurse is preparing to administer a drug that is ordered to be given twice daily. The nurse reviews the medication information and learns that the drug has a half-life of 24 hours. What will the nurse do next? a. Administer the medication as ordered. b. Contact the provider to discuss daily dosing. c. Discuss every-other-day dosing with the provider. d. Hold the medication and notify the provider.

ANS: B b. Contact the provider to discuss daily dosing. A drug with a longer half-life should be given at longer intervals to avoid drug toxicity.

4. A patient who takes a drug that undergoes gastric absorption will begin taking an opioid analgesic after sustaining an injury in a motor vehicle accident. The nurse will observe the patient closely for which effects? a. Decreased effects of the first drug b. Increased effects of the first drug c. Decreased effects of the narcotic d. Increased effects of the narcotic

ANS: B b. Increased effects of the first drug Opioids slow gastric emptying, allowing more time for drugs absorbed in the stomach to be absorbed. The nurse should expect increased effects of the first drug.

8. A patient is taking phenytoin to prevent seizures. The nurse knows that phenytoin is highly protein-bound and has sedative side effects. The nurse reviews the patient's chart and notes a low serum albumin. The nurse will notify the provider and observe the patient for which effects? a. Decreased sedative effects b. Increased sedative effects c. Increased seizures d. No change in effects

ANS: B b. Increased sedative effects Phenytoin is protein-bound. When patients have a low serum albumin, there are fewer protein-binding sites, leaving more free drug in the system. The nurse should expect an increase in sedative side effects.

19. The nurse understands that the length of time needed for a drug to reach the minimum effective concentration (MEC) is the a. duration of action. b. onset of action. c. peak action time. d. time response curve.

ANS: B b. onset of action. The onset of action is the time it takes to reach the MEC. Duration of action is the length of time a drug has a pharmacologic effect. Peak action time occurs when the drug reaches its highest blood level. The time response curve is an evaluation of the other three measures.

5. The nurse is preparing to administer an oral medication that is water-soluble. The nurse understands that this drug a. must be taken on an empty stomach. b. requires active transport for absorption. c. should be taken with fatty foods. d. will readily diffuse into the gastrointestinal tract.

ANS: B b. requires active transport for absorption. Water-soluble drugs require a carrier enzyme or protein to pass through the GI membrane.

(Mod 7) 10. The nurse provides teaching for a patient who will begin taking indomethacin (Indocin) to treat rheumatoid arthritis. Which statement by the patient indicates a need for further teaching? a. "I should limit sodium intake while taking this drug." b. "I should take indomethacin on an empty stomach." c. "I will need to check my blood pressure frequently." d. "I will take the medication twice daily."

ANS: B b. "I should take indomethacin on an empty stomach." Indomethacin is very irritating to the stomach and should be taken with food. It can cause sodium retention and elevated blood pressure, so patients should limit sodium intake. The medication is taken twice daily.

(Mod 7) 19. The nurse provides teaching for a patient who will begin taking allopurinol. Which statement by the patient indicates understanding of the teaching? a. "I should increase my vitamin C intake." b. "I will get yearly eye exams." c. "I will increase my protein intake." d. "I will limit fluids to prevent edema."

ANS: B b. "I will get yearly eye exams." Patients taking allopurinol can have visual changes with prolonged use and should have yearly eye exams. It is not necessary to increase vitamin C. Protein can increase purine intake, which is not recommended. Patients should consume extra fluids.

(Mod 6) 16. A patient is diagnosed with histoplasmosis and will begin taking ketoconazole. What information will the nurse include when teaching this patient about this medication? a. "Take the medicine twice daily." b. "Take the medication with food." c. "You may consume small amounts of alcohol." d. "You will not need lab tests while taking this drug."

ANS: B b. "Take the medication with food." Ketoconazole should be taken with food. It is administered once daily. Patients taking antifungals should not consume alcohol. Antifungals can cause liver and renal toxicity, so patients will need lab monitoring.

(Mod 7) 14. The nurse is discussing celecoxib (Celebrex) with a patient who will use the drug to treat dysmenorrhea. What information will the nurse include in teaching? a. "Do not take the medication during the first 2 days of your period." b. "The initial dose will be twice the amount of subsequent doses." c. "Take this medication with food to minimize gastrointestinal upset." d. "Take the drug on a regular basis to prevent dysmenorrhea."

ANS: B b. "The initial dose will be twice the amount of subsequent doses." The initial dose of Celebrex is twice that of subsequent doses. The medication should not be taken just before a period. It does not need to be taken with food. It is taken as needed.

(Mod 7) 8. The nurse is teaching a patient about using high-dose aspirin to treat arthritis. What information will the nurse include when teaching this patient? a. "A normal serum aspirin level is between 30 and 40 mg/dL." b. "You may need to stop taking this drug a week prior to surgery." c. "You will need to monitor aspirin levels if you are also taking warfarin." d. "Your stools may become dark, but this is a harmless side effect."

ANS: B b. "You may need to stop taking this drug a week prior to surgery." Aspirin should be discontinued prior to surgery to avoid prolonged bleeding time. A normal serum level is 15 to 30 mg/dL. Patients taking warfarin and aspirin will have increased amounts of warfarin, so the INR will need to be monitored. Tarry stools are a symptom of gastrointestinal bleeding and should be reported.

(Mod 6) 2. A nurse whose last flu vaccine was 1 year prior is exposed to the influenza A virus. The occupational health nurse will administer which medication? a. Acyclovir (Zovirax) b. Amantadine HCl (Symmetrel) c. Influenza vaccine d. Oseltamivir phosphate (Tamiflu)

ANS: B b. Amantadine HCl (Symmetrel) The primary use for amantadine is prophylaxis against influenza A. Acyclovir is used to treat herpes virus. Oseltamivir phosphate (Tamiflu) is to be taken once flu symptoms appear.

(Mod 6) 12. A patient will begin taking streptomycin as part of the medication regimen to treat tuberculosis. Before administering this medication, the nurse will review which laboratory values in the patient's medical record? a. Complete blood count (CBC) with differential white cell count b. Blood urea nitrogen (BUN) and creatinine c. Potassium and magnesium levels d. Serum fasting glucose

ANS: B b. Blood urea nitrogen (BUN) and creatinine Streptomycin can cause significant renal toxicity.

(Mod 7) 15. The nurse is caring for a patient who has rheumatoid arthritis and who is receiving infliximab (Remicade) IV every 8 weeks. Which laboratory test will the nurse anticipate that this patient will need? a. Calcium level b. Complete blood count c. Electrolytes d. Potassium

ANS: B b. Complete blood count Infliximab is an immunomodulator and can cause agranulocytosis, so patients should have regular CBC evaluation.

(Mod 6) 15. The patient has been ordered treatment with rimantadine (Flumadine). The patient has renal impairment. The nurse anticipates what change to the dose of medication? a. Increased b. Decreased c. Unchanged d. Held

ANS: B b. Decreased The dosage of the medication will be decreased when the patient has renal impairment.

(Mod 6) 4. The nurse receives an order to administer a purine nucleoside antiviral medication. The nurse understands that this medication treats which type of virus? a. Hepatitis virus b. Herpes virus c. HIV d. Influenza virus

ANS: B b. Herpes virus Purine nucleosides, such as acyclovir, are used to treat herpes simplex viruses 1 and 2, herpes zoster virus, varicella-zoster virus, and cytomegalovirus.

(Mod 6) 13. The nurse is preparing to administer an intravenous polymyxin antibiotic. The patient reports dizziness along with numbness and tingling of the hands and feet. The nurse will perform which action? a. Administer the drug since these are harmless side effects. b. Hold the drug and notify the provider of these adverse reactions. c. Obtain an order for an oral form of this medication. d. Request an order for serum electrolytes.

ANS: B b. Hold the drug and notify the provider of these adverse reactions. Polymyxins can cause nephrotoxicity and neurotoxicity. This patient has signs of neurotoxicity, so the nurse should notify the provider. These effects are generally reversible when the drug is discontinued. It is not correct to administer the drug when these symptoms are present. Polymyxins are not absorbed orally. Serum electrolytes are not indicated.

(Mod 6) 13. A female patient will receive doxycycline to treat a sexually transmitted illness (STI). What information will the nurse include when teaching this patient about this medication? a. Nausea and vomiting are uncommon adverse effects. b. The drug may cause possible teratogenic effects. c. Increase intake of dairy products with each dose of this medication. d. Use a backup method of contraception if taking oral contraceptives.

ANS: D d. Use a backup method of contraception if taking oral contraceptives. The desired action of oral contraceptives can be lessened when taken with tetracyclines, so patients taking oral contraceptives should be advised to use a backup contraception method while taking tetracyclines. Nausea and vomiting are common adverse effects. Doxycycline should not be taken with dairy products. Tetracycline may cause teratogenic effects.

(Mod 6) 6. The nurse is preparing to give a dose of trimethoprim-sulfamethoxazole (TMP-SMX) and learns that the patient takes warfarin (Coumadin). The nurse will request an order for a. a decreased dose of TMP-SMX. b. a different antibiotic. c. an increased dose of warfarin. d. coagulation studies.

ANS: D d. coagulation studies. Sulfonamides can increase the anticoagulant effects of warfarin. The nurse should request INR levels. An increased dose of warfarin would likely lead to toxicity and to undesirable anticoagulation.

(Mod 6) 11. The nurse is preparing to give trimethoprim-sulfamethoxazole (TMP-SMX) to a patient and notes a petechial rash on the patient's extremities. The nurse will perform which action? a. Hold the dose and notify the provider. b. Request an order for a blood glucose level. c. Request an order for a BUN and creatinine level. d. Request an order for diphenhydramine (Benadryl).

ANS:A a. Hold the dose and notify the provider. A petechial rash can indicate a severe adverse reaction and should be reported.

(Mod 11) 7. The patient experiences the Somogyi effect. Which statement regarding the Somogyi effect does the nurse identify as being true? a. This is a hyperglycemic condition. b. The condition usually occurs immediately after dinner. c. It is a response to excessive insulin. d. Management usually requires increase of the bedtime insulin dose.

ANS: C The Somogyi effect is a response to excessive insulin resulting in a hypoglycemic condition usually occurring in the predawn hours of 2:00 to 4:00 am. A rapid decrease in blood glucose during the nighttime hours stimulates a release of hormones (e.g., cortisol, glucagon, epinephrine) to increase blood glucose by lipolysis, gluconeogenesis, and glycogenolysis, thus creating the Somogyi effect. Management of the Somogyi effect involves monitoring blood glucose between 2:00 am and 4:00 am and reducing the bedtime insulin dosage.

(Mod 8) 12. The nurse is teaching a patient who is going on a cruise about the use of transdermal scopolamine . What information will the nurse include when teaching this patient? a. "Apply the patch as needed for nausea and vomiting." b. "Apply the patch to your upper arm." c. "Change the patch every 3 days." d. "Restrict fluids while using this patch."

ANS: C c. "Change the patch every 3 days." The transdermal scopolamine patch is designed to last for 72 hours. The patient should be taught to change it every 3 days. It works best when worn at all times and not just for symptomatic relief. The patch should be applied behind the ear. Patients should not restrict fluids.

6. The nurse reviews a patient's database and learns that the patient lives alone, is forgetful, and does not have an established routine. The patient will be sent home with three new medications to be taken at different times of day. The nurse develops a daily medication chart and enlists a family member to put the patient's pills in a pill organizer. This is an example of which phase of the nursing process? a. Assessment b. Evaluation c. Implementation d. Planning

ANS: C c. Implementation The implementation phase involves education and patient care in order to assist the patient to accomplish the goals of treatment.

(Mod 12) 7. The nurse is caring for an 80-year-old patient who has just begun taking a thiazide diuretic to treat hypertension. What is an important aspect of care for this patient? a. Encouraging increased fluid intake b. Increasing activity and exercise c. Initiating a fall risk protocol d. Providing a low potassium diet

ANS: C c. Initiating a fall risk protocol Older patients experience a higher risk of orthostatic hypotension when taking antihypertensive medications. Fall risk also increases with a need for increased trips to the bathroom. A fall risk protocol should be implemented. Increasing fluids and activity and limiting potassium are not indicated.

(Mod 8) 12. A nurse is teaching a patient how to use phenylephrine (Neo-Synephrine) nasal spray. To avoid systemic absorption, the nurse teaches the patient to perform which action? a. Apply pressure to the nose after spraying. b. Administer the spray while in the supine position. c. Insert the spray while sitting up. d. Exhale deeply while injecting the nasal spray.

ANS: C c. Insert the spray while sitting up. The patient should insert the spray while sitting up to avoid it being absorbed systemically.

(Mod 9) 4. The nurse is caring for a postpartum woman who is refusing opioid analgesics but is rating her pain as a 7 or 8 on a 10-point pain scale. The nurse will contact the provider to request an order for which analgesic medication? a. Diclofenac sodium (Voltaren) b. Ketoprofen (Orudis) c. Ketorolac (Toradol) d. Naproxyn (Naprosyn)

ANS: C c. Ketorolac (Toradol) Ketorolac is the first injectable NSAID and has shown analgesic efficacy equal or superior to that of opioid analgesics. The other NSAIDs listed are not used for postoperative pain.

(Mod 8) 1. The nurse assumes care of a patient who has myasthenia gravis and notes that a dose of neostigmine (Prostigmin) due 1 hour prior was not given. The nurse will anticipate the patient to exhibit which symptoms? a. Excessive salivation b. Muscle spasms c. Muscle weakness d. Respiratory paralysis

ANS: C c. Muscle weakness Neostigmine must be given on time to prevent myasthenic crisis, which is characterized by generalized, severe muscle weakness. The other symptoms are characteristic of cholinergic crisis, caused by too much medication.

(Mod 8) 2. The nurse is caring for a patient who has myasthenia gravis (MG) and takes pyridostigmine bromide (Mestinon) 60 mg every 4 hours. The patient's last dose was 45 minutes prior. The nurse notes severe muscle weakness, excess salivation, fasciculations of facial muscles, and pupil constriction. The nurse will perform which action? a. Assess the patient for signs of ptosis. b. Notify the provider to discuss an order for intravenous immune globulin (IVIG). c. Obtain an order for atropine sulfate. d. Request an order for an extra dose of pyridostigmine.

ANS: C c. Obtain an order for atropine sulfate. Severe muscle weakness, excess salivation, fasciculations of facial muscles, and pupil constriction are the major signs of cholinergic crisis, caused by excess pyridostigmine. The antidote is atropine, so the nurse should obtain an order to give this. Ptosis is sign of myasthenic crisis. IVIG is given to treat symptoms of MG and not used for cholinergic crisis. Giving extra pyridostigmine would increase the symptoms.

(Mod 8) 14. The nurse is caring for a patient in the post-anesthesia recovery unit. The nurse notes that the patient received atropine sulfate 2 mg 30 minutes prior to anesthesia induction. The patient has received 1,000 mL of intravenous fluids and has 700 mL of urine in the urinary catheter bag. The patient reports having a dry mouth. The nurse notes a heart rate of 82 beats per minute. What action will the nurse perform? a. Administer a fluid bolus. b. Give the patient ice chips. c. Palpate the patient's bladder. d. Reassess the patient in 15 minutes.

ANS: C c. Palpate the patient's bladder. Atropine can cause urinary retention. The patient's urine output is less than the fluid intake, so the nurse should palpate the bladder to assess for distension. Dry mouth is an expected side effect and does not indicate dehydration.

(Mod 6) 8. The nurse is preparing to give a dose of oral clindamycin (Cleocin) to a patient who is being treated for a skin infection caused by Staphylococcus aureus. The patient has had several doses of the medication and reports having nausea. Which action will the nurse take next? a. Administer the next dose when the patient has an empty stomach. b. Hold the next dose and contact the patient's provider. c. Instruct the patient to take the next dose with a full glass of water. d. Request an order for an antacid to give along with the next dose.

ANS: C c. Instruct the patient to take the next dose with a full glass of water. Clindamycin should be taken with a full glass of water to minimize gastronintestinal (GI) irritation such as nausea, vomiting, and stomatitis. Giving the medication on an empty stomach will increase the likelihood of GI upset. It is not necessary to hold the next dose or to give an antacid. 84

(Mod 9) 4. The nurse is performing an admission assessment on an adolescent who reports taking extra-strength acetaminophen (Tylenol) regularly to treat daily headaches. The nurse will notify the patient's provider and discuss an order for a. a selective serotonin receptor agonist (SSRA). b. hydrocodone with acetaminophen for headache pain. c. liver enzyme tests. d. serum glucose testing.

ANS: C c. liver enzyme tests. Large doses or overdoses of acetaminophen can be toxic to hepatic cells, so when large doses are administered over a long period, liver function should be assessed. Daily headaches are not typical of migraine headaches, so SSRA medication is not indicated. Hydrocodone with acetaminophen is not indicated without further evaluation of headaches. Serum glucose is not indicated.

(Mod 13) A patient who is taking warfarin has an international normalized ratio (INR) of 5.5. The nurse will anticipate giving a. fresh frozen plasma. b. intravenous iron. c. oral vitamin K. d. protamine sulfate.

ANS: C c. oral vitamin K. Vitamin K is an antagonist against warfarin, an oral anticoagulant. Patients with an INR of 5.5 should be given a low dose of oral vitamin K. Too much vitamin K may reduce the effectiveness of warfarin for up to 2 weeks. Fresh frozen plasma and intravenous iron are given for anemia caused by blood loss. Protamine sulfate is given for heparin overdose

(Mod 12) 5. The nurse understands that a medication such as carvedilol (Coreg) may not be effective in an African-American patient because of its effects on a. cardiac contractility. b. heart rate. c. renin release. d. vascular resistance.

ANS: C c. renin release. African Americans are more likely to be susceptible to low-renin hypertension. Beta blockers reduce heart rate, contractility, and renin release, and there is a greater hypotensive response in patients with higher renin levels. Changes in heart rate, contractility, and vascular resistance explain why there is some response in this group.

(Mod 7) 11. The nurse is caring for a postpartum woman who is refusing opioid analgesics but is rating her pain as a 7 or 8 on a 10-point pain scale. The nurse will contact the provider to request an order for which analgesic medication? a. Diclofenac sodium (Voltaren) b. Ketoprofen (Orudis) c. Ketorolac (Toradol) d. Naproxyn (Naprosyn)

ANS: C c. Ketorolac (Toradol) Ketorolac is the first injectable NSAID and has shown analgesic efficacy equal or superior to that of opioid analgesics. The other NSAIDs listed are not used for postoperative pain.

2. The nurse reviews information about a drug and learns that it is best absorbed in an acidic environment. When giving this drug to a 1-year-old patient, the nurse will expect to administer a dose that will be a. equal to an adult dose. b. less than an adult dose. c. more than an adult dose. d. twice the usual adult dose.

ANS: C c. more than an adult dose. Because the child's gastric pH is more alkaline than the adult's, less drug will be absorbed. Therefore, the dose should be increased.

(Mod 6) 17. A patient who has AIDS is at risk to contract aspergillosis. The nurse will anticipate that which antifungal medication will be ordered prophylactically for this patient? a. Metronidazole (Flagyl) b. Micafungin (Mycamine) c. Posaconazole (Noxafil) d. Voriconazole (Vfend)

ANS: C c. Posaconazole (Noxafil) Posaconazole is given for prophylactic treatment of Aspergillus and Candida infections

(Mod 6) 3. A female patient who is taking trimethoprim-sulfamethoxazole (TMP-SMZ) (Bactrim, Septra) to treat a urinary tract infection reports vaginal itching and discharge. The nurse will perform which action? a. Ask the patient if she might be pregnant. b. Reassure the patient that this is a normal side effect. c. Report a possible superinfection to the provider. d. Suspect that the patient is having a hematologic reaction.

ANS: C c. Report a possible superinfection to the provider. Superinfection can occur with a secondary infection. Vaginal itching and discharge is a sign of superinfection. This is not symptomatic of pregnancy. These are not common side effects and do not indicate a hematologic reaction.

(Mod 7) 5. A patient is taking aspirin to help prevent myocardial infarction and is experiencing moderate gastrointestinal upset. The nurse will contact the patient's provider to discuss changing from aspirin to which drug? a. A COX-2 inhibitor b. Celecoxib (Celebrex) c. Enteric-coated aspirin d. Nabumetone (Relafen)

ANS: C c. Enteric-coated aspirin Aspirin is used to inhibit platelet aggregation to prevent cardiovascular accident and myocardial infarction. Patients taking aspirin for this purpose would not benefit from COX-2 inhibitors, since the COX-1 enzyme is responsible for inhibiting platelet aggregation. The patient should take enteric-coated aspirin to lessen the gastrointestinal distress. Celecoxib and nabumetone are both COX-2 inhibitors.

(Mod 6) 7. A female patient who is allergic to penicillin will begin taking an antibiotic to treat a lower respiratory tract infection. The patient tells the nurse that she almost always develops a vaginal yeast infection when she takes antibiotics and that she will take fluconazole (Diflucan) with the antibiotic being prescribed. Which macrolide order would the nurse question for this patient? a. Azithromycin (Zithromax) b. Clarithromycin (Biaxin) c. Erythromycin (E-Mycin) d. Fidaxomicin (Dificid)

ANS: C c. Erythromycin (E-Mycin) When erythromycin is given concurrently with fluconazole, erythromycin blood concentration and the risk of sudden cardiac death increase.

(Mod 6) 18. A patient is diagnosed with tinea capitis. The provider will order which systemic antifungal medication for this patient? a. Anidulafungin (Eraxis) b. Fluconazole (Diflucan) c. Griseofulvin (Fulvicin) d. Ketoconazole (Nizoral)

ANS: C c. Griseofulvin (Fulvicin) Griseofulvin is used to treat tinea capitis. Anidulafungin is used to treat esophageal candidiasis, candidemia, and other Candida infections. Fluconazole is used to treat Candida infections and cryptococcal meningitis. Ketoconazole is used to treat Candida infections, histoplasmosis, blastomycosis, and other infections.

(Mod 6) 7. The nurse is teaching a patient who will receive acyclovir for a herpes virus infection. What information will the nurse include when teaching this patient? a. Blood cell counts should be monitored closely. b. Dizziness and confusion are harmless side effects. c. Increase fluid intake while taking this medication. d. Side effects are rare with this medication.

ANS: C c. Increase fluid intake while taking this medication. Patients taking acyclovir should increase fluid intake to maintain hydration. A complete blood count is not required. Dizziness and confusion should be reported to the provider. Antiviral medications have many side effects.

(Mod 7) 16. The nurse is teaching a patient about taking colchicine to treat gout. What information will the nurse include when teaching this patient about this drug? a. Avoid all alcohol except beer. b. Include salmon in the diet. c. Increase fluid intake. d. Take on an empty stomach

ANS: C c. Increase fluid intake. The patient who is taking colchicine should increase fluid intake to promote uric acid excretion and prevent renal calculi. Foods rich in purine should be avoided, including beer, and some sea foods, such as salmon. Gastric irritation is a common problem, so colchicine should be taken with food.

(Mod 6) 17. The nurse is reviewing a patient's chart prior to administering gentamycin (Garamycin) and notes that the last serum peak drug level was 9 mcg/mL and the last trough level was 2 mcg/mL. What action will the nurse take? a. Administer the next dose as ordered. b. Obtain repeat peak and trough levels before giving the next dose. c. Report possible drug toxicity to the patient's provider. d. Report a decreased drug therapeutic level to the patient's provider.

ANS: C c. Report possible drug toxicity to the patient's provider. Gentamycin peak values should be 5 to 8 mcg/mL, and trough levels should be 0.5 to 2 mcg/mL. Peak levels give information about whether or not a drug is at toxic levels, while trough levels indicate whether a therapeutic level is maintained. This drug is at a toxic level, and the next dose should not be given.

(Mod 10) 5. A patient is brought to the ED by a family member. The patient reports seeing colored lights and describes feeling bugs crawling under the skin. The nurse suspects that this patient is abusing which drug? A) alcohol B) cocaine C) LSD D) methamphetamine

B) cocaine A stimulant psychosis can occur with chronic use of any stimulant and, with cocaine, progresses to visual hallucinations of colored lights and tactile hallucinations of bugs crawling under the skin. These are not signs of abuse with alcohol, LSD, or methamphetamine.

(Mod 10) 14. A patient is brought to the ED after ingesting an overdose of lorazepam (Ativan) several hours prior. The patient has a respiratory rate of 6 to 10 breaths per minute and is unconscious. The nurse will prepare to perform which action? A) administer activated charcoal B) give flumazenil (Romazicon) C) give naloxone (Narcan) D) perform gastric lovage

B) give flumazenil (Romazicon) Flumazenil is the antidote for benzodiazepine overdose. Activated charcoal is used for asymptomatic patients who have recently consumed the drug. Gastric lavage should no longer be performed for treatment.

(Mod 10) 16. The nurse is teaching a patient who will be discharged home with naltrexone (Re Via) after tx for opioid addiction. What information will the nurse include in the teaching for this patient? A) This drug will help control cravings B) You may take this drug once weekly C) Re Via blocks the pleasurable effects of opioids D) If you discontinue this drug abruptly, you will have withdrawal symptoms

C) Re Via blocks the pleasurable effects of opioids ReVia acts by blocking the pleasurable effects of opioids. It can precipitate withdrawal when given to opioid-dependent patients. This drug does not control cravings, and it is taken once daily or every other day. Naltrexone is an opioid antagonist.

(Mod 10) 6. A patient with asthma has been using a nicotine transdermal 24-hour patch for 3 weeks to quit smoking. The patient reports having difficulty sleeping. What action will the nurse take? A) ask the provider for a prescription for Nicotrol NS B) recommend removing the patch at bedtime C) suggest using an 18-hour patch instead D) tell the patient to stop the patch and join a support group

C) suggest using an 18-hour patch instead The patient should try an 18-hour patch to help with sleep. Nicotrol is not a good option for patients with asthma.

(Mod 10) 7. A nurse is assisting in the discharge planning for a client following alcohol detoxification. The nurse should anticipate presecriptions for which of the following medications to promote long-term abstinence from alcohol. Select All That Apply: A.) Larazepam (Ativan) B.) Diazepam (Valium) C.) Disulfiram (Antabuse) D.) Naltrexone (Vivitrol) E.) Acamprosate (Campral)

C. Disulfiram (Antabuse) **Through aversion therapy D. Naltrexone (Vivitrol) ** Through abstinence by suppressing cravings and pleasurable effects of alcohol E. Acamprosate (Campral) ** Decreases unpleasant effects resulting from abstinence Disulfiram causes unpleasant effects if ROH is consumed while taking it.

(Mod 10) 3. A nurse is planning care for a client who is experiencing benzodiazepine withdrawal. Which of the following is the priority nursing intervention? A..) Orient the client frequently to time, place and person B.) Offer fluids and nourishing diet as tolerated C.) Implement seizure precautions D.) encourage participation in group therapy sessions

C.) Implement seizure precautions

(Mod 6) 6. The nurse is preparing to administer clarithromycin to a patient. When performing a medication history, the nurse learns that the patient takes warfarin to treat atrial fibrillation. The nurse will perform which action? a. Ask the provider if azithromycin may be used instead of clarithromycin. b. Obtain an order for continuous cardiovascular monitoring. c. Request an order for periodic serum warfarin levels. d. Withhold the clarithromycin and notify the provider.

ANS: C c. Request an order for periodic serum warfarin levels. Macrolides can increase serum levels of other drugs such as warfarin. If these drugs are used with macrolides, serum drug levels should be monitored. All macrolides have this drug interaction. Cardiovascular monitoring is not indicated. The drug may be given as long as serum drug levels are monitored. 82

(Mod 6) 1. A patient calls the clinic in November to report a temperature of 103° F, headache, a nonproductive cough, and muscle aches. The patient reports feeling well earlier that day. The nurse will schedule the patient to see the provider and will expect the provider to order which medication? a. Amantadine HCl (Symmetrel) b. Influenza vaccine c. Rimantadine HCl (Flumadine) d. An over-the-counter drug for symptomatic treatment

ANS: C c. Rimantadine HCl (Flumadine) Rimantadine is used for treatment of influenza. Amantadine is used primarily for prophylaxis, and this patient already has symptoms. The influenza vaccine may be given later to protect against other strains. Over-the-counter medications may be used as adjunct treatment.

(Mod 6) 7. A patient who is taking trimethoprim-sulfamethoxazole (TMP-SMX) calls to report developing an all-over rash. The nurse will instruct the patient to perform which action? a. Increase fluid intake. b. Take diphenhydramine. c. Stop taking TMP-SMX immediately. d. Continue taking the medication.

ANS: C c. Stop taking TMP-SMX immediately. A rash can indicate a serious drug reaction. Patients should stop taking the drug immediately and notify the provider.

(Mod 6) 3. A patient who has tuberculosis asks the nurse why three drugs are used to treat this disease. The nurse will explain that multi-drug therapy is used to reduce the likelihood of a. disease relapse. b. drug hypersensitivity reactions. c. drug resistance. d. drug adverse effects

ANS: C c. drug resistance Without multi-drug therapy, patients easily develop resistance to antitubercular drugs. Using more than one antitubercular drug does not prevent relapse, hypersensitivity reactions, or adverse effects.

(Mod 6) 10. The nurse is caring for a patient who is ordered to receive PO trimethoprim-sulfamethoxazole (TMP-SMX) 160/800 QID to treat a urinary tract infection caused by E. coli. The nurse will contact the provider to clarify the correct a. dose. b. drug. c. frequency. d. route.

ANS: C c. frequency. TMP-SMX is taken twice daily. This is the correct dose, drug, and route to treat this condition.

(Mod 7) 4. A patient who is taking aspirin for arthritis pain asks the nurse why it also causes gastrointestinal upset. The nurse understands that this is because aspirin a. increases gastrointestinal secretions. b. increases hypersensitivity reactions. c. inhibits both COX-1 and COX-2. d. is an acidic compound.

ANS: C c. inhibits both COX-1 and COX-2. Aspirin is a COX-1 and COX-2 inhibitor. COX-1 protects the stomach lining, so when it is inhibited, gastric upset occurs. Aspirin does not increase gastrointestinal secretions or hypersensitivity reactions. It is a weak acid.

(Mod 6) 15. The nurse is caring for a 70-kg patient who is receiving gentamicin (Garamycin) 85 mg 4 times daily. The patient reports experiencing ringing in the ears. The nurse will contact the provider to discuss a. decreasing the dose to 50 mg QID. b. giving the dose 3 times daily. c. obtaining a serum drug level. d. ordering a hearing test.

ANS: C c. obtaining a serum drug level. Aminoglycosides can cause ototoxicity. Any changes in hearing should be reported to the provider so that serum drug levels can be monitored. The dose is correct for this patient's weight (5 mg/kg/day in 4 divided doses). A hearing test is not indicated unless changes in hearing persist.

(Mod 9) 12. The nurse administers nalbuphine (Nubain) to a patient who is experiencing severe pain. Which statement by the patient indicates a need for further teaching about this drug? a. "I may experience unusual dreams while taking this medication." b. "I may need to use a laxative when taking this drug." c. "I should ask for assistance when I get out of bed." d. "I should expect to have more frequent urination."

ANS: D d. "I should expect to have more frequent urination." A common side effect of opioid agents is urinary retention. Patients should notify the nurse if they cannot void. Side effects may include unusual dreams, constipation, and dizziness.

8. A patient will be sent home with a metered-dose inhaler, and the nurse is providing teaching. Which is a correctly written goal for this process? a. The nurse will demonstrate correct use of a metered-dose inhaler to the patient. b. The nurse will teach the patient how to administer medication with a metered-dose inhaler. c. The patient will know how to self-administer the medication using the metered-dose inhaler. d. The patient will independently administer the medication using the metered-dose inhaler at the end of the session.

ANS: D d. The patient will independently administer the medication using the metered-dose inhaler at the end of the session. Goals must be patient-centered and clearly state the outcome with a reasonable deadline and should identity components for evaluation.

9. A patient who takes the anticoagulant warfarin will begin taking the anticonvulsant drug carbamazepine. The nurse reviews the drug information for these drugs and learns that carbamazepine is a hepatic enzyme inducer. The nurse anticipates that the provider will make which dosage adjustment? a. Decrease the dose of carbamazepine b. Increase the dose of carbamazepine c. Decrease the dose of warfarin d. Increase the dose of warfarin

ANS: D d. Increase the dose of warfarin Carbamazepine is a hepatic enzyme inducer, which can increase drug metabolism. Patients taking both drugs usually need a larger dose of warfarin.

18. The nurse is preparing to administer a drug that is eliminated through the kidneys. The nurse reviews the patient's chart and notes that the patient has increased serum creatinine and blood urea nitrogen (BUN). The nurse will perform which action? a. Administer the drug as ordered. b. Anticipate a shorter than usual half-life of the drug. c. Expect decreased drug effects when the drug is given. d. Notify the provider and discuss giving a lower dose.

ANS: D d. Notify the provider and discuss giving a lower dose. Increased creatinine and BUN indicate decreased renal function so a drug that is eliminated through the kidneys can become toxic. The nurse should discuss a lower dose with the provider. The drug will have a longer half-life and will exhibit increased effects with decreased renal function.

22. The nurse is preparing to give a dose of gentamicin to a patient and notes that the most recent serum gentamicin trough level was 2 mcg/mL. What will the nurse do next? a. Administer the drug as ordered. b. Administer the drug and monitor for adverse effects. c. Notify the provider to discuss decreasing the dose. d. Notify the provider to report a toxic drug level.

ANS: D d. Notify the provider to report a toxic drug level. The trough drug level for gentamicin should be less than 2 mcg/mL. The nurse should not administer the drug and should notify the provider of the toxic level.

13. The nurse is preparing to administer meperidine (Demerol), which is an opioid analgesic, and promethazine (Phenergan), which is an antiemetic and antihistamine. The nurse understands that these drugs are given in combination for which reason? a. They have antagonistic effects to reduce nausea. b. They have additive effects to enhance analgesia. c. They have potentiating effects to decrease an allergic response. d. They have synergistic effects to increase sedation.

ANS: D d. They have synergistic effects to increase sedation. Meperidine and promethazine have a synergistic effect on each other with a clinical effect that is substantially greater than the combined effect of the two.

12. A patient will receive penicillin to treat an infection. The provider orders probenecid (Probalan), a medication to treat gout, even though the patient does not have gout. Which action by the nurse is correct? a. Administer the drug since the provider ordered it. b. Recognize that it is being given prophylactically. c. Refuse to administer the medication since it is not indicated. d. Verify that it is being given for its secondary action.

ANS: D d. Verify that it is being given for its secondary action. Two or more drugs with the same route of excretion may compete with each other for elimination. Probenecid is given because it inhibits the excretion of penicillin, which may be desirable when the provider wants to prolong the plasma concentration of penicillin. The nurse should always verify an order that may not be clear.

10. The nurse is preparing to administer a drug and learns that it binds to protein at a rate of 90%. The patient's serum albumin level is low. The nurse will observe the patient for a. decreased drug absorption. b. decreased drug interactions. c. decreased drug toxicity. d. increased drug effects.

ANS: D d. increased drug effects. Drugs that are highly protein-bound bind with albumin and other proteins, leaving less free drug in circulation. If a patient has a low albumin, the drug is not bound, and there is more free drug to cause drug effects. There would be increased absorption, increased interactions with other drugs, and increased toxicity.

11. The nurse is administering two drugs to a patient and learns that both drugs are highly protein-bound. The nurse may expect a. decreased bioavailability of both drugs. b. decreased drug effects. c. decreased drug interactions. d. increased risk of adverse effects.

ANS: D d. increased risk of adverse effects. Two drugs that are highly protein-bound will compete for protein-binding sites, leaving more free drug in circulation and an increased risk of adverse effects as well as increased bioavailability, increased drug effects, and increased drug interactions.

7. The nurse notes that a patient of African American descent who is taking an oral antihypertensive medication continues to have elevated blood pressure three months after beginning the medication regimen. The nurse suspects that the patient may be a. consuming ethnic foods that interfere with absorption of the drug. b. discarding the medication. c. experiencing allergic reactions to the medication. d. metabolizing the drug differently than expected.

ANS: D d. metabolizing the drug differently than expected. Certain classifications of medications have different effects in individuals whose genetic markers are predominantly characteristic of a certain biologic group. African Americans respond poorly to several classes of antihypertensive agents.

17. A patient asks the nurse about using over-the-counter (OTC) medications. The nurse will tell the patient that OTC medications a. are not as effective as prescription medications. b. are not as safe as prescription medications. c. have fewer side effects and drug interactions than prescription medications. d. should be included when listing any medications taken by the patient.

ANS: D d. should be included when listing any medications taken by the patient. OTC medications should always be included when listing medications because they can cause drug interactions. OTC medications can be as effective and as safe as prescription medications and have as many side effects and adverse reactions.

24. The nurse administers a narcotic analgesic to a patient who has been receiving it for 1 day after orthopedic surgery. The patient reports no change in pain 30 minutes after the medication is given. The nurse recognizes that this patient is exhibiting a. drug-seeking behavior. b. drug tolerance. c. the placebo effect. d. tachyphylaxis.

ANS: D d. tachyphylaxis. Tachyphylaxis is a rapid decrease in response, or acute tolerance. Tolerance to drug effects can occur with narcotics, requiring increased doses in order to achieve adequate drug effects. Nurses often mistake drug-seeking behavior for drug tolerance. The placebo effect occurs when the patient experiences a response with an inactive drug.

(Mod 6) 11. A patient who has oral candidiasis will begin using nystatin suspension to treat the infection. What information will the nurse include when teaching this patient? a. "Coat the buccal mucosa with the drug and then rinse your mouth." b. "Gargle with the nystatin and then spit it out without swallowing." c. "Mix the suspension with 4 ounces of water and then drink it." d. "Swish the liquid in your mouth and then swallow after a few minutes."

ANS: D d. "Swish the liquid in your mouth and then swallow after a few minutes." Patients should be taught to swish the suspension in the mouth to coat the tongue and buccal mucosa and then swallow the medication. It should not be spit out, diluted with water, or swallowed with water.

(Mod 6) 1. Which person should be treated with prophylactic antitubercular medication? a. A child who attends the same school with a child who has tuberculosis b. A nurse who is working in a hospital c. An individual who is HIV-positive with a negative TB skin test d. A patient who has close contact with someone who has tuberculos

ANS: D d. A patient who has close contact with someone who has tuberculosis Personal contact with a person having a diagnosis of tuberculosis is required to indicate prophylactic treatment with antitubercular therapy. Attending the same school does not necessarily mean close contact occurs. Health care professionals do not need prophylactic treatment. HIV-positive individuals with negative TB skin tests do not need prophylaxis.

(Mod 6) 15. A patient will begin taking amoxicillin. The nurse should instruct the patient to avoid which foods? a. Green leafy vegetables b. Beef and other red meat c. Coffee, tea, and colas d. Acidic fruits and juices

ANS: D d. Acidic fruits and juices Acidic fruits and juices should be avoided while the client is being treated with amoxicillin because amoxicillin can be irritating to the stomach. Stomach irritation will be increased with the ingestion of citrus and acidic foods. Amoxicillin may also be less effective when taken with acidic fruit or juice.

(Mod 6) 16. The patient will begin taking penicillin G procaine (Wycillin).The nurse notes that the solution is milky in color. What action will the nurse take? a. Call the pharmacist and report the milky color. b. Add normal saline to dilute the medication. c. Call the physician and report the milky appearance. d. Administer the medication as ordered by the physician.

ANS: D d. Administer the medication as ordered by the physician. Penicillin G procaine (Wycillin) has a milky appearance; therefore, the appearance should not concern the nurse.

(Mod 10) 9. The nurse is caring for a patient who is chronically irritable and anxious and prone to violent behaviors. The patient has several teeth missing and has dental caries in the remaining teeth. The nurse suspects previous chronic use of which drug. A) alcohol B) cocaine C) LSD D) methamphetamine

D) methamphetamine Patients previously exposed to methamphetamine use will exhibit these symptoms, and the physical [exam]... I feel like the rationale got cut off. But sounds like the P/E is describing meth mouth.

(Mod 10) 3. What does the nurse understand must occur in order to produce withdrawal syndrome? A) addiction B) craving C) drug tolerance D) physical dependence

D) physical dependence Patients who develop a physical dependence on a drug will experience withdrawal syndrome when the drug is stopped. Intoxication is a condition that results in disturbances in the level of consciousness, cognition, perception, judgment, behavior, and other psycho-physiologic functions. Cravings can occur without physical dependence. Tolerance refers to a decrease in drug effects with repeated use.

(Mod 11) A patient who has type 1 diabetes mellitus asks the nurse about using a combination insulin product such as Humalog 75/25. The nurse will tell the patient that use of this product a. depends on individual insulin needs. b. is useful for patient with insulin resistance. c. means less rotation of injection sites. d. requires refrigeration at all times.

a. depends on individual insulin needs. Combination products are convenient because the patient does not have to mix insulin, but the products depend on individual needs, since the doses are fixed. They are not used for patients with insulin resistance. Patients must continue to rotate injection sites. They do not require refrigeration after first use.

(Mod 11) A patient who has type 2 diabetes mellitus asks the nurse why the provider has changed the oral antidiabetic agent from tolbutamide (Orinase) to glipizide (Glucotrol). The nurse will explain that glipizide a. has a longer duration of action. b. has fewer gastrointestinal side effects. c. may be taken on an as-needed basis. d. results in less hypoglycemic potential.

a. has a longer duration of action. Glipizide is a second-generation oral antidiabetic agent. It has a longer duration of action than the first-generation antidiabetic agents such as tolbutamide. It has many gastrointestinal side effects. It is taken once daily, not as needed. It has greater hypoglycemic activity than first-generation antidiabetics.

(Mod 11) The nurse receives the following order for insulin: IV NPH (Humulin NPH) 10 units. The nurse will perform which action? a. Administer the dose as ordered. b. Clarify the insulin type and route. c. Give the drug subcutaneously. d. Question the insulin dose.

b. Clarify the insulin type and route. Only regular insulin can be given intravenously. The nurse should clarify the order. It is not correct to give Humulin NPH insulin IV. The nurse should not administer the drug by a different route without first discussing with the provider.

(Mod 11) The nurse is teaching a group of nursing students about diabetes. The nurse explains that which type of diabetes is the most common? a. Type 1 diabetes mellitus b. Type 2 diabetes mellitus c. Diabetes insipidus d. Secondary diabetes

b. Type 2 diabetes mellitus Type 2 diabetes mellitus is the most common type of diabetes.

(Mod 11) A patient who is unconscious and has a pulse is brought to the emergency department. The patient is wearing a Medic-Alert bracelet indicating type 1 diabetes mellitus. The nurse will anticipate an order to administer a. cardiopulmonary resuscitation (CPR). b. glucagon. c. insulin. d. orange juice.

b. glucagon. This patient is most likely hypoglycemic and will need a carbohydrate. Glucagon is given parenterally if patients are unable to ingest a carbohydrate, such as orange juice. CPR is not indicated. Insulin will compound the hypoglycemia.

(Mod 11) The nurse is teaching a patient about home administration of insulin. The patient will receive regular (Humulin R) and NPH (Humulin NPH) insulin at 0700 every day. What is important to teach this patient? a. "Draw up the medications in separate syringes." b. "Draw up the NPH insulin first." c. "Draw up the regular insulin first." d. "Draw up the medications after mixing them in a vial."

c. "Draw up the regular insulin first." Patients should be instructed to draw up regular insulin first so that NPH is not mixed into the vial of regular insulin. It is not necessary to use separate syringes. Patients do not mix the medications in a vial.

(Mod 11) A patient who has been taking a sulfonylurea antidiabetic medication will begin taking metformin (Glucophage). The nurse understands that this patient is at increased risk for which condition? a. Hypoglycemia b. Hyperglycemia c. Renal failure d. Respiratory distress

c. Renal failure Metformin can lead to renal failure. It does not produce hypoglycemia or hyperglycemia. It does not increase the risk of respiratory distress.

(Mod 11) A patient develops type 2 diabetes mellitus. The nurse will explain that this type of diabetes a. is generally triggered by medications. b. is not as common as type 1 diabetes. c. is often related to heredity and obesity. d. will not require insulin therapy.

c. is often related to heredity and obesity. Type 2 diabetes is often caused by obesity and hereditary factors. Secondary diabetes is triggered by medications. Type 2 diabetes is the most common type of diabetes. Patients with type 2 diabetes may become insulin dependent.

(Mod 11) A 45-year-old patient who is overweight has had a diagnosis of type 2 diabetes for 2 years. The patient uses 20 units of insulin per day. The patient's fasting blood glucose (FBG) is 190 mg/dL. The patient asks the nurse about using an oral anti-diabetic agent. The nurse understands that oral anti-diabetic agents a. cannot be used if the patient is overweight. b. cannot be used once a patient requires insulin. c. may be used since this patient meets criteria. d. may not be used since this patient's fasting blood glucose is too high.

c. may be used since this patient meets criteria. Patients who require less than 40 units of insulin per day and who have a fasting blood glucose less than or equal to 200 mg/dL are candidates for oral anti-diabetic agents. Being overweight is an indication, not a contraindication.

(Mod 11) The nurse is teaching a patient how to administer insulin. The patient is thin with very little body fat. The nurse will suggest injecting insulin a. by pinching up the skin and injecting straight down. b. in the abdomen only with the needle at a 90-degree angle. c. subcutaneously with the needle at a 45- to 60-degree angle. d. using the thigh and buttocks areas exclusively.

c. subcutaneously with the needle at a 45- to 60-degree angle. In a thin person, with little fatty tissue, the needle is inserted at a 45- to 60-degree angle. In other patients, a 45- to 90-degree angle is acceptable. There is no recommendation for preferring one site over another.

(Mod 11) Which statement by a patient who will begin using an external insulin pump indicates understanding of this device? a. "I will have an increased risk for hypoglycemia." b. "I will leave this on when bathing or swimming." c. "I will not need to count carbohydrates anymore." d. "I will still need to monitor serum glucose."

d. "I will still need to monitor serum glucose." Patients using an insulin pump will still monitor serum glucose and count carbohydrates. The advantage of the pump is that it is programmed to deliver continuous rapid-acting insulin in varying amounts at different times throughout the day. Changes in food intake can alter the risk for hypoglycemia if the pump is not adjusted accordingly. They must be removed when bathing or swimming.

(Mod 11) The patient asks the nurse about storing insulin. Which response by the nurse is correct? a. "All insulin vials must be refrigerated." b. "Insulin will last longer if kept in the freezer." c. "Opened vials of insulin must be discarded." d. "Some combination pens do not require refrigeration."

d. "Some combination pens do not require refrigeration." Some combination pens do not require refrigeration after first use. Storing insulin in the freezer is not recommended. Opened vials may either be kept at room temperature for a month or refrigerated for 3 months.

(Mod 11) A patient who is overweight is being evaluated for diabetes. The patient has a blood glucose level of 160 mg/dL and a hemoglobin A1c of 5.8%. The nurse understands that this patient has which condition? a. Diabetes mellitus b. Hypoglycemia c. Normal blood levels d. Prediabetes

d. Prediabetes Patients with a hemoglobin A1c between 5.7% and 6.4% are considered to have prediabetes. A level of 6.5% or more indicates diabetes. The patient is hyperglycemic.

(Mod 11) The nurse will administer parenteral insulin to a patient who will receive a mixture of NPH (Humulin NPH) and regular (Humulin R). The nurse will give this medication via which route? a. Intradermal b. Intramuscular c. Intravenous d. Subcutaneous

d. Subcutaneous Insulin is given by the subcutaneous route. Only regular insulin may be given IV.

(Mod 11) A patient who has insulin-dependent diabetes mellitus must take a glucocorticoid medication for osteoarthritis. When teaching this patient, the nurse will explain that there may be a need to a. decrease the glucocorticoid dose. b. decrease the insulin dose. c. increase the glucocorticoid dose. d. increase the insulin dose.

d. increase the insulin dose. Glucocorticoids can cause hyperglycemia, so the insulin dose may need to be increased. Changing the glucocorticoid dose is not recommended. Decreasing the insulin dose will only compound the hyperglycemic effects.

(Mod 11) The parent of a junior high-school child who has type 1 diabetes asks the nurse if the child can participate in sports. The nurse will tell the parent a. that strenuous exercise is not recommended for children with diabetes. b. that the child must be monitored for hyperglycemia while exercising. c. to administer an extra dose of regular insulin prior to exercise. d. to send a snack with the child to eat just prior to exercise.

d. to send a snack with the child to eat just prior to exercise. Patients generally need less insulin with increased exercise, so the child should consume a snack to prevent hypoglycemia. Exercise is an integral part of diabetes management. Hypoglycemia is more likely to occur, and extra insulin is not indicated.

(Mod 10) 15. A patient with history of opioid abuse will be discharged home with buprenorphine to help prevent relapse. Which product will the nurse anticipate the provider to order? A) buprenex B) suboxone C) subutex D) vivitrol

A) buprenex Buprenex is an agonist-antagonist opioid that can be used for detoxification and maintenance therapy because it has a low potential for abuse. Suboxone and Subutex have abuse potential. Vivitrol does not contain buprenorphine and does not prevent cravings. Suboxone = buprenorphine+naloxone Subutex = buprenorphine Vivitrol = Naltrexone

(Mod 13) A patient experiences a blood clot in one leg, and the provider has ordered a thrombolytic medication. The patient learns that the medication is expensive and asks the nurse if it is necessary. Which response by the nurse is correct? a. "The drug will decrease the likelihood of permanent tissue damage." b. "This medication also acts to prevent future blood clots from forming." c. "You could take aspirin instead of this drug to achieve the same effect." d. "Your body will break down the clot, so the drug is not necessary."

ANS: A a. "The drug will decrease the likelihood of permanent tissue damage." Thrombolytic medications are given primarily to prevent permanent tissue damage caused by compromised blood flow to the affected area. Thrombolytics do not prevent clots from forming. Aspirin prevents, but does not dissolve, clots. Although the body will break down the clot, the drug is needed to prevent tissue damage due to active ischemia.

(Mod 8) 8. The nurse is caring for a postoperative patient and notes that the patient received atropine sulfate preoperatively. Which assessment finding would prompt the nurse to notify the provider? a. Absent bowel sounds b. Drowsiness c. Dry mouth d. Heart rate of 78 beats per minute

ANS: A a. Absent bowel sounds These are all side effects of atropine. Absent bowel sounds can indicate a paralytic ileus. The other side effects are not harmful.

(Mod 8) 9. A patient who has irritable bowel syndrome would most likely receive which type of drug to treat this condition? a. An anticholinergic b. A cholinergic esterase inhibitor c. A muscarinic agent d. A nicotinic agent

ANS: A a. An anticholinergic Anticholinergic drugs are used to treat peptic ulcers and intestinal spasticity because of their actions to decrease gastric secretions and gastrointestinal spasms.

(Mod 8) 17. The nurse is preparing to administer tolterodine tartrate (Detrol LA) to a patient who has incontinence. Which symptom would be a contraindication for this drug? a. Decreased bowel sounds b. Drooling c. Gastric upset d. Pain

ANS: A a. Decreased bowel sounds A decrease in bowel sounds could signal the beginning of paralytic ileus. Detrol is contraindicated in patients with paralytic ileus.

(Mod 13) A patient who has received heparin after previous surgeries will be given enoxaparin sodium (Lovenox) after knee-replacement surgery. The patient asks how this drug is different from heparin. The nurse will explain that enoxaparin a. decreases the need for laboratory tests. b. has a shorter half-life than heparin. c. increases the risk of hemorrhage. d. may be taken orally instead of subcutaneously.

ANS: A a. decreases the need for laboratory tests. Enoxaparin is a low-molecular-weight heparin, which produces more stable responses at lower doses, thus reducing the need for frequent lab monitoring. It has a longer half-life than heparin. It decreases the risk of hemorrhage because it is more stable at lower doses. It is given subcutaneously.

(Mod 6) 13. A child who weighs 10 kg will begin taking oral trimethoprim-sulfamethoxazole (TMP-SMX). The liquid preparation contains 40 mg of TMP and 200 mg of SMX per 5 mL. The nurse determines that the child's dose should be 8 mg of TMP and 40 mg of SMX/kg/day divided into two doses. Which order for this child is correct? a. 5 mL PO BID b. 5 mL PO daily c. 10 mL PO BID d. 10 mL PO daily

ANS: A a. 5 mL PO BID This child should receive (10 kg/8 mg) 80 mg of TMP and (10 kg ´ 40 mg) 400 mL of SMX per day. When divided into two doses, the correct dose is 40 mg TMP and 200 mg SMX, or 5 mL per dose. 107

8. The nurse is reviewing medication information with a nursing student prior to administering an oral drug and notes that the drug has extensive first-pass effects. Which statement by the student indicates a need for further teaching about this medication? a. "The first-pass effect means the drug may be absorbed into systemic circulation from the intestinal lumen." b. "The first-pass effect means the drug may be changed to an inactive form and excreted." c. "The first-pass effect means the drug may be changed to a metabolite, which may be more active than the original." d. "The first-pass effect means the drug may be unchanged as it passes through the liver."

ANS: A a. "The first-pass effect means the drug may be absorbed into systemic circulation from the intestinal lumen." Drugs that undergo first-pass metabolism are absorbed into the portal vein from the intestinal lumen and go through the liver where they are either unchanged or are metabolized to an inactive or a more active form.

(Mod 10) 2. The nurse performing an assessment on a patient identifies the manifestations of cocaine use, which include A. fatigue and hallucinations. B. cardiac dysrhythmias and violent behavior. C. shock and hypotension. D. shallow respirations and pallor.

ANS: B B. cardiac dysrhythmias and violent behavior. Cocaine increases norepinephrine at postsynaptic receptor sites, producing intense vasoconstriction and cardiovascular stimulation. Common manifestations of cocaine use include increased temperature, pulse, respiratory rate, and blood pressure; cardiac dysrhythmias; myocardial infarction; hallucinations; seizures; and possible death.

(Mod 11) 1. Which time frame would be most appropriate for administering sliding-scale lispro insulin? a. Within 30 minutes of consuming breakfast b. When the breakfast tray is served and ready to eat c. Within 1 hour of obtaining blood glucose measurement d. Within 15 minutes of obtaining blood glucose measurement

ANS: B Lispro should be given 5 minutes before eating because the onset of action is 5 to 15 minutes.

(Mod 7) 2. A nursing student asks how nonsteroidal antiinflammatory drugs (NSAIDs) work to suppress inflammation and reduce pain. The nurse will explain that NSAIDs a. exert direct actions to cause relaxation of smooth muscle. b. inhibit cyclooxygenase that is necessary for prostaglandin synthesis. c. interfere with neuronal pathways associated with prostaglandin action. d. suppress prostaglandin activity by blocking tissue receptor sites.

ANS: B b. inhibit cyclooxygenase that is necessary for prostaglandin synthesis. NSAIDs act by inhibiting COX-1 and COX-2 to help block prostaglandin synthesis. They do not have direct action on tissues, nor do they interfere with chemical receptor sites or neuronal pathways.

20. Which patients are at high risk for drug interactions? (Select all that apply.) a. Patients who are acutely ill b. Patients who are taking multiple medications c. Patients who see several specialists d. Patients who take supplements and OTC medications e. Patients who use one pharmacy for several medications

ANS: B, C, D b. Patients who are taking multiple medications c. Patients who see several specialists d. Patients who take supplements and OTC medications Patients who have chronic health conditions, take multiple medications, see more than one provider, and use supplements and OTC medications are at higher risk for drug interactions.

6. The nurse is preparing to administer a medication from a unit-dose system. The nurse verifies that the medication, dose, and time are correct and that the expiration date was the day prior. Which action is correct? a. Administer the medication and observe for adequate drug effects. b. Notify the pharmacist and provider of a medication error. c. Return the medication to the pharmacy to be replaced. d. Verify the right patient and administer the medication.

ANS: C c. Return the medication to the pharmacy to be replaced. If a drug expiration date has passed, it should be returned to the pharmacy or discarded, never used. A medication error would occur only if the medication was given.

(Mod 13) The nurse is caring for a postoperative patient. The nurse will anticipate administering which medication to this patient to help prevent thrombus formation caused by slow venous blood flow? a. Alteplase (Activase) b. Aspirin c. Clopidogrel (Plavix) d. Low-molecular-weight heparin

ANS: D d. Low-molecular-weight heparin Low-molecular-weight heparin is an anticoagulant, which is used to inhibit clot formation and is used prophylactically to prevent postoperative deep vein thrombosis. Alteplase is a thrombolytic, which is used to break down clots after they form; alteplase is contraindicated in any patient with recent surgery. Aspirin and clopidogrel are antiplatelet drugs and are used to prevent arterial thrombosis.

(Mod 8) 5. A patient exhibits ptosis of both eyes, and the provider orders edrophonium (Tensilon). The nurse notes immediate improvement of the ptosis. The nurse understands that this patient most likely has which disorder? a. Cerebral palsy b. Multiple sclerosis c. Muscle spasms d. Myasthenia gravis

ANS: D d. Myasthenia gravis Improvement of symptoms after administration of edrophonium is diagnostic for myasthenia gravis.

7. A patient who is hospitalized for chronic obstructive pulmonary disease wants to go home. The nurse and the patient discuss the patient's situation and decide that the patient may go home when able to perform self-care without dyspnea and hypoxia. This is an example of which phase of the nursing process? a. Assessment b. Evaluation c. Implementation d. Planning

ANS: D d. Planning Planning involves goal-setting which, for this patient, means being able to perform self-care activities without dyspnea and hypoxia.

(Mod 6) 1. The nurse is caring for a patient who is receiving an intravenous antibiotic. The nurse notes that the provider has ordered serum drug peak and trough levels. The nurse understands that these tests are necessary for which type of drugs? a. Drugs with a broad spectrum b. Drugs with a narrow spectrum c. Drugs with a broad therapeutic index d. Drugs with a narrow therapeutic index

ANS: D d. Drugs with a narrow therapeutic index Medications with a narrow therapeutic index have a limited range between the therapeutic dose and a toxic dose. It is important to monitor these medications closely by evaluating regular serum peak and trough levels.

(Mod 6) 20. A patient who takes an oral sulfonylurea medication will begin taking fluconazole (Diflucan). The nurse will expect to monitor which lab values in this patient? a. Blood urea nitrogen (BUN) and creatinine b. Electrolytes c. Fluconazole levels d. Glucose

ANS: D d. Glucose Patients taking sulfonylurea drugs may have altered serum glucose when taking antifungal medications.

21. The nurse is explaining to the patient why a nonspecific drug has so many side effects. Which statement by the patient indicates a need for further teaching? a. "Nonspecific drugs can affect specific receptor types in different body tissues." b. "Nonspecific drugs can affect a variety of receptor types in similar body tissues." c. "Nonspecific drugs can affect hormone secretion as well as cellular functions." d. "Nonspecific drugs require higher doses than specific drugs to be effective."

ANS: D (It should be B and D) d. "Nonspecific drugs require higher doses than specific drugs to be effective." Nonspecific drugs can act on one type of receptor but in different body tissues, or a variety of receptor types (this is actually nonselective), or act on hormones to produce effects. Nonspecific drugs do not require higher doses.

(Mod 6) 5. The nurse is preparing to administer trimethoprim-sulfamethoxazole (TMP-SMX) to a patient who is being treated for a urinary tract infection. The nurse learns that the patient has type 2 diabetes mellitus and takes a sulfonylurea oral antidiabetic drug. The nurse will monitor this patient closely for which effect? a. Headaches b. Hypertension c. Hypoglycemia d. Superinfection

ANS:C c. Hypoglycemia Taking oral antidiabetic agents (sulfonylurea) with sulfonamides increases the hypoglycemic effect. Sulfonylureas do not increase the incidence of headaches, hypertension, or superinfection when taken with sulfonamides. Examples of antidiabetic sulfonylurea medications are glipizide, glimepride, glyburide, tolaamide, and tolbutamide_

(Mod 10) 1. A nurse is planning a staff education program on substance use in older adults. Which of the following is appropriate for the nurse to include in the presentation? A.) Older adults require higher doses of a substance to achieve a desired effect. B.) Older adults commonly use rationalization to cope with a substance use disorder C.) Older adults are at a higher risk for substance use following retirement. D.) Older adults develop substance use to mask signs of dementia

C.) Older adults are at a higher risk for substance use following retirement.

(Mod 10) 5. A nurse is providing teaching to the family of a client who has a substance use disorder. Which of the following statements by a family member indicates a need for further teaching? A.) "We need to understand that she is not responsible for her disorder." B.) "eliminating any codependent behavior will promote her recovery." C.) "She should participate in an Al-Anon group to help her recover" D.) "The primary goal of her treatment is abstinence from substance abuse."

C.) She should participate in an Al-Anon group to help her recover (AL-Anon is meant for the family, not client)

Lab Values to Know for Clotting (Drug?) PT (nl not on anticoags) INR (nl not on anticoags): INR (therapeutic): INR (for prosthetic heart valves): (Drug?) PTT aPTT aPTT (therapeutic) Fibrinogen: Plt:

(Warfarin) PT (nl not on anticoags): 11-15s INR (nl not on anticoags): 1.3-2.0 INR (therapeutic): 2.0-3.0 INR (for prosthetic heart valves): up to 3.5 (Heparin) PTT: 60-70s aPTT: 30-40s aPTT (therapeutic): 30-85s Fibrinogen: 200-400 mg/dL Plt: 150,000-400,000 (<150k = thrombocytopenia)

(Mod 10) 8. A patient is using the Commit lozenge 2 mg to help quit smoking and reports nausea and indigestion. The nurse will instruct the patient to perform which action? A) allow the lozenge to dissolve slowly over 20 to 30 minutes B) chew the lozenge thoroughly before swallowing it. C) increase to 4 mg and use less often D) take a the lozenge with food and a glass of water

A) allow the lozenge to dissolve slowly over 20 to 30 minutes

(Mod 10) 7. The nurse is discussing smoking cessation with a nurse colleague who smokes. Which statement indicates a readiness to quit smoking? A) I don't smoke around my children or inside the house B) I want to stop smoking, but I will need help to do it C) I will quit smoking so my co-workers will stop harassing me about it. D) if I cut down gradually, I should be able to quit

B) I want to stop smoking, but I will need help to do it Patients exhibit readiness when they state a desire to quit along with a request for professional assistance. Other factors, such as children or co-workers, do not indicate a desire to quit.

(Mod 10) 1. The nurse is teaching a group in the community about drug abuse. Which statement by the nurse is correct? A) cue-induced cravings eventually disappear after long periods of abstinence by the person addicted to drugs. B) drug abuse and drug addiction are synonymous terms, describing dependence on drugs. C) Drug addiction is characterized by emotional, mental, and sometimes sometimes physical dependence. D) drug addiction occurs when physical dependence is present.

C) Drug addiction is characterized by emotional, mental, and sometimes sometimes physical dependence Drug addiction occurs when emotional and mental dependence on a drug are present. Although physical dependence may often occur, it is not always present. Cue-induced cravings may diminish after long abstinence but do not disappear completely. Drug abuse may occur without addiction.

(Mod 10) 13. A patient who is unconscious arrives in the ED with clammy skin and constricted pupils. The nurse assesses a respiratory rate of 8 to 10 breaths per minute. The paramedics report obvious signs of drug abuse in the patients home. The nurse suspects that this patient has had an overdose of which substance? A) alcohol B) LSD C) an opioid D) methamphetamine

C) an opioid Opioid overdose is characterized by constricted pupils and respiratory depression.

(Mod 10) 4. A nurse is caring for client who has alcohol use disorder. The client is no longer experiencing withdrawal manifestations. Which of the following medications should the nurse anticipate administering to assist the client with maintaining abstinence from alcohol? A.) Chlordiasepoxide (Librium) B.) Bupropion (Zyban) C.) Disulfiram (Antabuse) D.) Carbamazepine ( Tegretol)

C.) Disulfiram (Antabuse) **Remember that this med makes them violently ill and has the potential for death if drinking occurs

(Mod 10) 2. The nurse is caring for a patient who is being treated for chronic alcohol intoxication. The nurse notes that the patient's serum alcohol level is 0.40 mg%. The patient is awake and talkative even though this is a potentially lethal dose. The nurse recognizes this as alcohol A) addiction B) dependence C) misuse D) tolerance

D) Tolerance Intoxication is a state of being influenced by a drug or other substance and may be a very small amount in the drug-naïve person or a potentially lethal amount in the chronic user. This person has developed tolerance to alcohol and is able to have a potentially lethal amount without severe effects. Addiction describes continued involvement in an activity despite the substantial harm it causes. Dependence describes physical need for the drug suchthat when the drug is stopped, withdrawal symptoms occur. Misuse refers to using a drug or substance to excess.

(Mod 10) 4. The nurse is counseling a patient who wants to stop smoking. Which statement by the nurse is correct? A) bupropion (Zyban) is effective and does not have serious adverse effects. B) nicotine replacement therapies are effective and eliminate the need for behavioral therapy. C) varenicline (Chantix) may be used short-term for 1 to 2 months. D) You may experience headaches and increased appetite for several months after stopping smoking.

D) You may experience headaches and increased appetite for several months after stopping smoking.

(Mod 11) A patient is ordered to receive insulin lispro at mealtimes. The nurse will instruct this patient to administer the medication at which time? a. 5 minutes before eating b. 15 minutes after eating c. 30 minutes before eating d. 10 minutes after eating

a. 5 minutes before eating Lispro acts faster than other insulins, and patients should be taught to give this medication not more than 5 minutes before eating.

(Mod 13) A patient is receiving a thrombolytic medication. The patient calls the nurse to report having bloody diarrhea. The nurse will anticipate administering which medication? a. Aminocaproic acid (Amicar) b. Enoxaparin sodium (Lovenox) c. Protamine sulfate d. Vitamin K

ANS: A a. Aminocaproic acid (Amicar) The antithrombotic drug aminocaproic acid is used to treat hemorrhage. Nurses giving thrombolytic drugs should monitor patients for bleeding from the mouth and rectum. Enoxaparin is given for DIC. Protamine sulfate is an antidote for heparin. Vitamin K is an antidote for warfarin.

(Mod 10) 6. A nurse is providing teaching to a client who has a new prescription for carbamazepine (tegretol). Which of the following should the nurse include in the teaching? A.) This medication will help prevent seizures during alcohol withdrawal. B.) Taking this medication will decrease your cravings for alcohol C.) This medication maintains your blood pressure at a normal level during alcohol withdrawal D.) Taking this medication will improve your ability to maintain abstinence from alcohol

A. This medication will help prevent seizures during alcohol withdrawal.

6. An infant will receive a topical medication. What instruction will the nurse include when teaching the parents how to administer the medication? a. "Apply a thin layer to the affected area." b. "Apply liberally to the skin on and around the area." c. "Use the medication less frequently than what is recommended for adults." d. "Use the medication more frequently than what is recommended for adults."

ANS: A a. "Apply a thin layer to the affected area." Topical medications may be altered by skin tissue condition. Children have thinner, more porous skin and have a proportionately higher skin surface area than adults and thus absorb topical medications more readily. Caregivers should be advised to use only a thin layer on the affected body part. This difference in skin does not affect the frequency of administering topical medications.

(Mod 9) 2. The nurse is caring for a patient who has migraine headaches. The patient reports having these headaches more frequently. Which is an appropriate recommendation for this patient? a. "Avoid chocolate and caffeine." b. "Engage in strenuous exercise." c. "Have a glass of red wine with dinner." d. "Take ibuprofen prophylactically."

ANS: A a. "Avoid chocolate and caffeine." Triggering factors for migraine headache include foods such as chocolate, caffeine, and red wine. Intense physical exertion can trigger migraines. Prophylactic ibuprofen is not indicated.

14. An adolescent patient who has acne is given a regimen of topical medications and an oral antibiotic that generally clears up lesions to fewer than 10 within 6 to 8 weeks. At a 2-month follow-up, the patient continues to have more than 25 lesions. The child's parent affirms that the child is using the medications as prescribed. Which evaluation statement is correct for this patient? a. "Goal of fewer than 10 lesions in 6 to 8 weeks is not met." b. "Goal that the medication will be effective is not met." c. "Goal that the patient will take medications as prescribed is not met." d. "Goal that the patient understands the medication regimen is not met."

ANS: A a. "Goal of fewer than 10 lesions in 6 to 8 weeks is not met." All indications are that this patient is taking the medications and they are not effective. The first statement is correct because it identifies a measurable goal and a specific time frame.

(Mod 9) 2. The nurse is teaching a female patient who will begin taking 2 tablets of 325 mg acetaminophen every 4 to 6 hours as needed for pain. Which statement by the patient indicates understanding of the teaching? a. "I may take acetaminophen up to 6 times daily if needed." b. "I should increase the dose of acetaminophen if I drink caffeinated coffee." c. "If I take oral contraceptive pills, I should use back-up contraception." d. "It is safe to take acetaminophen with any over-the-counter medications."

ANS: A a. "I may take acetaminophen up to 6 times daily if needed." The maximum daily dose of acetaminophen is 4000 mg. If this patient takes 650 mg/dose 6 times daily, this amount is safe. Taking acetaminophen with caffeine increases the effect of the acetaminophen. Taking acetaminophen with OCPs decreases the effect of the acetaminophen but does not diminish the effect of the OCP. Many over-the-counter medications contain acetaminophen, so patients should be advised to read labels carefully to avoid overdose.

(Mod 9) 6. The nurse is providing teaching to a patient who will begin taking aspirin to treat arthritis pain. Which statement by the patient indicates a need for further teaching? a. "I should increase fiber and fluids while taking aspirin." b. "I will call my provider if I have abdominal pain." c. "I will drink a full glass of water with each dose." d. "I will notify my provider of ringing in my ears."

ANS: A a. "I should increase fiber and fluids while taking aspirin." Aspirin is not constipating, so patients do not need to be counseled to consume extra fluids and fiber. Abdominal pain can occur with gastrointestinal bleeding, and tinnitus (ringing in the ears) can be an early sign of toxicity, so patients should be taught to contact their provider if these occur. Taking a full glass of water with each dose helps minimize gastrointestinal side effects.

(Mod 9) 14. Which patient may require a higher than expected dose of an opioid analgesic? a. A patient with cancer b. A patient with a concussion c. A patient with hypotension d. A patient 3 days after surgery

ANS: A a. A patient with cancer Opioids are titrated for oncology patients until pain relief is achieved or the side effects become intolerable, and extremely high doses may be required. Patient with closed head injuries should receive reduced doses of opioids if at all to reduce the risk of increased intracranial pressure. Patients with hypotension should receive reduced doses to prevent further decrease in blood pressure. Patients who are 3 days post-operation should not be experiencing severe pain.

(Mod 9) 3. The parent of a 5-year-old child asks the nurse to recommend an over-the-counter pain medication for the child. Which analgesic will the nurse recommend? a. Acetaminophen (Tylenol) b. Aspirin (Ecotrin) c. Diflunisal (Dolobid) d. Ibuprofen (Motrin)

ANS: A a. Acetaminophen (Tylenol) Acetaminophen is safe to give children and does not cause gastrointestinal upset or interfere with platelet aggregation. Aspirin carries an increased risk of Reye's syndrome in children. Diflunisal (Dolobid) is not available over the counter.

(Mod 9) 13. The nurse is caring for a patient who was admitted with a fractured leg and for observation of a closed head injury after a motor vehicle accident. The patient reports having pain at a level of 3 on a 1 to 10 pain scale. The nurse will expect the provider to order which analgesic medication for this patient? a. Acetaminophen (Tylenol) PO b. Hydromorphone HCl (Dilaudid) IM c. Morphine sulfate PCA d. Transdermal fentanyl (Duragesic)

ANS: A a. Acetaminophen (Tylenol) PO Use of opioid analgesics is contraindicated for patients with head injuries because of the risk of increased intracranial pressure. If opioids are necessary because of severe pain, they must be given in reduced doses. This patient is experiencing mild pain, so acetaminophen is an appropriate analgesic.

(Mod 12) 9. The nurse is admitting a patient who has been taking minoxidil (Loniten) to treat hypertension. Prior to beginning therapy with this medication, the patient had a blood pressure of 170/95 mm Hg and a heart rate of 72 beats per minute. The nurse assesses the patient and notes a blood pressure of 130/72 mm Hg and a heart rate of 78 beats per minute, and also notes a 2.2-kg weight gain since the previous hospitalization and edema of the hands and feet. The nurse will contact the provider to discuss which intervention? a. Adding hydrochlorothiazide to help increase urine output b. Adding metoprolol (Lopressor) to help decrease the heart rate c. Increasing the dose of minoxidil to lower the blood pressure d. Restricting fluids to help with weight reduction

ANS: A a. Adding hydrochlorothiazide to help increase urine output Minoxidil is a direct-acting vasodilator which can cause sodium and water retention. Combining this drug with a diuretic can help reduce edema by increasing urine output. If the patient were tachycardic, a beta blocker might be added. It is not necessary to increase the minoxidil dose or to restrict fluids.

11. A 2-year-old child will receive several doses of an intramuscular medication. The nurse caring for this child will use which intervention to help the child cope with this regimen? a. Allowing the child to give "pretend" shots to a doll with an empty syringe b. Allowing the child to select a Band-Aid to wear after each medication is given c. Ensuring privacy while giving the medication d. Explaining that the medicine will help the child to feel better

ANS: A a. Allowing the child to give "pretend" shots to a doll with an empty syringe Simple explanations, a firm approach, and enlisting the imagination of a toddler through play may enhance cooperation. Allowing the child to practice on a doll may help the toddler tolerate the injections. Preschool and school-age children fear bodily injury, and Band-Aids are important with those age groups. Adolescents need privacy, and school-age children and adolescents can understand the use of a medication in relation to future outcomes.

(Mod 9) 1. Which are characteristic signs of inflammation? (Select all that apply.) a. Edema b. Erythema c. Heat d. Numbness e. Pallor f. Paresthesia

ANS: A, B, C a. Edema b. Erythema c. Heat Edema, erythema, and heat are signs of inflammation. The other three are signs of neurocirculatory compromise

7. The nurse is preparing to administer a chewable tablet to a preschool-age child. The child's parent reports always crushing the tablet and mixing it with pudding when giving it at home. What is the nurse's next action? a. Ask the pharmacist if the drug may be crushed. b. Crush the tablet and mix it with pudding. c. Insist that the tablet must be chewed as ordered. d. Request a liquid form of the medication from the pharmacy.

ANS: A a. Ask the pharmacist if the drug may be crushed. Nurses should not crush or mix medications in other substances without consultation with a pharmacist or a reliable drug reference. Even if the family has been doing this at home, the nurse must still determine safety and efficacy. If the medication cannot be crushed or mixed into another substance, the nurse may need to insist on the child taking the dose as ordered or may need to ask the provider to prescribe a different form of the medication.

3. A health care provider calls a nursing unit to leave a telephone order for a PRN antipyretic medication for a child. The provider tells the nurse to "give PO acetaminophen for a fever greater than 101° F per protocol." What will the nurse do next? a. Ask the provider to verify how many mg per kg per dose and how frequently to give the medication. b. Look up the protocol in the unit manual and write the drug order for the provider to sign. c. Provide the child's weight to the hospital pharmacist to write the order based on the protocol. d. Transcribe the verbal order to the order sheet as "give PO acetaminophen for a fever greater than 101° F per protocol."

ANS: A a. Ask the provider to verify how many mg per kg per dose and how frequently to give the medication. The components of a drug order include drug name, dose, route, frequency, and any instructions for dosing. A nurse receiving a telephone order should "read back" the order from the provider that includes this information. The provider, and not the nurse or the pharmacist, is responsible for writing the order with all components.

2. An older patient who reports a 2- to 3-year history of upper gastrointestinal symptoms will begin taking ranitidine (Zantac) to treat this disorder. The patient has completed a health history form. The nurse notes that the patient answered "no" when asked if any medications were being taken. Which action will the nurse take next? a. Ask whether the patient uses over-the-counter (OTC) medications. b. Obtain a careful dietary history for the past two weeks. c. Recommend that the patient take antacid tablets. d. Suggest that the patient add high-potassium foods to the diet.

ANS: A a. Ask whether the patient uses over-the-counter (OTC) medications. Many patients do not think of OTC products as medications and often do not list them when asked about medication use. A patient who takes ranitidine along with an OTC antacid could be duplicating medications. A dietary history is important as well but would not be the most important action in this case. The nurse should not recommend antacid tablets or high-potassium foods.

10. The nurse is performing an admission assessment on an 80-year-old patient who has frequent hospital admissions. The patient appears more disoriented and confused than usual. Which action by the nurse is correct? a. Asking about medication doses b. Asking for a neurologist consult c. Requesting orders for liver function tests d. Suspecting impaired renal function

ANS: A a. Asking about medication doses An initial sign of drug toxicity in elderly patients may be confusion or changes in behavior. The nurse should ask about drug doses and notify the provider of the behaviors. The provider may order further evaluation based examination of the patient.

(Mod 13) A patient who has recently had a myocardial infarction (MI) will begin taking clopidogrel (Plavix) to prevent a second MI. Which medication will the nurse expect the provider to order as adjunctive therapy for this patient? a. Aspirin b. Enoxaparin sodium (Lovenox) c. Ticagrelor (Brilinta) d. Warfarin (Coumadin)

ANS: A a. Aspirin Aspirin is often used with clopidogrel to inhibit platelet aggregation to increase the effectiveness of this drug. Enoxaparin is used to prevent venous thrombosis. Ticagrelor is similar to clopidogrel and is not used along with clopidogrel. Warfarin is used to prevent thrombosis

4. The nurse is preparing to administer a medication and reviews the patient's chart for drug allergies, serum creatinine, and blood urea nitrogen (BUN) levels. The nurse's actions are reflective of which phase of the nursing process? a. Assessment b. Evaluation c. Implementation d. Planning

ANS: A a. Assessment Assessment involves gathering information about the patient and the drug including any previous use of the drug.

(Mod 8) 16. A patient who is intubated develops bradycardia because of vagal stimulation. Which medication will the nurse anticipate administering to treat this symptom? a. Atropine sulfate (Atropine) b. Benztropine (Cogentin) c. Bethanechol chloride (Urecholine) d. Metoclopramide (Reglan)

ANS: A a. Atropine sulfate (Atropine) Atropine is used to treat bradycardia caused by vagal stimulation. (Atropine is one of the cardiac code drugs)

(Mod 8) 6. The nurse is caring for a male patient with myasthenia gravis who will begin taking ambenonium chloride (Mytelase). When performing a health history, the nurse will be concerned about a history of which condition in this patient? a. Benign prostatic hypertrophy b. Chronic constipation c. Erectile dysfunction d. Upper respiratory infection

ANS: A a. Benign prostatic hypertrophy This drug is a reversible cholinesterase inhibitor and is given to increase muscle strength. Cholinesterase inhibitors are contraindicated in patients with urinary tract obstruction. (Cholinesterase inhibs increase ACh, increase Parasym effects -> increase GI motility. CI if have GI or urinary obstruction)

(Mod 9) 16. A postoperative patient has a history of opioid abuse. Which analgesic medication will the nurse expect the provider to order for this patient? a. Buprenorphine (Buprenex) b. Butorphanol tartrate (Stadol) c. Naloxone (Narcan) d. Pentazocine (Talwin)

ANS: A a. Buprenorphine (Buprenex) Buprenorphine is an opioid agonist-antagonist analgesic and was developed to help decrease opioid abuse. Butophanol and pentazocine are also in this class, but reports say that they cause dependence. Naloxone is an opioid antagonist and is given to reverse the effects of opioids if toxicity occurs.

(Mod 8) 10. A patient is taking doxazosin mesylate (Cardura) 1 mg per day to treat hypertension. The nurse notes a blood pressure of 110/72 mm Hg and a heart rate of 92 beats per minute. The nurse will contact the provider to discuss which change to the drug regimen? a. Changing to a beta-adrenergic blocker b. Decreasing the drug dose c. Increasing the drug dose d. Adding a diuretic

ANS: A a. Changing to a beta-adrenergic blocker Alpha-adrenergic blockers can cause orthostatic hypotension and reflex tachycardia. Beta blockers do not cause reflex tachycardia. Decreasing or increasing the drug dose is not recommended. Diuretics are added if blood pressure is not well-controlled.

(Mod 9) 5. The nurse is caring for a postoperative older patient who received PO hydrocodone with acetaminophen (Lortab) 45 minutes prior after reporting a pain level of 8 on a scale of 1 to 10. The patient reports a pain level of 4, and the nurse notes a respiratory rate of 20 breaths per minute, a heart rate of 92 beats per minute, and a blood pressure of 170/95 mm Hg. Which action will the nurse take? a. Contact the provider and request an order for a more potent opioid analgesic. b. Reassess the patient in 30 minutes. c. Request an order for ibuprofen to augment the opioid analgesic. d. Suggest that the patient use nonpharmacologic measures to relieve pain.

ANS: A a. Contact the provider and request an order for a more potent opioid analgesic. Even though the patient reports decreased pain, the patient's vital signs indicate continued discomfort. The nurse should contact the provider to request a stronger analgesic. The pain medication should have been effective within 30 minutes. Ibuprofen is used for musculoskeletal pain. Nonpharmacologic measures may be useful, but the patient still needs a stronger analgesic.

(Mod 13) A patient is taking clopidogrel bisulfate (Plavix). When teaching this patient about dietary restrictions while taking this medication, the nurse will instruct the patient to avoid excessive consumption of which food? a. Garlic b. Grapefruit c. Green, leafy vegetables d. Red meats

ANS: A a. Garlic Patients taking this drug may experience increased bleeding when taken with garlic. There is no restriction for grapefruit as there is with many other medications. Green, leafy vegetables should be restricted in patients taking warfarin. Red meats are not contraindicated.

3. A 5-year-old child with type 1 diabetes mellitus has repeated hospitalizations for episodes of hyperglycemia related to poor control. The parents tell the nurse that they can't keep track of everything that has to be done to care for their child. The nurse reviews medications, diet, and symptom management with the parents and draws up a daily checklist for the family to use. This is an example of the principles outlined in a. Guiding Principles of Patient Engagement. b. National Alliance for Quality Care. c. Nursing Process. d. Quality and Safety Education for Nurses.

ANS: A a. Guiding Principles of Patient Engagement Guiding Principles of Patient Engagement address the dynamic partnership among patients, families, and health care providers.

(Mod 8) 1. The nurse is caring for a patient who has asthma and administers a selective beta2-adrenergic agonist to treat bronchospasm. The nurse will expect this drug to also cause which side effect a. Increased blood glucose b. Increased blood pressure c. Increased heart rate d. Increased gastrointestinal (GI) motility

ANS: A a. Increased blood glucose Drugs that act on beta2 receptors activate glyconeogenesis in the liver causing increased blood glucose. Selective beta2 drugs act on beta2 receptors only and not on beta1 receptors, so they do not cause increased blood pressure or increased heart rate. Adrenergic agonists cause decreased GI motility.

(Mod 13) The nurse is caring for a patient who is receiving warfarin (Coumadin) and notes bruising and petechiae on the patient's extremities. The nurse will request an order for which laboratory test? a. International normalized ratio (INR) b. Platelet level c. PTT and aPTT d. Vitamin K level

ANS: A a. International normalized ratio (INR) The INR is the test used most frequently to report prothrombin time results in patients taking warfarin. Warfarin is not an antiplatelet drug, so platelet levels are not indicated. PTT and aPTT are used to monitor heparin therapy. Vitamin K is an antidote for warfarin; levels are not routinely checked.

(Mod 9) 3. A patient who is 7 months pregnant and who has arthritis asks the nurse if she can take aspirin for pain. The nurse will tell her not to take aspirin for which reason? a. It can result in adverse effects on her fetus. b. It causes an increased risk of Reye's syndrome. c. It increases hemorrhage risk. d. It will cause increased gastrointestinal distress.

ANS: A a. It can result in adverse effects on her fetus. Patients should not take aspirin during the third trimester of pregnancy because it can cause premature closure of the ductus arteriosus in the fetus. It does not increase her risk of Reye's syndrome. Aspirin taken within a week of delivery will increase the risk of bleeding. It can cause gastrointestinal distress, but this is not the reason for caution.

(Mod 8) 9. A patient will begin taking albuterol (Proventil) to treat asthma. When teaching the patient about this drug, the nurse will make which recommendation? a. Report rapid or irregular heart rate. b. Drink 8 to 16 extra ounces of fluid each day. c. Monitor serum glucose daily. d. Take a calcium supplement.

ANS: A a. Report rapid or irregular heart rate. High dosages of albuterol may affect beta1 receptors, causing an increase in heart rate that could be dangerous. It is not necessary to consume extra fluids or take a calcium supplement while using this drug. Serum glucose may be elevated slightly, but this is not a concern in non-diabetic patients.

(Mod 8) 6. The nurse is teaching a patient how to use phenylephrine HCl (Neo-Synephrine) nasal spray to treat congestion from a viral upper respiratory infection. What instruction will the nurse give the patient? a. Stop using the medication after 3 days. b. Spray the medication into the nose while lying supine. c. Use frequently since systemic side effects do not occur. d. Use the medication with any other over-the-counter medications

ANS: A a. Stop using the medication after 3 days. Nurses should explain to patients that continuous use of nasal sprays containing adrenergic agonists may result in rebound nasal congestion; these sprays should not be used more than 3 days. To avoid systemic absorption, spray should be administered while the patient is in an upright position. The medication may cause systemic side effects and should not be routinely used with other OTC cold medications.

(Mod 13) A nursing student asks why the anticoagulant heparin is given to patients who have disseminated intravascular coagulation (DIC) and are at risk for excessive bleeding. The nurse will explain that heparin is used in this case for which reason? a. To decrease the risk of venous thrombosis b. To dissolve blood clots as they form c. To enhance the formation of fibrous clots d. To preserve platelet function

ANS: A a. To decrease the risk of venous thrombosis The primary use of heparin for patients with DIC is to prevent venous thrombosis, which can lead to pulmonary embolism or stroke. Heparin does not break down blood clots, enhance the formation of fibrous clots, or preserve platelet function.

(Mod 7) 1. Which are characteristic signs of inflammation? (Select all that apply.) a. Edema b. Erythema c. Heat d. Numbness e. Pallor f. Paresthesia

ANS: A, B, C a. Edema b. Erythema c. Heat Edema, erythema, and heat are signs of inflammation. The other three are signs of neurocirculatory compromise

(Mod 13) The nurse is assessing a patient who takes warfarin (Coumadin). The nurse notes a heart rate of 92 beats per minute and a blood pressure of 88/78 mm Hg. To evaluate the reason for these vital signs, the nurse will assess the patient's a. gums, nose, and skin. b. lung sounds and respiratory effort. c. skin turgor and oral mucous membranes. d. urine output and level of consciousness.

ANS: A a. gums, nose, and skin. An increased heart rate followed by a decreased systolic pressure can indicate a fluid volume deficit caused by internal or external bleeding. The nurse should examine the patient's mouth, nose, and skin for bleeding. These vital signs do not indicate a pulmonary problem. Skin turgor and mucous membranes as well as urine output and level of consciousness may be assessed to determine the level of fluid deficit, but finding the source of blood loss is more important. Signs of gastrointestinal bleeding should also be assessed.

(Mod 8) 2. A nursing student asks why a direct-acting cholinergic agonist drug that is selective to muscarinic receptors is described as being non-specific. The nurse will explain that this is because a. muscarinic receptors are present in many different tissues. b. the action of cholinesterase alters the bioavailability at different sites. c. these drugs can also affect nicotinic receptors. d. they vary in their reversible and irreversible effects.

ANS: A a. muscarinic receptors are present in many different tissues. Although drugs classified as direct-acting cholinergic agonists are primarily selective for muscarinic receptors, they are non-specific because muscarinic receptors are located in different sites, causing actions in various organs. They are not affected differently by cholinesterase activity and have negligible actions on nicotinic receptors.

(Mod 8) 4. An adult patient is brought to the emergency department for treatment of an asthma exacerbation. The patient uses inhaled albuterol as needed to control wheezing. The nurse notes expiratory wheezing, tremors, restlessness, and a heart rate of 120 beats per minute. The nurse suspects that the patient has a. over-used the albuterol. b. not been using albuterol. c. taken a beta-adrenergic blocker. d. taken a monoamine oxidase (MAO) inhibitor.

ANS: A a. over-used the albuterol. High doses of albuterol may affect beta1 receptors, causing an increase in heart rate. Patients having an asthma exacerbation may over-use their albuterol inhalers when seeking relief. Patients may have wheezing and increased heart rate during an untreated asthma exacerbation, but they will not have tremors and restlessness.

(Mod 13) The nurse has just begun administering intravenous streptokinase (Streptase). The nurse assesses vital signs and notes a temperature of 37° C, a heart rate of 70 beats per minute, and a blood pressure of 88/58 mm Hg. The nurse will contact the provider to a. request an adjustment of the streptokinase dose. b. request an order for aminocaproic acid (Amicar). c. request epinephrine to prevent anaphylaxis. d. report potential hemorrhage in this patient.

ANS: A a. request an adjustment of the streptokinase dose. Patients receiving streptokinase may experience hypotension when it is first administered and may require an adjustment in dosage. Aminocaproic acid is used to stop bleeding. Epinephrine is given for anaphylaxis, which is characterized by difficulty breathing. A patient with hemorrhage would typically also have tachycardia.

(Mod 6) 13. The nurse is caring for a patient who takes low-dose erythromycin as a prophylactic medication. The patient will begin taking cefaclor for treatment of an acute infection. The nurse should discuss this with the provider because taking both of these medications simultaneously can cause which effect? a. Decreased effectiveness of cefaclor. b. Increased effectiveness of cefaclor. c. Decreased effectiveness of erythromycin. d. Increased effectiveness of erythromycin.

ANS: A a. Decreased effectiveness of cefaclor. The interaction of cefaclor and erythromycin will produce a decrease in the action of the cefaclor.

(Mod 11) 5. Which statement by a patient taking glipizide indicates that more teaching is indicated? a. "I will use a new needle every time I take the medication." b. "I will take the medication once a day in the morning." c. "I will eat my breakfast very soon after taking my Glucotrol." d. "This medication stimulates my pancreatic cells to make insulin."

ANS: A Glipizide (Glucotrol) is an oral antidiabetic agent. It is well absorbed from the GI tract and is highly protein-bound. Parenteral administration of this medication is not indicated. All other options are correct.

(Mod 11) 3. A patient received regular insulin at 7:30 am. At 9:30 am the patient feels slightly hungry and has a dull headache. The nurse should a. test the patient's blood glucose level. b. ensure that the patient has a meal. c. provide the patient with 4 ounces of orange juice. d. administer the next dose of insulin.

ANS: A The peak time for regular insulin is 2 to 4 hours. It is most important for the nurse to check the patient's blood glucose level to prevent a possible hypoglycemic reaction (insulin shock).

18. A patient who takes warfarin (Coumadin) asks the nurse about taking a medication for headaches. The nurse will recommend which medication? a. Acetaminophen b. Aspirin c. Ibuprofen d. No medication

ANS: A a. Acetaminophen Aspirin and NSAIDs can increase the risk of hemorrhage in patients taking anticoagulants. Acetaminophen is safe and may be recommended.

23. The nurse is preparing to administer the first dose of digoxin (Lanoxin) to a patient and notes that the dose ordered is much higher than the usual recommended dose. Which action will the nurse perform? a. Administer the dose as ordered. b. Give the dose and monitor for toxicity. c. Hold the dose until reviewing it with the provider. d. Refuse to give the dose.

ANS: A a. Administer the dose as ordered. Digoxin requires a loading dose when first prescribed.

5. The nurse will administer an intravenous medication to an adolescent patient. When preparing the adolescent for the IV insertion, which is an appropriate action by the nurse? a. Allowing the patient to verbalize concerns about the procedure b. Covering the insertion site with a bandage after the procedure is completed c. Explaining any possible adverse drug reactions d. Reassuring the patient that only one body part will be used

ANS: A a. Allowing the patient to verbalize concerns about the procedure Allowing the adolescent to verbalize concerns about the medication and its regimen may offer opportunities to clarify misconceptions and teach new information. Preschool-age children may have concerns about harm to their body and need to have sites covered. Adolescents still have a present focus, so discussing future adverse reactions is not especially helpful. Preschool and school-age children fear bodily harm and require reassurance that only one body part will be affected.

5. The nurse is caring for a patient who is a member of the local Native American community. The patient is refusing medications and treatments in spite of repeated attempts to explain the importance of these interventions. Which is an appropriate nursing action? a. Ask a family member about traditional healing practices that might be better accepted. b. Enlist the help of a family member to explain the need for the medications and treatments. c. Find a hospital staff member who is Native American to help provide teaching for this patient. d. Suggest a Social Work consult to the patient's provider.

ANS: A a. Ask a family member about traditional healing practices that might be better accepted. Members of some cultures may use traditional healers, and this should be accommodated whenever possible. Showing respect for this patient's culture will help to establish trust and thus greater cooperation. It is important for the nurse not to make generalizations within and among cultural groups, so asking a family member to describe what this particular patient needs is the better choice. Finding a hospital staff member who is Native American assumes that all Native Americans have the same practices. Deferring to a Social Worker is not necessary. Enlisting a family member to explain the need for the medications is just another way of imposing treatments on this person without respecting their cultural needs.

9. The nurse is caring for an African-American patient who is taking warfarin (Coumadin) to prevent blood clots. The nurse will monitor this patient carefully for which effect? a. Decreased therapeutic effects b. Heightened risk for hemorrhage c. Increased risk of hypersensitivity d. Potential risk of paradoxical effects

ANS: A a. Decreased therapeutic effects Certain classifications of medications have different effects in individuals whose genetic markers are predominantly characteristic of a certain biologic group. African-American patients will tend to have a decreased therapeutic effect from warfarin (Coumadin).

7. The nurse is preparing an injectable drug and wants to administer it for rapid absorption. How will the nurse give this medication? a. IM into the deltoid muscle b. IM into the gluteal muscle c. SubQ into abdominal tissue d. SubQ into the upper arm

ANS: A a. IM into the deltoid muscle Drugs given IM are absorbed faster in muscles that have more blood vessels, such as the deltoid, rather than those with fewer blood vessels, such as the gluteals. Subcutaneous routes are used when absorption needs to be slower and more sustained.

20. The nurse administers albuterol to a patient who has asthma. The albuterol acts by stimulating beta2-adrenergic receptors to cause bronchodilation. The nurse understands that albuterol is a beta-adrenergic a. agonist. b. antagonist. c. inhibitor. d. depressant.

ANS: A a. agonist. An agonist medication is one that stimulates a certain type of cell to produce a response

9. The nurse prepares to change a patient's medication from an intravenous to an oral form and notes that the oral form is ordered in a higher dose. The nurse understands that this is due to differences in a. bioavailability. b. pinocytosis. c. protein binding. d. tachyphylaxis.

ANS: A a. bioavailability. Oral drugs may have less bioavailability because a lower percentage of the drug reaches the systemic circulation. Pinocytosis refers to the process by which cells carry a solute across a membrane. Protein binding can occur with both routes. Tachyphylaxis describes a rapid decrease in response to drugs that occurs when tolerance develops quickly.

5. The nurse is preparing to administer erythromycin to a patient who takes digoxin. The nurse will plan to monitor the patient for a. digoxin toxicity. b. decreased digoxin effects. c. erythromycin toxicity. d. decreased erythromycin effects.

ANS: A a. digoxin toxicity Intestinal flora have the ability to metabolize digoxin, and any drug that destroys or inhibits growth of these gastrointestinal microflora can increase digoxin levels leading to toxicity.

6. A young adult female patient who takes a combination oral contraceptive (OCP) will begin taking an antibiotic. When teaching the patient about this medication, the nurse will a. recommend using a backup method of contraception. b. suggest that she switch to an injectable form of contraception. c. tell her that the antibiotic is less effective if she is taking OCPs. d. tell her the antibiotic has a greater risk for toxicity while taking OCPs.

ANS: A a. recommend using a backup method of contraception. Gut bacteria are necessary to hydrolyze estrogen conjugates into free estrogens. Concurrent antibiotic administration can alter these bacteria and prevent the optimal absorption and effectiveness of OCPs. A backup contraceptive method is recommended.

(Mod 6) 10. The nurse is teaching a nursing student about the antifungal drug amphotericin B. Which statement by the student indicates a need for further teaching? a. "Amphotericin B may be given intravenously or by mouth." b. "Patients who take this drug should have potassium and magnesium levels assessed." c. "Patients with renal disease should not take amphotericin B." d. "This drug is used for severe systemic infections."

ANS: A a. "Amphotericin B may be given intravenously or by mouth." Amphotericin B is not absorbed from the gastrointestinal tract, so is not given by mouth. It can cause nephrotoxicity and electrolyte imbalance. It is highly toxic and is reserved for severe, systemic infections.

(Mod 6) 2. The nurse is counseling a patient who will begin taking a sulfonamide drug to treat a urinary tract infection. What information will the nurse include in teaching? a. "Drink several quarts of water daily." b. "If stomach upset occurs, take an antacid." c. "Limit sun exposure to no more than 1 hour each day." d. "Sore throat is a common, harmless side effect."

ANS: A a. "Drink several quarts of water daily." Patients should drink several quarts of water daily while taking sulfonamides to prevent crystalluria. Patients should not take antacids with sulfonamides. Patients should not go out into the sun. Sore throat should be reported.

(Mod 6) 12. The nurse is providing teaching to a patient who will begin taking a cephalosporin to treat an infection. Which statement by the patient indicates a need for further teaching? a. "I may stop taking the medication if my symptoms clear up." b. "I should eat yogurt while taking this medication." c. "I should stop taking the drug and call my provider if I develop a rash." d. "I will not consume alcohol while taking this medication."

ANS: A a. "I may stop taking the medication if my symptoms clear up." Patients should take all of an antibiotic regimen even after symptoms clear to ensure complete treatment of the infection. Patients are often advised to eat yogurt or drink buttermilk to prevent superinfection. A rash is a sign of hypersensitivity, and patients should be counseled to stop taking the drug and notify the provider if this occurs. Alcohol consumption may cause adverse effects and should be avoided by patients while they are taking cephalosporins.

(Mod 6) 5. The nurse provides home care instructions for a patient who will take a high dose of azithromycin after discharge from the hospital. Which statement by the patient indicates understanding of the teaching? a. "I may take antacids 2 hours before taking this drug." b. "I should take acetaminophen for fever or mild pain." c. "I should expect diarrhea to be a common, mild side effect." d. "I should avoid dairy products while taking this drug."

ANS: A a. "I may take antacids 2 hours before taking this drug." Azithromycin peak levels may be reduced by antacids when taken at the same time so patients should be cautioned to take antacids 2 hours before or 2 hours after taking the drug. High-dose azithromycin carries a risk for hepatotoxicity when taken with other potentially hepatotoxic drugs such as acetaminophen. Diarrhea may indicate pseudomembranous colitis and should be reported. There is no restriction for dairy products when taking azithromycin.

(Mod 7) 2. The nurse is providing teaching to a patient who will begin taking aspirin to treat arthritis pain. Which statement by the patient indicates a need for further teaching? a. "I should increase fiber and fluids while taking aspirin." b. "I will call my provider if I have abdominal pain." c. "I will drink a full glass of water with each dose." d. "I will notify my provider of ringing in my ears."

ANS: A a. "I should increase fiber and fluids while taking aspirin." Aspirin is not constipating, so patients do not need to be counseled to consume extra fluids and fiber. Abdominal pain can occur with gastrointestinal bleeding, and tinnitus (ringing in the ears) can be an early sign of toxicity, so patients should be taught to contact their provider if these occur. Taking a full glass of water with each dose helps minimize gastrointestinal side effects.

(Mod 6) 8. The nurse is teaching a patient about rifampin. Which statement by the patient indicates understanding of the teaching? a. "I should not wear soft contact lenses while taking rifampin." b. "I will need regular eye examinations while taking this drug." c. "I will report orange urine to my provider immediately." d. "I understand that renal toxicity is a common adverse effect."

ANS: A a. "I should not wear soft contact lenses while taking rifampin." Patients taking rifampin should be warned that urine, feces, saliva, sputum, sweat, and tears may turn a harmless red-orange color. Patients should not wear soft contact lenses to avoid permanent staining. Regular eye exams are necessary for patients who receive isoniazid and ethambutol. Orange urine is a harmless side effect and does not need to be reported. Renal toxicity is not common with rifampin.

(Mod 7) 12. A patient who has osteoarthritis with mild to moderate pain asks the nurse about taking over-the-counter ibuprofen (Motrin). What will the nurse tell this patient? a. "It may take several weeks to achieve therapeutic effects." b. "Unlike aspirin, there is no increased risk of bleeding with ibuprofen." c. "Take ibuprofen twice daily for maximum analgesic benefit." d. "Combine ibuprofen with acetaminophen for best effect."

ANS: A a. "It may take several weeks to achieve therapeutic effects." OTC NSAIDs can be effective for mild to moderate arthritis pain, but the effects may not appear for several weeks. NSAIDs carry a risk for bleeding. Ibuprofen is taken every 4 hours or QID. Ibuprofen should not be combined with aspirin or acetaminophen.

(Mod 6) 10. The nurse is preparing to administer the first dose of intravenous ceftriaxone (Rocephin) to a patient. When reviewing the patient's chart, the nurse notes that the patient previously experienced a rash when taking amoxicillin. What is the nurse's next action? a. Administer the drug and observe closely for hypersensitivity reactions. b. Ask the provider whether a cephalosporin from a different generation may be used. c. Contact the provider to report drug hypersensitivity. d. Notify the provider and suggest an oral cephalosporin.

ANS: A a. Administer the drug and observe closely for hypersensitivity reactions. A small percentage of patients who are allergic to penicillin could also be allergic to a cephalosporin product. Patients should be monitored closely after receiving a cephalosporin if they are allergic to penicillin. There is no difference in hypersensitivity potential between different generations or method of delivery of cephalosporins.

(Mod 7) 17. Which antigout medication is used to treat chronic tophaceous gout? a. Allopurinol (Zyloprim) b. Colchicine c. Probenecid (Benemid) d. Sulfinpyrazone (Anturane)

ANS: A a. Allopurinol (Zyloprim) Allopurinol inhibits the biosynthesis of uric acid and is used long-term to manage (tophaceous) chronic gout. Colchicine does not inhibit uric acid synthesis or promote uric acid secretion and is not used for chronic gout. Probenecid can be used for chronic gout but is not the first choice. Sulfinpyrazone has many serious side effects.

(Mod 6) 14. A young adult female who is taking metronidazole (Flagyl) to treat trichomoniasis calls the nurse to report severe headache, flushing, palpitations, cramping, and nausea. What will the nurse do next? a. Ask about alcohol consumption. b. Reassure her that these are harmless side effects. c. Tell her that this signals a worsening of her infection. d. Tell her to go to the emergency department immediately.

ANS: A a. Ask about alcohol consumption. Patients who are taking metronidazole can experience a disulfiram-like reaction when they drink alcohol. These are not harmless adverse effects or a sign of worsening of her infection.

(Mod 7) 7. The nurse is performing a health history on a patient who has arthritis. The patient reports tinnitus. Suspecting a drug adverse effect, the nurse will ask the patient about which medication? a. Aspirin (Bayer) b. Acetaminophen (Tylenol) c. Anakinra (Kineret) d. Prednisone (Deltasone)

ANS: A a. Aspirin (Bayer) Aspirin causes tinnitus at low toxicity levels. The nurse should question the patient about this medication. The other medications do not have this side effect.

(Mod 7) 9. A patient who takes high-dose aspirin to treat rheumatoid arthritis has a serum salicylate level of 35 mg/dL. The nurse will perform which action? a. Assess the patient for tinnitus. b. Monitor the patient for signs of Reye's syndrome. c. Notify the provider of severe aspirin toxicity. d. Request an order for an increased aspirin dose.

ANS: A a. Assess the patient for tinnitus. Mild toxicity occurs at levels above 30 mg/dL, so the nurse should assess for signs of toxicity, such as tinnitus. This level will not increase the risk for Reye's syndrome. Severe toxicity occurs at levels greater than 50 mg/dL. The dose should not be increased.

(Mod 6) 12. The nurse is caring for a patient who is taking trimethoprim-sulfamethoxazole (TMP-SMX). The nurse learns that the patient takes an angiotension-converting enzyme (ACE) inhibitor. To monitor for drug interactions, the nurse will request an order for which laboratory test(s)? a. A complete blood count b. BUN and creatinine c. Electrolytes d. Glucose

ANS: C c. Electrolytes TMP-SMX can result in hyperkalemia when taken with an ACE inhibitor.

(Mod 7) 3. A patient is taking ibuprofen 400 mg every 4 hours to treat moderate arthritis pain and reports that it is less effective than before. What action will the nurse take? a. Counsel the patient to discuss a prescription NSAID with the provider. b. Recommend adding aspirin to increase the antiinflammatory effect. c. Suggest asking the provider about a short course of corticosteroids. d. Tell the patient to increase the dose to 800 mg every 4 hours.

ANS: A a. Counsel the patient to discuss a prescription NSAID with the provider. The patient should discuss another NSAID with the provider if tolerance has developed to the over-the-counter NSAID. Patients should not take aspirin with NSAIDs because of the increased risk of bleeding and gastrointestinal upset. Steroids are not the drugs of choice for arthritis because of their side effects and are not used unless inflammation is severe. A prescription NSAID would be used prior to starting corticosteroids. Increasing the dose will increase side effects but may not increase desired effects. The maximum dose per day is 2400 mg, which would most likely be exceeded when increasing the dose to 800 mg every 4 hours.

(Mod 6) 6. The nurse receives the following order for a patient who is diagnosed with herpes zoster virus: PO acyclovir (Zovirax) 400 mg TID for 7 to 10 days. The nurse will contact the provider to clarify which part of the order? a. Dose and frequency b. Frequency and duration c. Drug and dose d. Drug and duration

ANS: A a. Dose and frequency Acyclovir is used for herpes zoster, but the dose should be 800 mg 5 times daily for 7 to 10 days. The nurse should clarify the dose and frequency. For herpes simplex, 400 mg 3 times daily is correct.

(Mod 7) 18. The nurse is assessing a patient who has gout who will begin taking allopurinol (Zyloprim). The nurse reviews the patient's medical record and will be concerned about which laboratory result? a. Elevated BUN and creatinine b. Increased serum uric acid c. Slight increase in the white blood count d. Increased serum glucose

ANS: A a. Elevated BUN and creatinine Antigout drugs are excreted via the kidneys, so patients should have adequate renal function.

(Mod 6) 2. The nurse is caring for a patient who is receiving an intravenous antibiotic. The patient has a serum drug trough of 1.5 mcg/mL. The normal trough for this drug is 1.7 mcg/mL to 2.2 mcg/mL. What will the nurse expect the patient to experience? a. Inadequate drug effects b. Increased risk for superinfection c. Minimal adverse effects d. Slowed onset of action

ANS: A a. Inadequate drug effects Low peak levels may indicate that the medication is below the therapeutic level. They do not indicate altered risk for superinfection, a decrease in adverse effects, or a slowed onset of action.

(Mod 6) 5. The nurse is caring for an infant who has respiratory syncytial virus (RSV) and who will receive ribavirin. The nurse expects to administer this drug by which route? a. Inhalation b. Intramuscular c. Intravenous d. Oral

ANS: A a. Inhalation Ribavirin is given by inhalation to treat RSV. Oral ribavirin is used to treat hepatitis C, and intravenous ribavirin is used to treat hepatitis C and Lassa fever

(Mod 6) 5. The nurse is caring for a patient who is diagnosed with tuberculosis. The patient tells the nurse that the provider plans to order a prophylactic antitubercular drug for family members and asks which drug will be ordered. The nurse will expect the provider to order which drug? a. Isoniazid (INH) b. Pyrazinamide c. Rifampin (Rifadin) d. Streptomycin

ANS: A a. Isoniazid (INH) INH is the drug of choice for prophylactic treatment of patients who have had close contact with a patient who has tuberculosis.

(Mod 7) 6. A patient who is 7 months pregnant and who has arthritis asks the nurse if she can take aspirin for pain. The nurse will tell her not to take aspirin for which reason? a. It can result in adverse effects on her fetus. b. It causes an increased risk of Reye's syndrome. c. It increases hemorrhage risk. d. It will cause increased gastrointestinal distress.

ANS: A a. It can result in adverse effects on her fetus. Patients should not take aspirin during the third trimester of pregnancy because it can cause premature closure of the ductus arteriosus in the fetus. It does not increase her risk of Reye's syndrome. Aspirin taken within a week of delivery will increase the risk of bleeding. It can cause gastrointestinal distress, but this is not the reason for caution.

(Mod 6) 8. A patient is admitted to the hospital for treatment of pneumonia after complaining of high fever and shortness of breath. The patient was not able to produce sputum for a culture. The nurse will expect the patient's provider to order a. a broad-spectrum antibiotic. b. a narrow-spectrum antibiotic. c. multiple antibiotics. d. the pneumococcal vaccine.

ANS: A a. a broad-spectrum antibiotic. Broad-spectrum antibiotics are frequently used to treat infections when the offending organism has not been identified by culture and sensitivity (C&S). Narrow-spectrum antibiotics are usually effective against one type of organism and are used when the C&S indicates sensitivity to that antibiotic. The use of multiple antibiotics, unless indicated by C&S, can increase resistance. The pneumococcal vaccine is used to prevent, not treat, an infection.

(Mod 6) 4. The nurse is caring for a patient who has recurrent urinary tract infections. The patient's current infection is not responding to an antibiotic that has been used successfully several times in the past. The nurse understands that this is most likely due to a. acquired bacterial resistance. b. cross-resistance. c. inherent bacterial resistance. d. transferred resistance.

ANS: A a. acquired bacterial resistance. Acquired resistance occurs when an organism has been exposed to the antibacterial drug. Cross-resistance occurs when an organism that is resistant to one drug is also resistant to another. Inherent resistance occurs without previous exposure to the drug. Transferred resistance occurs when the resistant genes of one organism are passed to another organism.

(Mod 7) 1. A nursing student asks the nurse to explain the role of cyclooxygenase-2 (COX-2) and its role in inflammation. The nurse will explain that COX-2 a. converts arachidonic acid into a chemical mediator for inflammation. b. directly causes vasodilation and increased capillary permeability. c. irritates the gastric mucosa to cause gastrointestinal upset. d. releases prostaglandins, which cause inflammation and pain in tissues.

ANS: A a. converts arachidonic acid into a chemical mediator for inflammation COX-2 is an enzyme that converts arachidonic acid into prostaglandins and their products, and this synthesis causes pain and inflammation. They do not act directly to cause inflammation. COX-1 irritates the gastric mucosa. COX-2 synthesizes but does not release prostaglandins.

(Mod 12) The nurse teaches a patient about antihypertensive medication. Which statements by the patient indicate understanding of the teaching? (Select all that apply.) a. "I should be careful when I stand up from a chair." b. "I should not add extra salt to my foods." c. "If I have side effects, I should stop taking the drug immediately." d. "If my blood pressure returns to normal, I can stop taking this drug." e. "I may need to take a combination of drugs, including diuretics." f. "I will not need to make lifestyle changes since I am taking a medication."

ANS: A, B, E a. "I should be careful when I stand up from a chair." b. "I should not add extra salt to my foods." e. "I may need to take a combination of drugs, including diuretics." The patient receiving an antihypertensive medication should be warned to rise slowly to avoid orthostatic hypotension. Patients should be counseled to continue to make lifestyle changes, including decreasing sodium. Often, more than one medication is required. Patients should not stop taking the drug abruptly to avoid rebound hypertension and will not stop the drug when blood pressure returns to normal.

14. Which drug properties are problematic for older patients? (Select all that apply.) a. Drugs with anticholinergic effects b. Drugs that are highly protein-bound c. Drugs with a short half-life d. Drugs that undergo hepatic conjugation e. Drugs with a narrow therapeutic range

ANS: A, B, E a. Drugs with anticholinergic effects b. Drugs that are highly protein-bound e. Drugs with a narrow therapeutic range Older patients are more susceptible to drug side effects, especially those that cause anticholinergic effects. Older patients have a loss of protein-binder sites for drugs, so those that are highly protein-bound will have higher than usual serum levels and can cause toxicity. Drugs with a narrow therapeutic range require close monitoring in all patients, but especially in older patients. Drugs with a short half-life are preferred because older patients have a decreased ability to metabolize and excrete drugs. Hepatic conjugation is usually not influenced by older age, live diseases, or drug interaction.

(Mod 8) 18. The nurse caring for a patient who is taking an adrenergic agent will expect which side effects? (Select all that apply.) a. Dilated pupils b. Increased heart rate c. Increase gastrointestinal motility d. Vasodilation e. Bronchospasm f. Relaxed uterine muscles

ANS: A, B, F a. Dilated pupils b. Increased heart rate f. Relaxed uterine muscles Adrenergic agents stimulate the sympathetic nervous system, evoking the "fight or flight" response. This response increases those functions needed to respond to stress (increased heart rate to perfuse muscles, bronchodilation to increase oxygen exchange). Adrenergic drugs shunt blood away from the reproductive tract and gastrointestinal organs as these functions are not needed during a fight or flight response.

13. The nurse is preparing an 80-year-old patient for discharge home from the hospital. The patient will receive several new medications. The patient lives alone but has several family members who stop by every day. Which suggestions will the nurse make for this family? (Select all that apply.) a. Ask the pharmacy for non-childproof medication bottles. b. Ask the patient to record all medications and the times they are taken. c. Place the pills in an organizer container. d. Provide the patient with the drug manufacturer information sheets. e. Put water bottles near pills for convenience.

ANS: A, C, E a. Ask the pharmacy for non-childproof medication bottles. c. Place the pills in an organize container. e. Put water bottles near pills for convenience. To help older patients with compliance, medications should be convenient and easy to open. Asking the pharmacist for non-childproof containers will make medications easier to get. Using an organizer container helps patients remember which drugs should be taken at what time. Placing water bottles nearby eliminates a step in the process and increase the likelihood that a medication will be taken on time.

(Mod 6) 17. Which actions can contribute to bacterial resistance to antibiotics? (Select all that apply.) a. Frequent use of antibiotics b. Giving large doses of antibiotics c. Skipping doses d. Taking a full course of antibiotics e. Treating viral infections with antibiotics

ANS: A, C, E a. Frequent use of antibiotics c. Skipping doses e. Treating viral infections with antibiotics Frequent use of antibiotics increases the exposure of bacteria to an antibiotic and results in acquired resistance. Skipping doses of an antibiotic can lead to incomplete treatment of an infection, and the remaining bacteria may develop acquired resistance. Treating viral infections with antibiotics is unnecessary and may cause acquired resistance to develop from unneeded exposure to a drug. Infections adequately treated with an antibiotic do not result in resistance.

(Mod 8) 7. A client with myasthenia gravis is experiencing a cholinergic crisis. Which symptoms are associated with this condition? (Select all that apply.) a. Bradycardia b. Rash c. Vomiting d. Fever e. Drooling f. Weakness

ANS: A, C, E, F a. Bradycardia c. Vomiting e. Drooling f. Weakness Bradycardia, drooling, and weakness can all occur with cholinergic crisis.

(Mod 6) 21. A client is being treated for tuberculosis. Which medications are used to treat this condition? (Select all that apply.) a. Streptomycin sulfate b. Amoxicillin (Amoxil) c. Ethambutol (Myambutol) d. Gentamicin (Garamycin) e. Rifabutin (Mycobutin) f. Ethionamide (Trecator-SC) g. Pyrazinamide

ANS: A, C, E, F, G a. Streptomycin sulfate c. Ethambutol (Myambutol) e. Rifabutin (Mycobutin) f. Ethionamide (Trecator-SC) g. Pyrazinamide Streptomycin sulfate, ethambutol (Myambutol), rifabutin (Mycobutin), ethionamide (Trecator-SC), and pyrazinamide are used to treat tuberculosis. The other medications are not used.

(Mod 6) 17. Which diseases are caused by herpes viruses? (Select all that apply.) a. Chicken pox b. Hepatitis c. Influenza d. Mononucleosis e. Shingles

ANS: A, D, E a. Chicken pox d. Mononucleosis e. Shingles Herpes viruses cause chicken pox, mononucleosis, and shingles.

11. The nurse is caring for a patient who will begin taking a thiazide diuretic to treat hypertension. The patient says, "I know this will lower my blood pressure, but how does it work?" How will the nurse respond? a. "It can cause orthostatic hypotension, so be careful." b. "It reduces the volume of fluid in your blood stream to lower blood pressure." c. "The actions are complicated, but it's an effective drug." d. "Your provider should explain this medication to you."

ANS: B b. "It reduces the volume of fluid in your blood stream to lower blood pressure." Patients have a right to understand how the drugs they are taking work and to know about side effects. The nurse should explain how the medication can cause orthostatic hypotension after addressing the patient's current question. Telling the patient that the drug actions are complicated is disrespectful. Nurses are responsible for educating patients about medications.

13. Which is a violation of a nurse's right when administering medications? a. A hospital policy for off-label drug uses b. A medication preparation area at the unit secretary's desk c. A multiple-dose drug vial requiring the nurse to calculate and measure the dose d. A new drug ordered that the nurse must look up in a drug manual

ANS: B b. A medication preparation area at the unit secretary's desk Nurses have a right to administer drugs safely and have the right to stop, think, and be vigilant when administering medications. Another worker's desk will be noisy, with many distractions. Many drugs are used for off-label purposes; having a hospital policy helps ensure safety. Single-dose vials are more convenient and help to reduce calculation errors, but multi-dose vials are often used; nurses unsure of calculations should check their work with another nurse. Nurses frequently have to look up information on new drugs, and hospitals should offer reasonable access to current information.

10. The nurse assumes care of a patient who had surgery that morning. The provider has ordered hydrocodone (Lortab) every 4 hours for mild to moderate pain and morphine sulfate for moderate to severe pain. The nurse reviews the patient's record and notes the patient has received two doses of hydrocodone 4 hours apart for a pain level of 7 to 8 on a scale of 1 to 10 and has reported a decrease in pain to a level of 6 to 7 after 30 minutes. It has been 4 hours since the last dose, and the patient reports a pain level of 7. What will the nurse do? a. Administer the hydrocodone. b. Administer morphine. c. Ask the patient which drug to give. d. Notify the provider of the patient's current pain level.

ANS: B b. Administer morphine. The previous nurse has documented poor response to pain medication given for mild to moderate pain. The nurse should administer the medication ordered for moderate to severe pain. Hydrocodone has not been effective and should not be given. The nurse bases the drug selection on the patient's evaluation of pain, not on which drug the patient wants. The provider has written an order with nursing evaluations specified, so there is no need for the nurse to notify the provider.

(Mod 13) The nurse is caring for a patient who is receiving clopidogrel (Plavix). The patient calls the nurse to report flulike symptoms. The nurse notes a heart rate of 76 beats per minute, a blood pressure of 110/76 mm Hg, and a respiratory rate of 20 breaths per minute. The nurse suspects that the patient is experiencing which condition? a. Anaphylaxis b. An expected drug side effect c. Hemorrhage d. Possible MI

ANS: B b. An expected drug side effect Flulike symptoms are a side effect of clopidogrel. The patient has normal vital signs. Anaphylaxis is characterized by respiratory distress and hypotension. Hemorrhage is characterized by tachycardia and hypotension.

9. A 75-year-old patient is readmitted to the hospital to treat recurrent pneumonia. The patient had been discharged home with a prescription for antibiotics 5 days prior. The nurse admitting this patient will take which initial action? a. Ask the patient about over-the-counter drug use. b. Ask the patient how many doses of the antibiotic have been taken. c. Discuss increasing the antibiotic dose with the provider. d. Obtain an order for a creatinine clearance test.

ANS: B b. Ask the patient how many doses of the antibiotic have been taken There are many reasons for non-adherence to a drug regimen in an older patient, so if a patient is readmitted, the nurse should first ascertain whether or not the medications have been used. Asking the patient how many doses have been taken will help to assess this. If it is determined that the patient is taking the drug as ordered, the other steps may be taken.

(Mod 9) 9. The nurse is performing an admission assessment on a stable patient admitted after a motor vehicle accident. The patient reports having "bad pain." What will the nurse do first? a. Administer acetaminophen (Tylenol). b. Ask the patient to rate the pain on a 1 to 10 scale. c. Attempt to determine what type of pain the patient has. d. Request an order for an intravenous opioid analgesic.

ANS: B b. Ask the patient to rate the pain on a 1 to 10 scale. To ascertain severity of pain, the nurse should ask the patient to rate the pain on a scale of 1 to 10. Further assessments include location and type of pain. Pain medication should be given after the severity of pain is assessed so that an appropriate analgesic may be given.

(Mod 13) The nurse is caring for a postoperative patient who is receiving alteplase tPA (Activase) after developing a blood clot. The nurse notes a heart rate of 110 beats per minute and a blood pressure of 90/60 mm Hg. The nurse will perform which action? a. Ask the patient about itching or shortness of breath. b. Assess the surgical dressing for bleeding. c. Evaluate the patient's urine output and fluid intake. d. Recheck the patient's vital signs in 15 minutes.

ANS: B b. Assess the surgical dressing for bleeding. Tachycardia and hypotension indicate bleeding. The nurse should check the patient's surgical dressing to assess for bleeding. These signs do not indicate anaphylaxis. They may indicate dehydration, but bleeding is the more likely cause of fluid volume deficit. The nurse should continue to evaluate vital signs, but it is imperative that the nurse assess the patient to explore the potential cause.

(Mod 8) 5. The nurse is caring for an 80-year-old patient who has Alzheimer's disease who will begin taking rivastigmine (Exelon). What will the nurse include in the plan of care for this patient? a. Administer the drug once daily. b. Assist the patient to stand and walk. c. Give the drug with food to increase absorption. d. Use nonsteroidal anti-inflammatory drugs (NSAIDs) instead of acetaminophen for pain.

ANS: B b. Assist the patient to stand and walk. Patients taking rivastigmine for Alzheimer's disease are at risk for falls and loss of balance. Caregivers should assist with standing and walking. The drug is taken twice daily, and it should be taken on an empty stomach. NSAIDs increase gastrointestinal side effects.

9. The nurse is caring for a patient who has asthma. The provider has ordered an albuterol metered-dose inhaler (MDI), 2 puffs q4 to 6h PRN wheezing. The patient's last dose was 4 hours ago. What is the nurse's next action? a. Administer 2 puffs of albuterol with the MDI. b. Auscultate the patient's lung sounds. c. Give the albuterol if the patient reports wheezing. d. Give the medication and evaluate its effectiveness.

ANS: B b. Auscultate the patient's lung sounds. The albuterol is to be given PRN if the patient is wheezing. The nurse should assess breath sounds and give the medication if the patient is wheezing. Even is the patient reports wheezing, the nurse should make and document an assessment.

(Mod 8) 3. The nurse is caring for a patient who has myasthenia gravis (MG) and is receiving pyridostigmine bromide (Mestinon). The nurse notes ptosis of both eyelids and observes that the patient has difficulty swallowing. What action will the nurse perform next? a. Contact the provider to request an order for atropine sulfate. b. Contact the provider to request an order for edrophonium chloride (Tensilon). c. Report signs of cholinergic crisis to the provider. d. Report signs of myasthenic crisis to the provider.

ANS: B b. Contact the provider to request an order for edrophonium chloride (Tensilon). Overdosing and underdoing of AChE inhibitors have similar symptoms: muscle weakness, dyspnea, and dysphagia. Edrophonium may be used to diagnose MG or to distinguish between myasthenic crisis and cholinergic crisis since it is a very short-acting AChE inhibitor. When given, if the symptoms are alleviated, the cause is myasthenic crisis; if symptoms worsen, it is cholinergic crisis. Since patients can have similar symptoms, the nurse cannot report one or the other to the provider without more information.

(Mod 8) 3. The nurse administers subcutaneous epinephrine to a patient who is experiencing an anaphylactic reaction. The nurse should expect to monitor the patient for which symptom? a. Bradycardia b. Decreased urine output c. Hypotension d. Nausea and vomiting

ANS: B b. Decreased urine output Epinephrine can cause renal vasoconstriction and thereby reduce renal perfusion and decrease urinary output. Epinephrine causes tachycardia and elevates blood pressure. Nausea and vomiting are not expected to occur.

12. The nurse is caring for a 78-year-old patient who lives independently. The patient will begin a new drug regimen that requires taking multiple drugs at various times per day. Which intervention is appropriate for the nurse to implement with this patient? a. Ask the patient's family members to monitor the patient's drug regimen. b. Develop a log to record the times each drug will be taken. c. Reinforce the need to take the drugs as scheduled. d. Write the medication administration times on each prescription label.

ANS: B b. Develop a log to record the times each drug will be taken. The patient should be advised to keep a medication record of drugs and when they will be taken. The patient is independent, and this helps maintain independence. Family member support is essential when older patients are confused. Reinforcing information without providing a means to keep track of the medications does not necessarily improve compliance. Writing medication times on prescription labels does not help to organize the medication schedule.

13. The nurse is preparing to administer an intramuscular medication to a 4-year-old child who starts to cry and screams, "I don't want a shot!" What is the nurse's next action? a. Acknowledge that shots hurt and tell the child to be brave. b. Engage the child in a conversation about preschool and favorite activities. c. Enlist the assistance of another nurse to help restrain the child. d. Explain to the child that it will only hurt for a few seconds.

ANS: B b. Engage the child in a conversation about preschool and favorite activities. Distraction may be used for pain and anxiety control in this age group. Engaging the child in a conversation may district the child from the anxiety of the imminent injection. It is not correct to tell the child to be brave since this belittles the feelings expressed by the child. Preschool children have a limited sense of time, so telling the child that the pain will only last a few seconds may not be effective. Restraining the child with other staff should be used last after other methods have failed.

(Mod 9) 15. The nurse assesses an older patient 60 minutes after administering 4 mg of intravenous morphine sulfate (MS) for postoperative pain. The patient's analgesia order is for 2 to 5 mg of MS IV every 2 hours. The nurse notes that the patient is lying very still. The patient's heart rate is 96 beats per minute, respiratory rate is 14 breaths per minute, and blood pressure is 140/90 mm Hg. When asked to rate the level of pain, the patient replies "just a 5." The nurse will perform which action? a. Give 3 mg of MS at the next dose. b. Give 5 mg of MS at the next dose. c. Request an order for an oral opioid to give now. d. Request an order for acetaminophen to give now.

ANS: B b. Give 5 mg of MS at the next dose. Older patients often minimize pain when asked, so the nurse should evaluate nonverbal cues to pain such as elevated heart rate and blood pressure and the fact that the patient is lying very still. The nurse should increase the dose the next time the pain medication is given.

(Mod 8) 13. The nurse is preparing to administer benztropine (Cogentin) to a patient who has Parkinson's disease. When performing an assessment, which aspect of the patient's history would cause the nurse to hold the medication and notify the provider? a. Asthma b. Glaucoma c. Irritable bowel syndrome d. Motion sickness

ANS: B b. Glaucoma Patients who have glaucoma should not take anticholinergic medications. (Cholinergics cause miosis to increase drainage to decrease IOP, Anticholinergics will do the opposite)

(Mod 8) 3. The nurse is preparing to administer a first dose of benztropine (Cogentin) to a patient diagnosed with parkinsonism. The nurse would notify the patient's provider if the patient had a history of which condition? a. Asthma b. Glaucoma c. Hypertension d. Irritable bowel disease

ANS: B b. Glaucoma Patients with a history of glaucoma should not take anticholinergic medications. Anticholinergics are not contraindicated in patients who have asthma, hypertension, or irritable bowel disease.

5. The nurse is caring for an 82-year-old patient who takes digoxin to treat chronic atrial fibrillation. When caring for this patient, to monitor for drug side effects, what will the nurse will carefully assess? a. Blood pressure b. Heart rate c. Oxygen saturation d. Respiratory rate

ANS: B b. Heart rate Most of digoxin is eliminated by the kidneys, so a decline in kidney function can cause digoxin accumulation, which can cause bradycardia. Digoxin should be be given to any patient with a pulse less than 60 beat per minute.

(Mod 12) 12. The nurse is preparing to administer an angiotensin-converting enzyme (ACE) inhibitor to a patient who has hypertension. The nurse notes peripheral edema and swelling of the patient's lips. The patient has a blood pressure of 160/80 mm Hg and a heart rate of 76 beats per minute. What is the nurse's next action? a. Administer the dose and observe carefully for hypotension. b. Hold the dose and notify the provider of a hypersensitivity reaction. c. Notify the provider and request an order for a diuretic medication. d. Request an order for serum electrolytes and renal function tests.

ANS: B b. Hold the dose and notify the provider of a hypersensitivity reaction. The patient has signs of angioedema which indicates a hypersensitivity reaction. The nurse should hold the dose and notify the provider. Giving the dose will make the reaction more serious. These are not signs of edema, so a diuretic is not indicated. Electrolytes and renal function tests are not indicated.

3. The nurse assumes care for an infant who is showing signs of drug toxicity to a drug given several hours prior. The nurse checks the dose and confirms that the dose is consistent with standard dosing guidelines. Which characteristic of the drug will likely explain this response in this patient? a. It is acidic. b. It is highly protein-bound. c. It is not fat-soluble. d. It is water-soluble.

ANS: B b. It is highly protein-bound. With fewer protein-binding sites, there is more active drug available. This requires a reduction in the dose for infants. Drugs that are acidic are not as readily absorbed in infants, since their gastric pH tends to be more alkaline. Infants have a lower proportion of body fat; fat-soluble drugs would need to be decreased to prevent toxicity. Until about age 2 years of age, pediatric patients require larger than usual doses of water-soluble drugs to achieve therapeutic effects.

3. A Native American patient has just been diagnosed with diabetes mellitus. The nurse preparing a teaching plan for this patient understands that which aspect of the disease and disease management may be most difficult for this patient? a. Body image changes b. Management of meal and medication schedules c. Perception of the disease as punishment from God d. The sense of dependence on others

ANS: B b. Management of meal and medication schedules Non-European cultural groups such as those of Native American descent have less dependence on time schedules. Disease management will likely focus more on present concerns about alleviating current discomfort and less on measures to promote long-term wellness or treat a chronic illness.

(Mod 6) 9. The nurse assumes care for a patient who is currently receiving a dose of intravenous vancomycin (Vancocin) infusing at 20 mg/min. The nurse notes red blotches on the patient's face, neck, and chest and assesses a blood pressure of 80/55 mm Hg. Which action will the nurse take? a. Request an order for IV epinephrine to treat anaphylactic shock. b. Slow the infusion to 10 mg/min and observe the patient closely. c. Stop the infusion and obtain an order for a BUN and serum creatinine. d. Suspect Stevens-Johnson syndrome and notify the provider immediately.

ANS: B b. Slow the infusion to 10 mg/min and observe the patient closely. When vancomycin is infused too rapidly, "red man" syndrome may occur; the rate should be 10 mg/min to prevent this. This is a toxic reaction, not an allergic one, so epinephrine is not indicated. Stevens-Johnson syndrome is characterized by a rash and fever. Red man syndrome is not related to renal function. book has a similar question with the answer of stopping the infusion and notifying HCP (with an answer choice of slowing infusion)

14. The provider has ordered amoxicillin with clavulanate (Augmentin) for a child who has otitis media. The child's parent asks why this drug is necessary when amoxicillin is less expensive. The nurse will explain that clavulanate is added to amoxicillin because it a. binds with albumin to increase the amount of available amoxicillin. b. broadens the spectrum of amoxicillin by inhibiting bacterial enzymes. c. inhibits hepatic blood flow, leading to increased serum drug levels of amoxicillin. d. inhibits the excretion of amoxicillin by interfering with renal function.

ANS: B b. broadens the spectrum of amoxicillin by inhibiting bacterial enzymes. Clavulanate is a bacterial enzyme inhibitor, specifically beta-lactamase, which inactivates amoxicillin. When added to amoxicillin, it broadens the antibacterial spectrum.

1. The nurse is preparing to administer a medication to a 6-month-old infant. The nurse will monitor closely for signs of drug toxicity based on the knowledge that, compared to adults, infants have a. an increased percentage of total body fat. b. immature hepatic and renal function. c. more protein receptor sites. d. more rapid gastrointestinal transit time.

ANS: B b. immature hepatic and renal function. The liver and kidneys are the primary organs for metabolism and excretion and are immature in infants. This allows drugs to accumulate and increases the risk for drug toxicity. Infants have a lower proportion of body fat than adults and fewer protein receptors. They do have more rapid gastrointestinal transit time, but this decreases the amount of drug absorbed.

2. The nurse is preparing to administer two intravenous medications that should not be given using the same IV tubing. The nurse understands that this is because of drug a. adverse reactions. b. incompatibility. c. interactions. d. potentiation.

ANS: B b. incompatibility. Drugs that are incompatible cannot be mixed together in solution and cannot be mixed in a syringe, IV bag, or other artificial environment. Adverse reactions are symptoms occurring from drug effects. Drug interactions occur in vivo. Potentiation is when one drug causes an enhanced response in another drug.

13. The nurse gives a medication to a patient with a history of liver disease. The nurse will monitor this patient for a. decreased drug effects. b. increased drug effects. c. decreased therapeutic range. d. increased therapeutic range.

ANS: B b. increased drug effects. Liver diseases such as cirrhosis and hepatitis alter drug metabolism by inhibiting the drug-metabolizing enzymes in the liver. When the drug metabolism rate is decreased, excess drug accumulation can occur and lead to toxicity.

10. The nurse is caring for a patient who receives theophylline, which has a narrow therapeutic index. The patient has been receiving cimetidine but will stop taking that drug in 2 days. In 2 days, the nurse will observe the patient closely for a. decreased effectiveness of theophylline. b. increased effectiveness of theophylline. c. decreased toxicity of theophylline. d. prolonged effectiveness of theophylline.

ANS: B b. increased effectiveness of theophylline. Cimetidine is an enzyme inhibitor that decreases the metabolism of drugs such as theophylline. If the cimetidine is discontinued, the theophylline dose should be decreased to avoid toxicity. The nurse should observe the patient for increased theophylline effects. I think the answer is A. If cimetidine is an inhibitor, it's decreasing the metabolism of theophylline. When you stop taking an inhibitor, more of the drug will be metabolized. Less effect.

7. A patient has been taking warfarin (Coumadin), which is highly protein-bound. The patient will begin taking gemfibrozil, which is also highly protein-bound. The nurse will observe the patient closely for a. decreased effects of warfarin. b. increased effects of warfarin. c. decreased effects of gemfibrozil. d. decreased effects of both drugs.

ANS: B b. increased effects of warfarin. The addition of a highly protein-bound drug will compete with warfarin for protein- binding sites, releasing more free warfarin into the system, increasing drug effects and increasing the chance of toxicity.

12. A school-age child will begin taking a medication to be administered 5 mL three times daily. The child's parent tells the nurse that, with a previous use of the drug, the child repeatedly forgot to bring the medication home from school, resulting in missed evening doses. What will the nurse recommend? a. Asking the provider if the medication may be taken before school, after school, and at bedtime b. Putting a note on the child's locker to encourage the child to take responsibility for medication administration c. Asking the provider if 7.5 mL may be taken in the morning and 7.5 mL may be taken in the evening so that the correct amount is given daily d. Taking the noon dose to school every day and giving it to the school nurse to administer

ANS: C c. Asking the provider if 7.5 mL may be taken in the morning and 7.5 mL may be taken in the evening so that the correct amount is given jail. For busy families with school-age children, it may be necessary to adjust the medication schedule to one that fits their schedule. The nurse should ask the provider if a revised schedule is possible. In this case, the revised schedule would involve not taking the medication at school. Putting a note on the locker is not likely to be effective. It is not correct to adjust the dose. This sounds like the right answer is actually a. b/c in c, the dose is adjusted.

(Mod 13) The nurse is assessing a patient prior to administering thrombolytic therapy. Which is an important assessment for this patient? a. Determining whether the patient has a history of diabetes b. Finding out about a history of renal disease c. Assessing which medications are taken for discomfort d. Assessing whether the patient eats green, leafy vegetables

ANS: C c. Assessing which medications are taken for discomfort Patients who take aspirin or NSAIDs should be monitored closely for excessive bleeding when given thrombolytics. There are no contraindications or precautions for patients with diabetes or renal disease. Foods rich in vitamin K are of concern for patients taking warfarin.

4. The pediatric nurse reviews a hand-written medication order which reads, "09/16/2013, acetaminophen 160 mg (5 mL) PO q4h for fever." What will the nurse do next? a. Administer the drug when indicated. b. Ask the provider to confirm if dose is correct for the patient's age. c. Clarify the nursing assessments necessary for giving a dose. d. Contact the provider to request patient allergy information.

ANS: C c. Clarify the nursing assessments necessary for giving a dose. This order contains all components except the level of temperature necessary to justify a dose of an antipyretic (assessment). The patient's age and allergy information are part of the medical record data base.

8. The nurse is caring for a patient who will have surgery that morning. The patient usually takes an antihypertensive medication every morning. The patient has been NPO since midnight. What action will the nurse perform? a. Ask the patient to swallow the pill without water. b. Give the medication with a small sip of water. c. Consult the provider and surgeon about giving the medication. d. Hold the medication until after the patient's surgery.

ANS: C c. Consult the provider and surgeon about giving the medication. The patient's provider or surgeon should determine the importance of giving the medication along with the safety of administering it prior to anesthesia. The nurse should not give the medication with or without the provider's permission.

(Mod 8) 11. Which cholinesterase inhibitor would be prescribed for a patient who has Alzheimer's disease? a. Ambenonium chloride (Myletase) b. Benztropine (Cogentin) c. Donepezil HCl (Aricept) d. Neostigmine methylsulfate (Prostigmin)

ANS: C c. Donepezil HCl (Aricept) Donepezil is used to treat Alzheimer's disease. Ambenonium and neostigmine are used to treat myasthenia gravis. Benztropine is used to treat Parkinson's disease.

(Mod 9) 18. The nurse is caring for a 6-year-old child who had surgery that morning. The child is awake and lying very still in bed and won't respond when the nurse asks about pain. The nurse will perform which action? a. Ask the child to rate the pain on a scale of 1 to 10. b. Encourage the child to request pain medication when needed. c. Evaluate the child's pain using an "ouch" scale. d. Plan to administer pain medication if the child begins to cry.

ANS: C c. Evaluate the child's pain using an "ouch" scale. Some children will not verbalize discomfort even when they have severe pain because they fear injections. Nurses may use an "ouch" scale or a faces scale to evaluate pain if the child won't respond. Waiting for severe pain is not appropriate

8. The nurse is caring for a 5-year-old child. The child is taking a drug that has a known therapeutic range in adults, and the nurse checks that the ordered dose is correct and notes that the child's serum drug level is within normal limits. The child complains of a headache, which is a common sign of toxicity for this drug. Which action will the nurse take? a. Administer the drug since the drug levels are normal. b. Attribute the headache to non-drug causes. c. Hold the next dose and contact the provider. d. Request an order for an analgesic medication.

ANS: C c. Hold the next dose and contact the provider. The therapeutic ranges established for many drug levels are based on adult studies, so it is important for the nurse to assess pediatric patients in conjunction with monitoring drug levels. The nurse should notify the provider of the reaction. Because headaches are a symptom of toxicity for this drug, the nurse should not ignore the symptom.

14. A 14-year-old female who has type 1 diabetes mellitus that has been well-controlled for several years is admitted to the hospital for treatment of severe hyperglycemia. The patient's lab values indicate poor glycemic control for the past 3 months. The nurse caring for this patient will suspect which cause for the change in diabetic control? a. Adolescent rebellion and noncompliance b. Changes in cognitive function c. Hormonal fluctuations d. Possible experimentation with drugs or alcohol

ANS: C c. Hormonal fluctuations In adolescence, hormonal changes and growth spurts may necessitate changes in medication dosages; many children with chronic illnesses require dosage adjustments in the early teen years.

(Mod 8) 13. A patient will be discharged on beta blockers. Which skill is essential for the nurse to teach the patient's family? a. How to prepare a low-sodium diet b. Assessments to detect fluid retention c. How to monitor heart rate and blood pressure d. Early signs of changing level of consciousness

ANS: C c. How to monitor heart rate and blood pressure Because of the action and side effects of beta blockers, heart rate and blood pressure should be monitored frequently.

(Mod 8) 1. A nursing student asks the nurse to differentiate the pathology of Alzheimer's disease from that of Parkinson's disease. Which description is correct? a. Alzheimer's disease involves a possible excess of acetylcholine and neuritic plaques. b. Alzheimer's disease is caused by decreased amounts of dopamine and degeneration of cholinergic neurons. c. Parkinson's disease is characterized by an imbalance of dopamine and acetylcholine. d. Parkinson's disease involves increased dopamine production and decreased acetylcholine.

ANS: C c. Parkinson's disease is characterized by an imbalance of dopamine and acetylcholine. Parkinson's disease (PD) is characterized by an imbalance of dopamine (DA) and acetylcholine (ACh) caused by an unexplained degeneration of the dopaminergic neurons allowing the excitatory response of acetylcholine to exceed the inhibitory response of dopamine. Alzheimer's disease (AD) may result from decreased ACh, degeneration of cholinergic neurons, and neuritic plaques. Dopamine does not appear to play a role in Alzheimer's disease.

1. The nurse provides teaching about the sedative side effects of a medication ordered to be given at 8:00 PM daily. The patient works a 7:00 PM to 7:00 AM shift. The nurse explores options including taking the medication at 8:00 AM instead of in the evening. Which QSEN competency do the nurse's actions best demonstrate? a. Collaboration and teamwork b. Evidence-based practice c. Patient-centered care d. Patient safety

ANS: C c. Patient-centered care Patient centered care recognizes the patient as the source of control and provides care based on respect for the patient's preferences, values, and needs.

(Mod 8) 8. The nurse is caring for a patient who has recently begun taking atenolol (Tenormin) to treat hypertension. The patient reports dizziness, nausea, vomiting, and decreased libido since beginning the medication. What will the nurse do? a. Hold the next dose until the provider can be notified of these side effects. b. Reassure the patient that these symptoms are common and not worrisome. c. Recommend that the patient discuss these effects with the provider. d. Suggest that the patient request a different beta-adrenergic blocker.

ANS: C c. Recommend that the patient discuss these effects with the provider. Beta-adrenergic blockers can cause these side effects, which are often dose-related. Patients experiencing these side effects should be encouraged to discuss them with their providers. Beta blockers should not be discontinued abruptly, or rebound symptoms may occur. Since symptoms may be dose-related, reassuring the patient is not correct. All beta blockers have similar side effects.

(Mod 8) 4. The nurse is caring for a patient who is receiving trihexyphenidyl (Artane) to treat parkinsonism. The patient reports having a dry mouth, and the nurse notes a urine output of 300 mL in the past 8 hours. Which action will the nurse perform? a. Encourage increased oral fluids. b. Obtain an order for intravenous fluids. c. Report the urine output to the provider. d. Request an order for renal function tests.

ANS: C c. Report the urine output to the provider. Urinary retention can occur with anticholinergic medications. Dry mouth is a harmless side effect. The nurse should report the lower than expected urine output to the provider. Increasing fluid intake will not increase urine output in the patient with urinary retention. Renal function tests are not indicated since this is a neuromuscular problem of the bladder caused by the medication.

(Mod 12) 11. A patient who has recently begun taking captopril (Capoten) to treat hypertension calls a clinic to report a persistent cough. The nurse will perform which action? a. Instruct the patient to go to an emergency department because this is a hypersensitivity reaction. b. Reassure the patient that this side effect is nothing to worry about and will diminish over time. c. Schedule an appointment with the provider to discuss changing to an angiotensin II receptor blocker (ARB). d. Tell the patient to stop taking the drug immediately since this is a serious side effect of this drug.

ANS: C c. Schedule an appointment with the provider to discuss changing to an angiotensin II receptor blocker (ARB). An angiotensin-converting enzyme (ACE) inhibitor, such as captopril, can cause a constant, irritated cough. The cough will stop with discontinuation of the drug, and many patients can switch to an ARB medication. It does not indicate a hypersensitivity reaction. The cough will not diminish while still taking the drug. The patient does not need to stop taking the drug immediately.

11. An older patient must learn to administer a medication using a device that requires manual dexterity. The patient becomes frustrated and expresses lack of self-confidence in performing this task. Which action will the nurse perform next? a. Ask the patient to keep trying until the skill is learned. b. Provide written instructions with illustrations showing each step of the skill. c. Schedule multiple sessions and practice each step separately. d. Teach the procedure to family members who can administer the medication for the patient.

ANS: C c. Schedule multiple sessions and practice each step separately. Nurses should be sensitive to patient's level of frustration when teaching skills. In this case, breaking the steps down into individual parts will help with this patient's frustration level.

1. The nurse is caring for an older adult patient who is receiving multiple medications. When monitoring this patient for potential drug toxicity, the nurse should review which lab values closely? a. Complete blood count and serum glucose levels b. Pancreatic enzymes and urinalysis c. Serum creatinine and liver function tests (LFTs) d. Serum lipids and electrolytes

ANS: C c. Serum creatinine and liver function tests (LFTs) With liver and kidney dysfunction, the efficacy of drugs is generally increased and may cause toxicity. The nurse should review serum creatinine levels to monitor renal function and LFTs to monitor hepatic function. The other lab tests may be ordered for specific drugs if they affect those body systems.

(Mod 12) 3. A patient has a blood pressure of 155/95 mm Hg. The nurse understands that this patient's risk of cardiovascular disease is _____ greater than normal. a. two times b. three times c. four times d. six times

ANS: C c. four times Cardiovascular disease (CVD) risk doubles with each increase of 20/10 mm Hg above normal, starting at 115/75 mm Hg. This patient's blood pressure is 40/20 above normal, which increases the risk four times. A blood pressure of 135/85 would be two times greater. The patient's risk would still be four times greater with a blood pressure of 155/70 or 130/95, since systolic and diastolic blood hypertension are each powerful predictors of CVD.

13. A high-school student regularly forgets to use a twice-daily inhaled corticosteroid to prevent asthma flares and is repeatedly admitted to the hospital. The child's parent tells the nurse that the child has been told that forgetting to take the medication causes frequent hospitalizations. The nurse will a. encourage the child to take responsibility for taking the medication. b. reinforce the need to take prescribed medications to avoid hospitalizations. c. suggest putting the inhaler with the child's toothbrush to use before brushing teeth. d. suggest that the child's parents administer the medication to increase compliance.

ANS: C c. suggest putting the inhaler with the child's toothbrush to use before brushing teeth. It is important to empower patients to take responsibility for managing medications. Putting the medication with the toothbrush can help this child remember to use it. Telling the child to take medications and reminding the child that failure to do so results in hospitalization is not working. Asking the child's parents to administer the medication does not empower the adolescent to take responsibility.

15. The nurse is teaching a 15-year-old female patient and her parents about an antibiotic the adolescent will begin taking. The drug is known to decrease the effectiveness of oral contraceptive pills (OCPs). The nurse will a. ask the adolescent and her parents whether she is taking OCPs. b. tell her parents privately that pregnancy may occur if she is taking OCPs. c. tell her privately that the medication may decrease the effectiveness of OCPs. d. warn her and her parents that she may get pregnant if she is relying on OCPs.

ANS: C c. tell her privately that the medication may decrease the effectiveness of OCPs. When soliciting adolescent health histories, the nurse should consider issues related to sexual practices and should provide privacy when asking sensitive questions or giving sensitive information. The other actions do not allow for patient privacy.

(Mod 6) 3. The nurse is caring for several patients who are receiving antibiotics. Which order will the nurse question? a. Azithromycin (Zithromax) 500 mg IV in 500 mL fluid b. Azithromycin (Zithromax) 500 mg PO once daily c. Erythromycin 300 mg IM QID d. Erythromycin 300 mg PO QID

ANS: C c. Erythromycin 300 mg IM QID Erythromycin and other macrolides should not be given intramuscularly because they cause painful tissue irritation.

(Mod 6) 4. The nurse is caring for a patient who is receiving a high dose of intravenous azithromycin to treat an infection. The patient is also taking acetaminophen for pain. The nurse should expect to review which lab values when monitoring for this drug's side effects? a. Complete blood counts b. Electrolytes c. Liver enzymes d. Urinalysis

ANS: C c. Liver enzymes High doses of macrolides, when taken with other, potentially hepatotoxic drugs such as acetaminophen may cause hepatotoxicity, so liver enzymes should be carefully monitored.

(Mod 11) 2. A patient is prescribed metformin. Which is a side effect/adverse effect common to metformin? a. Seizures b. Constipation c. Bitter or metallic taste d. Polyuria and polydipsia

ANS: C Metformin has a bitter or metallic taste. Seizures, constipation, polyuria, and polydipsia are not side effects/adverse effects of metformin.

(Mod 11) 4. A nurse gives a patient NPH insulin at 8:00 am. At 2:00 pm the nurse finds the patient extremely lethargic but conscious. The patient is diaphoretic and slightly combative. The nurse should a. call the health care provider. b. ensure that the patient has a meal. c. provide the patient with 4 ounces of orange juice. d. administer the next dose of insulin.

ANS: C NPH is an intermediate-acting insulin that peaks in 6 to 12 hours. Because the patient is conscious, it is most important for the nurse to provide the orange juice to prevent a possible hypoglycemic reaction (insulin shock).

(Mod 11) 6. When teaching the patient about the storage of insulin, which statement will the nurse include? a. Keep the insulin in the freezer. b. Warm the insulin in the microwave before administration. c. Do not place insulin in sunlight or a warm environment. d. Open insulin vials lose their strength after one year.

ANS: C Unopened insulin vials are refrigerated until needed. Once an insulin vial has been opened, it may be kept (1) at room temperature for 1 month or (2) in the refrigerator for 3 months. Insulin is less irritating to the tissues when injected at room temperature. Insulin vials should not be put in the freezer. In addition, insulin vials should not be placed in direct sunlight or in a high-temperature area. Prefilled syringes should be stored in the refrigerator and should be used within 1 to 2 weeks. Opened insulin vials lose their strength after approximately 3 months.

3. The nurse is teaching a patient who will be discharged home with a prescription for an enteric-coated tablet. Which statement by the patient indicates understanding of the teaching? a. "I may crush the tablet and put it in applesauce to improve absorption." b. "I should consume acidic foods to enhance absorption of this medication." c. "I should expect a delay in onset of the drug's effects after taking the tablet." d. "I should take this medication with high-fat foods to improve its action."

ANS: C c. "I should expect a delay in onset of the drug's effects after taking the tablet." Enteric-coated tablets resist disintegration in the acidic environment of the stomach and disintegrate when they reach the small intestine. There is usually some delay in onset of actions after taking these medications. Enteric-coated tablets should not be crushed or chewed, which would alter the time and location of absorption. Acidic foods will not enhance the absorption of the medication. The patient should not to eat high-fat food before ingesting an enteric-coated tablet, because high-fat foods decrease the absorption rate.

14. The nurse gives 800 mg of a drug that has a half-life of 8 hours. How much drug will be left in the body in 24 hours if no additional drug is given? a. None b. 50 mg c. 100 mg d. 200 mg

ANS: C c. 100 mg Eight hours after the drug is given, there will be 400 mg left. Eight hours after that (16 hours), there will be 200 mg left. At 24 hours, there will be 100 mg left.

1. The nurse is performing a pain assessment on a patient of Asian descent. The patient does not describe the pain when asked to do so and looks away from the nurse. What will the nurse do next? a. Ask the patient's family member to evaluate the patient's pain. b. Conclude that the patient's pain is minimal. c. Evaluate the patient's non-verbal pain cues. d. Suspect that the patient is experiencing severe pain.

ANS: C c. Evaluate the patient's non-verbal pain cues. Patients of Asian descent might speak in soft tones and avoid direct eye contact while being comfortable with long silences. It is not correct to ask family members to evaluate pain. Without assessment of non-verbal cues, the nurse cannot determine whether the pain is minimal or severe.

4. A patient who is of Filipino descent is admitted to the hospital. The nurse goes to the room to start intravenous fluids and to perform an admission assessment and finds several family members in the room. Which action by the nurse is appropriate? a. Ask the family to wait in the hallway until the admission tasks are completed. b. Determine which family member is the family patriarch and address questions to him. c. Invite family members to assist with appropriate tasks during the admission process. d. Provide chairs for family members and ask them to stay seated during the admission.

ANS: C c. Invite family members to assist with appropriate tasks during the admission process. In general, the Filipino culture expects that family members will stay at a patient's bedside and participate in his or her care. The nurse should include the family in appropriate tasks. It is not correct to ask the family to wait in the hall or to sit in chairs and not participate. Filipino families do not necessarily depend on family patriarchs.

2. The nurse is preparing to administer an oral medication and wants to ensure a rapid drug action. Which form of the medication will the nurse administer? a. Capsule b. Enteric-coated pill c. Liquid suspension d. Tablet

ANS: C c. Liquid suspension Liquid drugs are already in solution, which is the form necessary for absorption in the GI tract. The other forms must disintegrate into small particles and then dissolve before being absorbed.

1. Which drugs will go through a pharmaceutic phase after it is administered? a. Intramuscular cephalosporins b. Intravenous vasopressors c. Oral analgesics

ANS: C c. Oral analgesics When drugs are administered parenterally, there is no pharmaceutic phase, which occurs when a drug becomes a solution that can cross the biologic membrane.

2. The nurse is preparing to discuss long-term care needs with a patient newly diagnosed with a chronic disease. The patient is of Latin American descent. The nurse will plan to take which action when teaching this patient? a. Discussing long-term outcomes associated with compliance of the prescribed regimen b. Highlighting various traditional healing practices that will not be effective for this patient's care c. Providing factual information and answering all questions as they arise d. Providing teaching in increments, allowing periods of silence to allow assimilation of information

ANS: C c. Providing factual information and answering all questions as they arise The nurse should provide factual information and answer questions. Persons of Latin American descent have less dependence on time schedules and do not tend to have a future orientation. They are not comfortable with periods of silence. Nurses should be receptive to traditional healing practices and seek ways to include those in care when they do not hinder safe and effective care; highlighting practices that won't work may convey a lack of respect for these traditions.

17. The nurse is caring for a patient who has taken an overdose of aspirin several hours prior. The provider orders sodium bicarbonate to be given. The nurse understands that this drug is given for which purpose? a. To counter the toxic effects of the aspirin b. To decrease the half-life of the aspirin c. To increase the excretion of the aspirin d. To neutralize the acid of the aspirin

ANS: C c. To increase the excretion of the aspirin Aspirin is a weak acid and is more readily excreted in alkaline urine. Sodium bicarbonate alkalizes the urine. It does not act as an antidote to aspirin, decrease the half-life, or neutralize its pH.

6. A nurse is preparing to administer an oral drug that is best absorbed in an acidic environment. How will the nurse give the drug? a. On an empty stomach b. With a full glass of water c. With food d. With high-fat food

ANS: C c. With food Food can stimulate the production of gastric acid so medications requiring an acidic environment should be given with a meal. High-fat foods are useful for drugs that are lipid soluble.

11. The nurse is caring for a patient who takes digoxin to treat heart failure. The provider orders furosemide to treat edema. The nurse will monitor the patient for digitalis toxicity because of a. adverse drug reactions caused by giving these drugs in combination. b. altered hepatic blood flow caused by the furosemide. c. changes in reabsorption of water and electrolytes in the kidneys. d. additive effects of these two drugs given together.

ANS: C c. changes in reabsorption of water and electrolytes in the kidneys. Diuretics such as furosemide promote water and sodium excretion from the renal tubules, especially sodium and potassium. Hypokalemia can result, and this will enhance the action of digoxin, and digitalis toxicity can occur.

4. A patient who is newly diagnosed with type 1 diabetes mellitus asks why insulin must be given by subcutaneous injection instead of by mouth. The nurse will explain that this is because a. absorption is diminished by the first-pass effects in the liver. b. absorption is faster when insulin is given subcutaneously. c. digestive enzymes in the gastrointestinal tract prevent absorption. d. the oral form is less predictable with more adverse effects.

ANS: C c. digestive enzymes in the gastrointestinal tract prevent absorption Insulin, growth hormones, and other protein-based drugs are destroyed in the small intestine by digestive enzymes and must be given parenterally. Because insulin is destroyed by digestive enzymes, it would not make it to the liver for metabolism with a first-pass effect. Subcutaneous tissue has fewer blood vessels, so absorption is slower in such tissue. Insulin is given subcutaneously because it is desirable to have it absorb slowly.

12. A patient has been taking a drug that has a protein-binding effect of 75%. The provider adds a new medication that has a protein-binding effect of 90%. The nurse will expect a. decreased drug effects of the first drug. b. decreased therapeutic range of the first drug. c. increased drug effects of the first drug. d. increased therapeutic range of the first drug.

ANS: C c. increased drug effects of the first drug. Adding another highly protein-bound drug will displace the first drug from protein-binding sites and release more free drug increasing the drug's effects. This does not alter the therapeutic range, which is the serum level between drug effectiveness and toxicity

15. The nurse assesses a patient who is receiving morphine sulfate intravenously using a patient-controlled analgesia pump. The nurse notes somnolence and respiratory depression, which are signs of morphine toxicity. The nurse will prepare to administer naloxone (Narcan) because it a. has synergistic effects with morphine. b. is a narcotic agonist. c. is a narcotic antagonist. d. potentiates narcotic effects.

ANS: C c. is a narcotic antagonist. Naloxone is a narcotic antagonist, meaning that it reverses the effects of morphine by blocking morphine receptor sites.

6. The nurse is caring for an African-American patient who appears to understand instructions for self-care but does not carry out basic self-care tasks. The nurse understands that the patient may a. be poorly educated and lack basic comprehension skills. b. need more time and personal space to assimilate what is taught. c. require the use of culturally appropriate words and phrases when teaching. d. view illness as punishment and lack desire to change the outcome.

ANS: C c. require the use of culturally appropriate words and phrases when teaching. African Americans may use a common style of speaking. This vernacular English may be quite different in some cases from standard English, so if things are misunderstood, it is possible that vernacular terminology may need to be used. This vernacular English does not mean that patients are poorly educated or uncommunicative. African Americans do not tend to need more space and do not necessarily view illness as punishment.

(Mod 6) 10. The nurse is caring for a patient who will begin taking doxycycline to treat an infection. The nurse should plan to give this medication a. 1 hour before or 2 hours after a meal. b. with an antacid to minimize GI irritation. c. with food to improve absorption. d. with small sips of water.

ANS: C c. with food to improve absorption. Doxycycline is a lipid-soluble tetracycline and is better absorbed when taken with milk products and food. It should not be taken on an empty stomach. Antacids impair absorption of tetracyclines. Small sips of water are not necessarily indicated.

(Mod 6) 3. The nurse is teaching a nursing student about the minimal effective concentration (MEC) of antibiotics. Which statement by the nursing student indicates understanding of this concept? a. "A serum drug level greater than the MEC ensures that the drug is bacteriostatic." b. "A serum drug level greater than the MEC broadens the spectrum of the drug." c. "A serum drug level greater than the MEC helps eradicate bacterial infections." d. "A serum drug level greater than the MEC increases the therapeutic index."

ANS: C c. "A serum drug level greater than the MEC helps eradicate bacterial infections." The MEC is the minimum amount of drug needed to halt the growth of a microorganism. A level greater than the MEC helps eradicate infections. Drugs at or above the MEC are usually bactericidal, not bacteriostatic. Raising the drug level does not usually broaden the spectrum or increase the therapeutic index of a drug.

(Mod 6) 9. The nurse is teaching a patient who will be discharged home from the hospital to take amoxicillin (Amoxil) twice daily for 10 days. Which statement by the nurse is correct? a. "Discontinue the antibiotic when your temperature returns to normal and your symptoms have improved." b. "If diarrhea occurs, stop taking the drug immediately and contact your provider." c. "Stop taking the drug and notify your provider if you develop a rash while taking this drug." d. "You may save any unused antibiotic to use if your symptoms recur."

ANS: C c. "Stop taking the drug and notify your provider if you develop a rash while taking this drug." Patients who develop signs of allergy, such as rash, should notify their provider before continuing medication therapy. Patients should be counseled to continue taking their antibiotics until completion of the prescribed regimen even when they feel well. Diarrhea is an adverse effect but does not warrant cessation of the drug. Before deciding to stop taking a medication due to a side effect, encourage the patient to contact the provider first. Patients should discard any unused antibiotic.

(Mod 6) 18. The nurse is providing discharge teaching for a patient who will receive oral levofloxacin (Levaquin) to treat pneumonia. The patient takes an oral hypoglycemic medication and uses over-the-counter (OTC) antacids to treat occasional heartburn. The patient reports frequent arthritis pain and takes acetaminophen when needed. Which statement by the nurse is correct when teaching this patient? a. "You may take antacids with levofloxacin to decrease gastrointestinal upset." b. "You may take nonsteroidal anti-inflammatory medications (NSAIDs) for arthritis pain." c. "You should monitor your serum glucose more closely while taking levofloxacin." d. "You should take levofloxacin on an empty stomach to improve absorption."

ANS: C c. "You should monitor your serum glucose more closely while taking levofloxacin." Levofloxacin may increase the effects of oral hypoglycemic medications, so patients taking these should be advised to monitor their serum glucose levels closely. Antacids decrease the absorption of levofloxacin. NSAIDs taken with levofloxacin can cause central nervous system reactions, including seizures.

(Mod 6) 8. The nurse is caring for a patient who is receiving sulfadiazine. The nurse knows that this patient's daily fluid intake should be at least which amount? a. 1000 mL/day b. 1200 mL/day c. 2000 mL/day d. 2400 mL/day

ANS: C c. 2000 mL/day To prevent crystalluria, patients should consume at least 2000 mL/day.

(Mod 6) 3. A patient is diagnosed with influenza and will begin taking a neuraminidase inhibitor. The nurse knows that this drug is effective when taken within how many hours of onset of flu symptoms? a. 12 hours b. 24 hours c. 48 hours d. 72 hours

ANS: C c. 48 hours Neuraminidase inhibitors, such as zanamivir and oseltamivir, should be taken within 48 hours of onset of symptoms for best effect.

(Mod 6) 6. The nurse caring for a patient who has tuberculosis and who is taking isoniazid, rifampin, and streptomycin reviews the medical record and notes the patient's sputum cultures reveal resistance to streptomycin. The nurse will anticipate that the provider will take which action? a. Add ethambutol (Myambutol). b. Change the streptomycin to clarithromycin. c. Change the streptomycin to kanamycin. d. Order renal function tests.

ANS: C c. Change the streptomycin to kanamycin. The patient's current regimen is first-phase treatment. If resistance to streptomycin develops, the provider can change to kanamycin or to ciprofloxacin. Ethambutol is added if there is resistance to isoniazid. Clarithromycin is used during phase II. Renal function tests are not indicated.

(Mod 6) 5. The nurse is preparing to administer amoxicillin (Amoxil) to a patient and learns that the patient previously experienced a rash when taking penicillin. Which action will the nurse take? a. Administer the amoxicillin and have epinephrine available. b. Ask the provider to order an antihistamine. c. Contact the provider to discuss using a different antibiotic. d. Request an order for a beta-lactamase resistant drug.

ANS: C c. Contact the provider to discuss using a different antibiotic. Patients who have previously experienced manifestations of allergy to a penicillin should not use penicillins again unless necessary. The nurse should contact the provider to discuss using another antibiotic from a different class. Epinephrine and antihistamines are useful when patients are experiencing allergic reactions, depending on severity.

(Mod 13) The nurse is teaching a patient who will begin taking warfarin (Coumadin) for atrial fibrillation. Which statement by the patient indicates understanding of the teaching? a. "I should eat plenty of green, leafy vegetables while taking this drug." b. "I should take a nonsteroidal anti-inflammatory drug (NSAID) instead of acetaminophen for pain or fever." c. "I will take cimetidine (Tagamet) to prevent gastric irritation and bleeding." d. "I will tell my dentist that I am taking this medication." .

ANS: D d. "I will tell my dentist that I am taking this medication." Patients taking warfarin should tell their dentists that they are taking the medication because of the increased risk for bleeding. Patients should avoid foods high in vitamin K, which can decrease the effects of warfarin. Patients should not take NSAIDs or cimetidine (Tagamet) because they can displace warfarin from protein-binding sites.

(Mod 8) 15. The patient has been started on a treatment regimen that includes atenolol (Tenormin) and complains to the nurse of feeling weak. Which is the best response from the nurse? a. "I will hold your next dose of the medication." b. "You may need an increase in your next dose of the medication." c. "This is an adverse reaction to the medication. I will stop the drug." d. "This is a side effect of the medication. I will notify your physician."

ANS: D d. "This is a side effect of the medication. I will notify your physician." Weakness can be a side effect of atenolol. Beta blockers should not be stopped abruptly.

(Mod 8) 10. The nurse is teaching a patient about the use of an anticholinergic medication. What information will the nurse include when teaching this patient about this medication? a. "Check your heart rate frequently to monitor for bradycardia." b. "Drink extra fluids while you are taking this medication." c. "Rise from a chair slowly to avoid dizziness when taking this drug." d. "Use gum or lozenges to decrease dry mouth caused by this drug."

ANS: D d. "Use gum or lozenges to decrease dry mouth caused by this drug." Anticholinergic medications cause dry mouth, so patients should be advised to use gum or lozenges to counter this side effect. Anticholinergics cause increased heart rate and increased blood pressure. Anticholinergics can cause urinary retention so patients should not increase fluid intake.

(Mod 12) 10. The nurse is teaching a patient who has hypertension about long-term management of the disease and a beta blocker. The patient reports typically consuming 1 to 2 glasses of wine each evening with meals. How will the nurse respond? a. "Beta blockers and wine cause a reflex hypertension." b. "Four to 6 ounces of wine is considered safe with these medications." c. "Wine in moderation helps you relax and get better blood pressure control." d. "Wine increases the hypotensive effects of the beta blocker."

ANS: D d. "Wine increases the hypotensive effects of the beta blocker." Patients who take beta blockers should avoid all alcohol because it increases the hypotensive effects. It does not cause reflex hypertension.

(Mod 12) A 65-year-old patient has a blood pressure of 155/95 mm Hg. The nurse understands that with treatment, the goal for this patient's blood pressure is a. 120/80 mm Hg b. 130/89 mm Hg c. 140/90 mm Hg d. 150/90 mm Hg

ANS: D d. 150/90 mm Hg According to the Joint National Committee (JNC) 8, a blood pressure of 140/90 is the goal for the population younger than 60 years, with a target of 150/90 for those above 60. The guideline for normal blood pressure is less than 120/80 mm Hg.

5. The nurse is caring for a 20-kg child who is ordered to receive amoxicillin 400 mg PO TID for 10 days. The nurse reviews the drug information and notes that the correct dose of amoxicillin is 40 to 50 mg/kg/day in two to three divided doses. Which action by the nurse is correct? a. Adjust the drug dose based on drug manufacturer dosing information. b. Administer the medication as ordered. c. Ask the pharmacist to double-check that the dose is correct. d. Contact the provider and ask whether the drug should be given BID instead of TID.

ANS: D d. Contact the provider and ask whether the drug should be given BID instead of TID. The correct range for this drug for this child is 800 to 1000 mg per day. If 400 mg were administered TID, it would result in 1200 mg per day being administered. Twice daily (BID) dosing would be in the correct range.

11. An older patient takes ibuprofen for arthritis pain. The patient tells the nurse that the ibuprofen causes gastrointestinal (GI) upset. Which action will the nurse take with this patient? a. Ask the provider about having the patient take a different medication. b. Instruct the patient to cut the ibuprofen dose in half to avoid GI upset. c. Explain that all drugs have adverse effects. d. Explore options to help decrease the drug side effects.

ANS: D d. Explore options to help decrease the drug side effects. Older adults are more likely to experience drug side effects, and nurses should be aware of measures that may decrease these side effects and thus improve adherence.

(Mod 8) 17. The patient has been ordered to receive Sudafed to treat nasal congestion. The nurse performing an admission assessment learns that the patient has diabetes mellitus. What action is appropriate for the nurse to take? a. Administer the medication as ordered. b. Contact the provider to discuss a lower dose. c. Give the medication and monitor serum glucose closely. d. Hold the medication and contact the provider.

ANS: D d. Hold the medication and contact the provider. Sympathetic drugs should not be taken by patients with diabetes. The medication should not be given. (sympathetic drugs increase glycogenolysis in liver to increase blood sugar)

(Mod 8) 3. The nurse is preparing to administer bethanechol (Urecholine) to a patient who is experiencing urinary retention. The nurse notes that the patient has a blood pressure of 90/60 mm Hg and a heart rate of 98 beats per minute. The nurse will perform which action? a. Administer the drug and monitor urine output. b. Administer the medication and monitor vital signs frequently. c. Give the medication and notify the provider of the increased heart rate. d. Hold the medication and notify the provider of the decreased blood pressure.

ANS: D d. Hold the medication and notify the provider of the decreased blood pressure. Side effects of this medication are a decrease in the pulse rate and vasodilation, which can exacerbate bradycardia and hypotension. The nurse should hold the drug and notify the provider.

9. The nurse is developing a plan of care for a patient who has chronic lung disease and hypoxia. The patient has been admitted for increased oxygen needs above a baseline of 2 L/min. The nurse develops a goal stating, "The patient will have oxygen saturations of > 95% on room air at the time of discharge from the hospital." What is wrong with this goal? a. It cannot be evaluated. b. It is not measurable. c. It is not patient-centered. d. It is not realistic.

ANS: D d. It is not realistic. This goal is not realistic because the patient is not usually on room air and should not be expected to attain that goal by discharge from this hospitalization.

(Mod 9) 11. One hour after receiving intravenous morphine sulfate, a patient reports generalized itching. The nurse assesses the patient and notes clear breath sounds, no rash, respirations of 14 breaths per minute, a heart rate of 68 beats per minute, and a blood pressure of 110/70 mm Hg. Which action will the nurse take? a. Administer naloxone to reverse opiate overdose. b. Have resuscitation equipment available at the bedside. c. Prepare an epinephrine injection in case of an anaphylactic reaction. d. Reassure the patient that this is a common side effect of this drug.

ANS: D d. Reassure the patient that this is a common side effect of this drug. Pruritis is a common opioid side effect and can be managed with diphenhydramine. Patients developing anaphylaxis will have urticaria and hypotension, and these patients will need epinephrine and resuscitation. Respiratory depression is a sign of morphine overdose, which will require naloxone.

(Mod 8) 2. A patient who has Parkinson's disease is being treated with the anticholinergic medication benztropine (Cogentin). The nurse will tell the patient that this drug will have which effect? a. Helping the patient to walk faster b. Improving mental function c. Minimizing symptoms of bradykinesia d. Reducing some of the tremors

ANS: D d. Reducing some of the tremors Benztropine is given to reduce rigidity and some of the tremors. It does not enhance walking or reduce bradykinesia or improve mental function.

10. The nurse is developing a teaching plan for an elderly patient who will begin taking an antihypertensive drug that causes dizziness and orthostatic hypotension. Which nursing diagnosis is appropriate for this patient? a. Deficient knowledge related to drug side effects b. Ineffective health maintenance related to age c. Readiness for enhanced knowledge related to medication side effects d. Risk for injury related to side effects of the medication

ANS: D d. Risk for injury related to side effects of the medication This patient has an increased risk for injury because of drug side effects, so this is an appropriate nursing diagnosis.

1. When the nurse practices the "5-plus-5" rights of medication administration, what does it ensure? a. Adequate information is given b. Cost-effective use of medications c. Informed consent for drug administration d. Safe administration of medications

ANS: D d. Safe administration of medications The "5-plus-5" rights ensure that the nurse has considered all of the details of safe medication administration. Giving information to patients and obtaining informed consent are part of the 5-plus-5 rights. Cost effectiveness is not part of the 5-plus-5 rights.

(Mod 9) 8. The emergency department nurse is caring for a patient who has received morphine sulfate for severe pain following an injury. The nurse performs a drug history and learns that the patient takes St. John's wort for symptoms of depression. The nurse will observe this patient closely for an increase in which opioid adverse effect? a. Constipation b. Pruritis c. Respiratory depression d. Sedation

ANS: D d. Sedation St. John's wort can increase the sedative effects of opioids. It does not enhance other side effects.

4. The nurse is caring for an older patient who is taking 25 mg per day of hydrochlorothiazide. The nurse will closely monitor which lab value in this patient? a. Coagulation studies b. White blood count c. Liver function tests d. Serum potassium

ANS: D d. Serum potassium Older patients who take doses of hydrochlorothiazide between 25 to 50 mg/day have increased risk of electrolyte imbalances, so potassium should be monitored closely.

(Mod 13) A patient who is taking clopidogrel (Plavix) and aspirin is preparing for orthopedic surgery. The nurse will consult with the surgeon and provide which instruction to the patient? a. Continue taking aspirin and stop taking clopidogrel 2 weeks prior to surgery. b. Continue taking clopidogrel and stop taking aspirin 5 days prior to surgery. c. Continue both medications to prevent thromboembolic events during surgery. d. Stop taking both medications 7 days prior to surgery.

ANS: D d. Stop taking both medications 7 days prior to surgery. Because both drugs can prolong bleeding time, patients should discontinue the drugs 7 days prior to surgery

(Mod 8) 15. A patient who has Parkinson's disease will begin treatment with benztropine (Cogentin). Which symptom of Parkinson's disease would be a contraindication for this drug? a. Drooling b. Muscle rigidity c. Muscle weakness d. Tardive dyskinesia

ANS: D d. Tardive dyskinesia Tardive dyskinesia is a contraindication for this drug.

9. The nurse is preparing to give a 7-year-old child a bitter-tasting oral medication. The child asks the nurse if the medicine tastes bad. To help the child take this medication, which action will the nurse take? a. Allow the child to delay taking the medication until the parent arrives. b. Enlist the assistance of other staff to help restrain the child. c. Tell the child that it doesn't taste bad if it is swallowed quickly. d. Tell the child that it tastes bad and offer a choice of beverages to drink afterwards.

ANS: D d. Tell the child that it tastes bad and offer a choice of beverages to drink afterwards. School-age children should be permitted more control, involvement in the process, and honest information. The nurse should tell the child the truth and offer the child a choice about what to drink to wash down the medicine. Medications must be given on schedule, so allowing the child is a choice about when to take a medication is not acceptable. Restraining a child should not be used unless other methods have failed. Telling the child the medication doesn't taste bad is not honest and will reduce the child's trust in the nurse.

(Mod 8) 6. The charge nurse observes a nurse administer undiluted intravenous pyridostigmine bromide (Mestinon) at a rate of 0.8 mg/min. The charge nurse will stop the infusion and perform which action? a. Administer atropine sulfate to prevent cholinergic crisis. b. Monitor the patient closely for respiratory distress. c. Suggest that the nurse dilute the medication with colloidal fluids. d. Tell the nurse to slow the rate of infusion of the pyridostigmine.

ANS: D d. Tell the nurse to slow the rate of infusion of the pyridostigmine. When given, IV pyridostigmine should be administered undiluted at a rate of 0.5 mg/min and should not be added to IV fluids. It is not necessary to administer atropine, since the patient is not symptomatic of cholinergic crisis.

4. The parent is concerned about giving a child medication because of the lack of knowledge about the effects of drugs on children. The nurse discusses legislation passed in 2002 and 2003 about pediatric pharmacology. Which is true about these laws? a. They forbid providers from prescribing medications unless they have been FDA- approved for use in children. b. They mandate consistent, evidence-based dosing guidelines for use in children. c. They provide federal grants to fund pediatric pharmaceutical research. d. They require drug manufacturers to study pediatric medication use.

ANS: D d. They require drug manufacturers to study pediatric medication use. In 2003, a law known as the Pediatric Research Equity Act joined the Best Pharmaceuticals Act of 2002 to require drug manufacturers to study pediatric medication use and offer incentives for pediatric pharmacology research. Providers are not forbidden to prescribe drugs in children that are not FDA-approved. The laws do not mandate the use of evidence-based guidelines and do not provide grants to fund research.

(Mod 8) 7. The nurse is preparing to administer the anticholinergic medication benztropine (Cogentin) to a patient who has Parkinson's disease. The nurse understands that this drug is used primarily for which purpose? a. To decrease drooling and excessive salivation b. To improve mobility and muscle strength c. To prevent urinary retention d. To suppress tremors and muscle rigidity

ANS: D d. To suppress tremors and muscle rigidity Antiparkinson-anticholinergic drugs are used mainly to reduce tremors and muscle rigidity.

12. A preschool-age child has moderate dehydration and needs a rapid bolus of fluids. To provide atraumatic care and administer fluids most effectively, what action will the nurse take? a. Apply a eutectic mixture of local anesthetic (EMLA) just before inserting an intravenous line. b. Ask the child's parents to restrain the child during venipuncture so fluids may be administered. c. Request an order for nasogastric (NG) fluids to avoid the trauma of venipuncture. d. Use a powdered lidocaine preparation prior to insertion of the intravenous needle.

ANS: D d. Use a powdered lidocaine preparation prior to insertion of the intravenous needle. One method to ensure atraumatic care is through the use of topical analgesics before IV injections. Powdered lidocaine preparations are effective in reducing the pain and fear associated with inverse procedures, such as venipuncture. EMLA is useful if applied 1 to 2.5 hours prior to IV insertion. Asking parents to restrain the child for a painful procedure can cause stress and anxiety for both the child and the parents. NG fluids are traumatic and are uncomfortable long past the insertion of the NG tube.

8. A 75-year-old patient will be discharged home with a prescription for an opioid analgesic. To help the patient minimize adverse effects, what will the nurse recommend for this patient? a. Sucking on lozenges to moisten oral mucosa b. Taking an antacid with each dose c. Taking the medication on an empty stomach d. Using a stool softener

ANS: D d. Using a stool softener Opioid analgesics can cause constipation. Stool softeners can help minimize this effect. Opioids do not cause dry mouth. Drug absorption may be decreased with an antacid. Opioid analgesics should be taken with food or milk to decrease gastrointestinal irritation.

2. In order to ensure that a medication is given to the right patient, the nurse must perform which action? a. Ask the patient to spell their last name. b. Match the patient with a photo ID. c. Swipe a bar code on the patient's ID bracelet. d. Verify the patient using two identifiers.

ANS: D d. Verify the patient using two identifiers. The Joint Commission requires two forms of identification before medication administration. Patients are asked to state their name and date of birth. Some, but not all institutions, use photos and bar codes to aid in identification.

(Mod 12) 1. A patient is diagnosed with borderline hypertension and states a desire to make lifestyle changes to avoid needing to take medication. The nurse will recommend which changes? a. Changing from weight bearing exercise to yoga b. Decreased fluid intake and increased potassium intake c. Stress reduction and increased protein intake d. Weight reduction and decreased sodium intake

ANS: D d. Weight reduction and decreased sodium intake Weight loss decreases the stress on the heart and the afterload. Decreasing salt intake decreases the amount of retained fluid. Changing to yoga from weight-bearing exercise, limiting fluids, and increasing potassium are not indicated. Stress reduction is recommended, but increasing protein is not.

(Mod 12) 2. A patient has a blood pressure of 135/85 mm Hg on three separate occasions. The nurse understands that this patient should be treated with a. a beta blocker. b. a diuretic and a beta blocker. c. a diuretic. d. lifestyle changes

ANS: D d. lifestyle changes Prehypertension is defined as a systolic pressure of 120 to 139 and a diastolic pressure between 80 and 89. Drug therapy is recommended if the blood pressure is greater than 20/10 over the goal, which would be140/90. Prehypertension is generally treated first with lifestyle changes.

(Mod 8) 11. A patient who has Raynaud's disease will begin taking an alpha-adrenergic blocker. The patient asks the nurse how the drug works to treat symptoms. The nurse explains that alpha-adrenergic blockers treat Raynaud's disease by causing a. decreased peripheral vascular resistance. b. orthostatic hypotension. c. reflex tachycardia. d. vasodilation.

ANS: D d. vasodilation. Alpha-adrenergic blockers can be used to treat peripheral vascular disease because they cause vasodilation.

16. The nurse is teaching a patient about a drug that causes photosensitivity. Which statement by the patient indicates a need for further teaching? a. "I should apply sunscreen with a sun protection factor greater than 15." b. "I should avoid sunlight when possible while taking this drug." c. "I will wear protective clothing when I am outdoors." d. "I will wear sunglasses even while I am indoors."

ANS: D d. "I will wear sunglasses even while I am indoors." Drugs that cause photosensitivity make sunburn more likely, so patients should stay out of the sun, wear protective clothing, and use sunscreen with an SPF greater than 15. It is not necessary to wear sunglasses indoors.

19. The nurse is educating the parent of a 20-month-old toddler about over-the-counter (OTC) products to treat cold symptoms. Which statement by the parent indicates understanding of the teaching? a. "I should check with the provider for proper dosing instructions." b. "OTC medications are less potent and have minimal side effects." c. "OTC medications can be given to children younger than 2 years old." d. "Using OTC medications may prevent accurate diagnosis of respiratory illness."

ANS: D d. "Using OTC medications may prevent accurate diagnosis of respiratory illness." OTC cold medications can mask symptoms and prevent accurate diagnosis of potentially serious illnesses. Their use in children is not recommended.

(Mod 6) 14. The nurse is preparing to administer intravenous gentamicin to an infant through an intermittent needle. The nurse notes that the infant has not had a wet diaper for several hours. The nurse will perform which action? a. Administer the medication and give the infant extra oral fluids. b. Contact the provider to request adding intravenous fluids when giving the medication. c. Give the medication and obtain a serum peak drug level 45 minutes after the dose. d. Hold the dose and contact the provider to request a serum trough drug level.

ANS: D d. Hold the dose and contact the provider to request a serum trough drug level. Gentamicin can cause nephrotoxicity. When changes in urine output occur, the provider should be notified, and serum trough levels should be obtained to make sure the drug is not at a toxic level. If the drug level is determined to be safe, giving extra fluids either orally or intravenously may be indicated. Serum peak levels give information about therapeutic levels but are not a substitution for avoiding nephrotoxicity in the face of possible oliguria. 90

(Mod 6) 7. The nurse is preparing to administer the first dose of an antibiotic to a patient admitted for a urinary tract infection. Which action is most important prior to administering the antibiotic? a. Administering a small test dose to determine whether hypersensitivity exists b. Having epinephrine available in the event of a severe hypersensitivity reaction c. Monitoring baseline vital signs, including temperature and blood pressure d. Obtaining a specimen for culture and sensitivity

ANS: D d. Obtaining a specimen for culture and sensitivity To obtain the most accurate culture, the specimen should be obtained before antibiotic therapy begins. It is important to obtain cultures when possible in order to correctly identify the organism and help determine which antibiotic will be most effective. Administering test doses to determine hypersensitivity is sometimes done when there is a strong suspicion of allergy when a particular antibiotic is needed. Epinephrine is kept close at hand when there is a strong suspicion of allergy.

8. The nurse is caring for a postoperative patient who is of Asian descent. The patient reports little relief from pain even while taking an opioid analgesic containing codeine and acetaminophen. What does the nurse suspect that this patient is exhibiting? a. Drug-seeking behavior b. Heightened pain perception c. Poor understanding of expected drug effects d. Rapid metabolism of one of the drug's components

ANS: D d. Rapid metabolism of one of the drug's components Certain classifications of medications have different effects in individuals whose genetic markers are predominantly characteristic of a certain biologic group. Persons of Asian descent may have a decreased response to some drugs because they are more likely to have higher levels of CYP2D6 enzymes.

(Mod 6) 11. The nurse is preparing to give a dose of a cephalosporin medication to a patient who has been receiving the antibiotic for 2 weeks. The nurse notes ulcers on the patient's tongue and buccal mucosa. Which action will the nurse take? a. Hold the drug and notify the provider. b. Obtain an order to culture the oral lesions. c. Gather emergency equipment to prepare for anaphylaxis. d. Report a possible superinfection side effect of the cephalosporin.

ANS: D d. Report a possible superinfection side effect of the cephalosporin. The patient's symptoms may indicate a superinfection and should be reported to the physician so it can be treated; however, the drug does not need to be held. It is not necessary to culture the lesions. The symptoms do not indicate impending anaphylaxis.

(Mod 6) 6. The nurse is preparing to administer an antibiotic to a patient who has been receiving the antibiotic for 2 days after a culture was obtained. The nurse notes increased erythema and swelling, and the patient has a persistent high fever of 39° C. What is the nurse's next action? a. Administer the antibiotic as ordered. b. Contact the provider to request another culture. c. Discuss the need to add a second antibiotic with the provider. d. Review the sensitivity results from the patient's culture.

ANS: D d. Review the sensitivity results from the patient's culture. The sensitivity results from the patient's culture will reveal whether the organism is sensitive or resistant to a particular antibiotic. The patient is not responding to the antibiotic being given, so the antibiotic should be held and the provider notified. Another culture is not indicated. Antibiotics should be added only when indicated by the sensitivity.

(Mod 6) 16. The nurse is preparing to begin a medication regimen for a patient who will receive intravenous ampicillin and gentamicin. Which is an important nursing action? a. Administer each antibiotic to infuse over 15 to 20 minutes. b. Order serum peak and trough levels of ampicillin. c. Prepare the schedule so that the drugs are given at the same time. d. Set up separate tubing sets for each drug labeled with the drug name and date.

ANS: D d. Set up separate tubing sets for each drug labeled with the drug name and date. Intravenous aminoglycosides can be given with penicillins and cephalosporins but should not be mixed in the same container. The IV line should be flushed between antibiotics, or separate tubing sets may be set up. Gentamicin must be infused over 30 to 60 minutes. It is not necessary to measure ampicillin peak and trough levels. Giving the drugs at the same time increases the risk of mixing them together.

(Mod 6) 7. A patient who is taking isoniazid (INH) as part of a two-drug tuberculosis treatment regimen reports tingling of the fingers and toes. The nurse will recommend discussing which treatment with the provider? a. Adding pyrazinamide b. Changing to ethambutol c. Increasing oral fluid intake d. Taking pyridoxine (B6)

ANS: D d. Taking pyridoxine (B6) Peripheral neuropathy is an adverse reaction to INH, so pyridoxine is usually given to prevent this. It is not necessary to change medications. Increasing fluids will not help with this.

(Mod 6) 11. The nurse is caring for a 7-year-old patient who will receive oral antibiotics. Which antibiotic order will the nurse question for this patient? a. Azithromycin (Zithromax) b. Clarithromycin (Biaxin) c. Clindamycin (Cleocin) d. Tetracycline (Sumycin)

ANS: D d. Tetracycline (Sumycin) Tetracyclines should not be given to children younger than 8 years of age because they irreversibly discolor the permanent teeth.

10. The nurse is preparing to administer an oral liquid medication to an 11-month-old child who is fussy and uncooperative. Which action will the nurse take to facilitate giving this medication? a. Adding honey to the medication to improve the taste b. Putting the medication in the infant's formula c. Requesting an injectable form of the medication d. Using a syringe and allowing the parent to give the medication

ANS:D d. using a syringe and allowing the parent to give the medication When possible, family members or caregivers should be solicited to assist in medication administration. Infants should not receive honey because of the risk of botulism. A syringe allows more control over the amount of medication in the infant's mouth and should be used. Mixing the medication in a bottle requires ensuring that the infant takes the entire bottle in order to get the medication dose. Using an injectable form of medication is more traumatic and should be used only when an oral route is not possible or is contraindicated.

(Mod 10) 8. A nurse is evaluating a client's understanding of a new prescription for clonidine (Catapres). Which of the following statements by the client indicates an understanding of the teaching? A.) Taking this medication will help reduce my craving for heroin B.) While taking this medication, I should keep a pack of sugarless gum C.) I can expect some diarrhea because of taking this medication D.) Each does of this medication should be placed under my tongue to dissolve

B.) While taking this medication, I should keep a pack of sugarless gum My rationale: Clonidine is a alpha 2 agonist BP med. It can cause dry mouth so chewing sugarless gum or sweets may help. the link to the heroin answer choice (my guess to why this question was included in the abuse chapter): clonidine, b/c it's an alpha 2+, it's central acting, suppressing the sympathetic NS; it's used to reduce symptoms of opioid detox/withdrawal.

(Mod 10) 11. A patient who has a long history of alcohol abuse is admitted to the hospital for detoxification. In addition to medications needed to treat withdrawal symptoms, the nurse will anticipate giving intravenous A) dopamine to restore BP B) fluid blouses to treat dehydration C) glucose to prevent hypoglycemia D) thiamine to treat nutritional deficiency

D) thiamine to treat nutritional deficiency Thiamine should be given to prevent Wernicke's encephalopathy in patients treated for alcoholism. If glucose is indicated, the thiamine should be given first. Other treatments are given as indicated. TMI: chronic alcoholics are at high risk for being deficient in B12 which increases risk for Wernicke-Korsakoff Syndrome. Give thiamine 100 mg IV before administering glucose containing IVFs. Failure to do so worsens the Wernicke-Kerosakoff Syndrome. Thiamine is needed in the TCA cycle to convert pyruvate to ATP. Giving glucose w/out first giving thiamine causes glycolysis to occur, Glu -> Pyr but the Pyr can't enter the TCA cycle so it gets converted to lactic acid which builds up -> acidosis.

(Mod 10) 9. A nurse is discussing the use of methadone (dolophine) with a newly licensed nurse. Which of the following statements by the newly licensed nurse indicates a need for further teaching? A.) Methadone is a replacement for the clients opioid addiction B.) Methadone reduces the unpleasant effects associated with abstinence syndrome C.) Methadone can be used during opioid withdrawal and to maintain abstinence D.) Methadone increases the clients risk for acetaldehydle syndrome

D. Methadone increases the clients risk for acetaldehyde syndrome My rationale: Acetaldehyde syndrome is disulfiram-like reactions. Disulfiram blocks acetaldehyde metabolism by inhibiting the acetaldehyde dehydrogenase.

(Mod 11) The nurse is teaching a patient who is newly diagnosed with type 1 diabetes mellitus about insulin administration.Which statement by the patient indicates a need for further teaching? a. "I may use a chosen site daily for up to a week." b. "I should give each injection a knuckle length away from a previous injection." c. "I will not be concerned about a raised knot under my skin from injecting insulin." d. "Insulin is absorbed better from subcutaneous sites on my abdomen.

c. "I will not be concerned about a raised knot under my skin from injecting insulin." Lipohypertrophy is a raised lump or knot on the skin surface caused by repeated injections into the same site, and this can interfere with insulin absorption. Patients are encouraged to use the same site for a week, giving each injection a knuckle length away from the previous injection. Insulin absorption is greater when given in abdominal areas.


Set pelajaran terkait

Ch 16 - Mastering Financial Management Test Yourself

View Set

Pathophysiology FINAL EXAM Review

View Set

Topic 4: America and the World (WWI & the 1920s)

View Set

Unit 3 Progress Check AP Psychology

View Set

General Psychology Chapter 13, Industrial-Organizational Psychology

View Set